You are on page 1of 167

Engineering

Mechanics
SECOND EDITION

BASUDEB BHATTACHARYYA
Associate Professor
Department of Aerospace Engineering and Applied Mechanics
Indian Institute of Engineering Science and Technology
(Previously Bengal Engineering and Science University)
Shibpur

Oxford University Press is a department of the University of Oxford.
It furthers the University’s objective of excellence in research, scholarship,
and education by publishing worldwide. Oxford is a registered trade mark of
Oxford University Press in the UK and in certain other countries.

Published in India by
Oxford University Press
YMCA Library Building, 1 Jai Singh Road, New Delhi 110001, India

© Oxford University Press 2008, 2014

The moral rights of the author/s have been asserted.

First Edition published in 2008


Second Edition published in 2014

All rights reserved. No part of this publication may be reproduced, stored in


a retrieval system, or transmitted, in any form or by any means, without the
prior permission in writing of Oxford University Press, or as expressly permitted
by law, by licence, or under terms agreed with the appropriate reprographics
rights organization. Enquiries concerning reproduction outside the scope of the
above should be sent to the Rights Department, Oxford University Press, at the
address above.

You must not circulate this work in any other form


and you must impose this same condition on any acquirer.

ISBN-13: 978-0-19-809632-0
ISBN-10: 0-19-809632-1

Typeset in Times New Roman MTStd


by Time Digitech Private Limited, Noida
Printed in India by Rajkamal Electric Press, Kundli, Haryana
List of Symbols

A Area Force
a Linear acceleration, p Pitch
Coefficient of rolling resistance PE Potential energy
Crr Coefficient of rolling friction R Normal reaction,
c Damping coefficient Range of projectile
cc Damping coefficient at critical r Radius,
damping Radius of gyration,
d Distance Total number of reactions in a
E Total energy truss,
e Coefficient of restitution, Ratio of forcing frequency and
Logarithmic base, natural frequency
Unit vector S Distance,
F Force Axial force
f Cyclic frequency T Time,
fd Damped natural cyclic Force (tension)
frequency t Time
fn Natural cyclic frequency U Potential energy
G Modulus of rigidity, u Velocity
Universal constant of V, v Velocity
gravitation W Weight,
g Acceleration due to gravity Work done
H, h Height w Weight
I Second moment of area, X, x Distance,
Mass moment of inertia, Coordinate axis
Instantaneous centre of rotation Y, y Distance,
i Unit vector along x-axis Coordinate axis
J Polar moment of area Z, z Distance,
j Total number of joints in a truss Coordinate axis
j Unit vector along y-axis ␣ Angular acceleration,
k Spring constant, Increase of belt per unit force
Radius of gyration ␤ Wrap angle
k Unit vector along z-axis ␤1 Wrap angle at follower
KE Kinetic energy ␤2 Wrap angle at driver
L, l Length ␥ Weight density
M Mass, ␦ Displacement,
Moment Logarithmic decrement,
m Mass, Differential quantity
Total number of members in a ␨ Damping ratio
truss ␪ Angle
N Normal reaction ␾ Angle of friction
P Power, ␾s Angle of static friction
List of Symbols xix

␾k Angle of kinetic friction ␻ Circular frequency


␮ Coefficient of friction ␻d Damped natural circular fre-
␮s Coefficient of static friction quency
␮k Coefficient of kinetic friction ␻n Natural circular frequency
␮r Coefficient of rolling friction ⌬ Triangle,
␳ Mass density Differential quantity
␶ Time period ᭙ Volume
Preface to the Second Edition

Engineering Mechanics is the practical application of mechanics concerned


with the behaviour of bodies or a system of bodies subjected to external forces
or displacements. The main objective of this course is to help students develop
a thorough understanding of the theories and principles and thereby acquire
the analytical capability required for solving real-life problems. It is one of the
foundation courses that form the basis of many of the traditional branches of
engineering, such as aerospace, civil, and mechanical engineering. This textbook
has been developed to provide a comprehensive and consistent coverage of the
fundamentals of statics and dynamics.

About the Book


This edition provides an in-depth coverage of the basic concepts of mechanics.
The text is divided into two parts—statics and dynamics. In statics, the concept of
force systems and equilibrium is followed by the applications of statics. Dynamics
deals with the motion of particles and rigid bodies under the application of forces.
A large number of solved examples (some solved using the vector approach) with
step-by-step explanation and numerous illustrations have been included to aid in
the understanding of theoretical concepts.
The book has been revised and updated by incorporating necessary additions
and alterations, so as to cover the syllabi of almost all the standard technical
universities in India. Although this book primarily targets the undergraduate
students of engineering and technology in the first and/or sophomore year, it can
also be used to prepare for various competitive examinations at graduate level
such as IAS, IES, WBCS, and WBAS.

New to the Second Edition


O A chapter on simple lifting machines
O Additional topics such as parabolic and catenary cables, product and principal
moment of area, stability of equilibrium, equations of motion with variable
acceleration, angular impulse, and angular momentum
O Numerous illustrations, solved problems, review questions, and numerical
problems
O Two new appendices—Appendix A on the timeline of important discoveries
and inventions in classical mechanics and Appendix H on shear force and
bending moment

Extended Chapter Material


Chapter 1 Basic concepts such as different types of vectors and vector algebra
have been moved to Appendix D from this chapter. New sections on force fields;
different approaches to find the resultant of coplanar, concurrent force systems
viii Preface to the Second Edition

(triangular/parallelogram/polygon law of forces); resolution of a force into com-


ponents; resultant of coplanar, non-concurrent force systems; resultant of non-
coplanar, concurrent force systems; and resultant of non-coplanar, non-concurrent
force systems have been included.
Chapter 2 New sections on parallel shifting of forces, active and reactive forces,
and different types of support have been added to this chapter.
Chapter 3 This chapter now includes different types of trusses, elements of sus-
pension cables, and catenary and parabolic cables. New figures have been added
to show the different types of trusses.
Chapter 4 As a result of reorganization, the sections on screwjack and differ-
ential screwjack have been moved to Chapter 7. A number of new examples have
been included.
Chapter 5 This chapter has been expanded by including topics such as definition
and formulation of product moment of area, rotation of axes and principal axes,
principal moments of area, and maximum and minimum values of moments of area.
Chapter 6 New sections on virtual displacement, virtual rotation, and stability
of equilibrium and necessary conditions have been added to this chapter.
Chapter 7 This is a new chapter which discusses different types of simple lifting
machines.
Chapter 8 This chapter now includes derivation of kinematic parameters in
angular motion, equations of motion with variable acceleration, and analysis of
projectile motion along an inclined plane.
Chapter 12 A new section on angular impulse and momentum has been included.
The section on conservation of momentum has been expanded.
Appendix H A new appendix on different types of beams and shear force and
bending moment diagrams has been included.

Content and Structure


The book has been structured into two parts and has 13 chapters.

Part I: Statics
Chapter 1 provides an introduction to statics and discusses the concepts of ideal-
ization of matter, force, principle of superposition, resultant of different types of
force systems, and moment of a force and a couple. Chapter 2 discusses the static
equilibrium (both force and moment) of rigid bodies and free-body diagrams.
Chapter 3 covers the analysis of trusses, frames, and suspension cables with point
and uniformly distributed loading.
Chapter 4 defines the phenomenon of friction and discusses its probable mecha-
nism and the laws of friction. It also describes different types of friction, belt
drives, and power transmission aspects. Chapter 5 illustrates the physical properties
of surfaces and solids which are very important for studying the mechanics of
deformable bodies. Chapter 6 deals with virtual work, which is another specialized
approach to equilibrium analysis.
Preface to the Second Edition ix

Chapter 7 elucidates different types of simple lifting machines. It also explains


the different terms which are needed for analysis and problem-solving purposes.
A number of figures have been included which show the schematic diagrams of
these machines.

Part II: Dynamics


Chapter 8 introduces the kinematics of particles for both rectilinear and curvilinear
motions in various reference coordinate systems, including the analysis of projec-
tile motion. Chapter 9 briefly describes the kinematics of rigid bodies. Chapter 10
discusses the kinetics of particles and rigid bodies. D’Alembert’s principle for
linear as well as angular motion has also been presented.
Chapter 11 discusses the potential and kinetic energy of particles and rigid bodies.
Mechanical efficiency and power aspects have been discussed at length. Chapter 12
delves on impulse and momentum. Principle of conservation, different types of
impact, and change of momentum are discussed in a comprehensive manner.
Chapter 13 presents the essential fundamentals of oscillation, free vibration,
forced vibration, and pendulum.
Appendix A provides the timeline of the events that contributed to the development
of the field of classical mechanics. Appendices B–H provide the dimensions and
SI units of commonly used physical quantities, common trigonometric formulae,
formulae for differentiation and integration, properties of geometrical figures,
and homogeneous solids, vector algebra and a small discourse on gravitation and
shear force and bending moment of beams, respectively.
Appendix I includes solutions to chapter-end multiple-choice questions and nu-
merical problems.

Acknowledgements
During the revision process, I received immense help from senior professors Asok
Kumar Mallik and Dipak Sengupta. I pay my most sincere gratitude to them. I
am very much indebted to my once colleague and currently Associate Professor
at Budge Budge Institute of Technology, Kolkata, Dr Suday Kumar Ghosh, who
painstakingly solved all the numerical problems in the exercises.
In various stages of the revision work, I utilized the feedback received from
several faculty development programmes organized by OUP, India. I am thankful
to Mr Subir Pal of Books and Equipment Distributors for his promotional effort.
Last but not the least I would like to thank the editorial team at Oxford Univer-
sity Press India, who diligently and sincerely worked for the successful completion
of this project.
It has been our utmost endeavour to publish the revised edition as an error-
free one. Any suggestions and comments for the improvement of the book can
be send to me at basubec@yahoo.com.

Basudeb Bhattacharyya
Brief Contents
Foreword v
Preface to the Second Edition vii
Preface to the First Edition x
Contents xiii
List of Symbols xix

PART ONE STATICS


CHAPTER 1 Introduction to Statics 3
CHAPTER 2 Equilibrium of Forces 46
CHAPTER 3 Truss, Frames, and Cables 98
CHAPTER 4 Friction 217
CHAPTER 5 Properties of Lines, Surfaces, and
Physical Bodies 287
CHAPTER 6 Virtual Work 366
CHAPTER 7 Simple Lifting Machines 420

PART TWO DYNAMICS


CHAPTER 8 Kinematics of Particles 457
CHAPTER 9 Kinematics of Rigid Bodies 510
CHAPTER 10 Kinetics of Particles and Rigid Bodies 555
CHAPTER 11 Principle of Work, Power, and Energy 622
CHAPTER 12 Principle of Impulse and Momentum 669
CHAPTER 13 Mechanical Vibration 716

Appendix A Major Timeline of Classical Mechanics 759


Appendix B Dimensions and Units 761
Appendix C Trigonometric and Hyperbolic Relations 763
Appendix D Differentiation and Integration Formulae 764
Appendix E Properties of Geometrical Figures 767
Appendix F Properties of Homogeneous Solids 771
Appendix G Some Prerequisites and Co-requisites 775
Appendix H Advance Topic (Shear Force and Bending Moment) 780
Appendix I Answers 787
Bibliography 793
Index 795
Contents
Foreword v
Preface to the Second Edition vii
Preface to the First Edition x
Brief Contents xii
List of Symbols xix

PART ONE STATICS


1. Introduction to Statics 3
1.1 Introduction 3
1.2 Idealization of matter 4
1.2.1 Continuum 4
1.2.2 Particle 4
1.2.3 Rigid body 4
1.2.4 Deformable body 5
1.3 Space 5
1.4 Scalars and vectors 5
1.5 Force 6
1.5.1 Force field 6
1.5.2 Classification of force 7
1.5.3 Force system 7
1.6 Superposition and transmissibility of force 8
1.7 Equilibrium 9
1.8 Resultant of a force system 10
1.9 Resultant and equilibrium of a coplanar, concurrent force system 11
1.9.1 Triangular law of forces 11
1.9.2 Parallelogram law of forces 12
1.9.3 Polygon law of forces 13
1.10 Resolution of a force into components 14
1.11 Moment of a force about a point 15
1.12 Moment of a force about a line 15
1.13 Moment of a couple 16
1.14 Resultant of a coplanar, non-concurrent force system 16
1.15 Resultant of a non-coplanar, concurrent force system 17
1.16 Resultant of a non-coplanar, non-concurrent force system 18

2. Equilibrium of Forces 46
2.1 Lami’s theorem 46
2.2 Theorem of Varignon 47
2.2.1 Resultant of two like parallel forces 48
2.2.2 Resultant of two unlike parallel forces 48
2.3 Parallel shifting of forces 49
2.4 Equivalence of force and force–couple system 50
xiv Contents

2.5 Generalized equations of equilibrium 50


2.5 Generalized equations of equilibrium 50
2.6 Active and reactive forces 51
2.7 Different types of supports 51
2.8 Free-body diagrams 52

3. Truss, Frames, and Cables 98


3.1 Introduction 98
3.2 Truss 98
3.3 Elements of a truss 99
3.4 Types of truss 99
3.5 Assumptions for truss analysis 100
3.6 Determinacy, stability, and redundancy 101
3.7 Methods of analysis of a truss 102
3.7.1 Method of joints 103
3.7.2 Method of sections (Ritter’s method) 104
3.7.3 Hybrid method 105
3.8 Frames 106
3.9 Method of analysis of a frame 106
3.10 Suspension cables 107
3.11 Elements of suspension cables 107
3.12 Suspension cables subjected to point loading 108
3.13 Suspension cables subjected to uniformly distributed loading 108
3.13.1 Catenary cables 109
3.13.2 Parabolic cables 111

4. Friction 217
4.1 Introduction 217
4.2 Classification 218
4.3 Probable mechanism 218
4.4 Laws of friction 219
4.5 Coefficient of friction 219
4.6 Angle of friction 221
4.7 Angle of repose 222
4.8 Wedge friction 223
4.9 Rolling friction 223
4.10 Energy of friction 224
4.11 Belt drives 225
4.12 Types of flat belt drives 226
4.13 Length of belt drives 226
4.14 Friction in belt drives 228
4.15 Centrifugal tension in belt drives 229
4.16 Initial tension in belt drives 230
4.17 Transmission of power in belt drives 231
4.18 Condition of transmission of maximum power 231

5. Properties of Lines, Surfaces, and Physical Bodies 287


5.1 Introduction 287
5.2 Centroid, centre of mass, and centre of gravity 287
Contents xv

5.3 Analytical expressions of centroid 288


5.4 Analytical expressions of centre of mass and centre of gravity 290
5.5 Pappus–Guldinus theorems 290
5.6 Second moment of area 292
5.7 Radius of gyration 293
5.8 Perpendicular axis theorem for second moment of area 293
5.9 Parallel axis theorem for second moment of area 293
5.10 Product moment of area 294
5.11 Parallel axis theorem for product moment of area 295
5.12 Rotation of axes 295
5.13 Principal axes and principal moments of area 297
5.14 Mass moment of inertia 297
5.15 Parallel axis theorem for mass moment of inertia 298
5.16 Relationship between mass moment of inertia and second
moment of area 299
6. Virtual Work 366
6.1 Introduction 366
6.2 Work done by force and moment 366
6.3 Virtual displacement, virtual rotation, and virtual work 367
6.4 Principle of virtual work 368
6.5 Stability of equilibrium 369

7. Simple Lifting Machines 420


7.1 Introduction 420
7.2 Simple and compound machines 420
7.3 Some basic definitions 421
7.3.1 Effort and load 421
7.3.2 Mechanical advantage 421
7.3.3 Velocity ratio 422
7.3.4 Efficiency and loss of energy 422
7.3.5 Ideal machine 422
7.3.6 Ideal effort, ideal load, and loss 422
7.3.7 Reversibility and irreversibility 423
7.3.8 Law of machine 424
7.4 Inclined plane 425
7.5 Simple screwjack 425
7.6 Differential screwjack 428
7.7 System of pulleys 429
7.8 Weston differential pulley block 430
7.9 Gear pulley block 431
7.10 Simple wheel and axle 432
7.11 Wheel and differential axle 433
7.12 Worm and worm wheel 434
7.13 Crab and winch 435
7.13.1 Single purchase crab and winch 435
7.13.2 Double purchase crab and winch 437
7.14 Worm geared screwjack 438
7.15 Worm geared pulley block 439
xvi Contents

PART TWO DYNAMICS


8. Kinematics of Particles 457
8.1 Introduction 457
8.2 Kinematics and kinetics 458
8.3 Patterns of motion 458
8.4 Kinematic parameters in linear motion 459
8.5 Equations of linear motion with constant acceleration 461
8.6 Kinematic parameters in angular motion 462
8.7 Equations of angular motion with constant acceleration 463
8.8 Equations of linear motion with variable acceleration 463
8.9 Relative motion along linear path 465
8.10 Relative motion along curvilinear path 466
8.11 Velocity and acceleration in polar reference system 467
8.12 Velocity and acceleration in n–t reference system 468
8.13 Analysis of projectile motion 468
8.13.1 Along horizontal plane 469
8.13.2 Along inclined plane 470

9. Kinematics of Rigid Bodies 510


9.1 Introduction 510
9.2 Properties of rigid body motion 510
9.3 Governing kinematic equations of rigid body motion 512
9.4 Fixed axis rotation of a rigid body 514
9.5 Contact of two rotating rigid bodies 514
9.6 Contact of a rotating rigid body with a translating rigid body 515
9.7 Instantaneous centre of rotation 515

10. Kinetics of Particles and Rigid Bodies 555


10.1 Introduction 555
10.2 Newton’s laws of motion 555
10.3 Forcing functions and kinematic parameters 557
10.4 D’Alembert’s principle 561
10.5 Pure translation of a rigid body 562
10.6 Fixed axis rotation of a rigid body 563
10.7 Planar motion of a rigid body 564
11. Principle of Work, Power, and Energy 622
11.1 Introduction 622
11.2 Work done and its varieties 623
11.3 Potential energy 624
11.4 Conservative force and potential energy 625
11.5 Work–energy principle for particle 627
11.6 Work done for rigid body 627
11.7 Kinetic energy of rigid body 628
11.8 Work–energy principle for rigid body 629
11.9 Principle of conservation of energy 629
11.10 Power 630
Contents xvii

12. Principle of Impulse and Momentum 669


12.1 Introduction 669
12.2 Linear impulse and momentum 669
12.3 Angular impulse and momentum 671
12.4 Principle of conservation of momentum 672
12.5 Impact and its variety 673
12.6 Mechanism of direct central impact 674
12.7 Coefficient of restitution 675
12.8 Perfectly elastic collision 676
12.9 Inelastic or semi-elastic collision 677
12.10 Loss of energy in direct central impact 677
12.11 Mechanism of oblique central impact 678

13. Mechanical Vibration 716


13.1 Vibration 716
13.2 Classification of vibration 716
13.3 Damping and vibration 717
13.4 Features of vibrating systems 717
13.5 Free vibration without damping 718
13.6 Free vibration with damping 719
13.7 Forced vibration without damping 721
13.8 Forced vibration with damping 722
13.9 Pendulum motion 723
13.9.1 Simple pendulum 724
13.9.2 Compound pendulum 725
13.9.3 Torsional pendulum 726

Appendix A Major Timeline of Classical Mechanics 759


Appendix B Dimensions and Units 761
Appendix C Trigonometric and Hyperbolic Relations 763
Appendix D Differentiation and Integration Formulae 764
Appendix E Properties of Geometrical Figures 767
Appendix F Properties of Homogeneous Solids 771
Appendix G Some Prerequisites and Co-requisites 775
Appendix H Advance Topic (Shear Force and Bending Moment) 780
Appendix I Answers 787
Bibliography 793
Index 795
PART one
STATICS

CHAPTER 1
Introduction to Statics

CHAPTER 2
Equilibrium of Forces

CHAPTER 3
Truss, Frames, and Cables

CHAPTER 4
Friction

CHAPTER 5
Properties of Lines, Surfaces, and
Physical Bodies

CHAPTER 6
Virtual Work

CHAPTER 7
Simple Lifting Machines
1

C H A P T E R
Introduction to Statics

Key Concepts
z Definition of continuum, particle, rigid body, and deformable body
z Discussion on force and force fields
z Discussion on classification of force and force system
z Discussion on triangular, parallelogram, and polygon law of forces
z Computation of moment of a force about a point, about a line, and moment
of a couple
z Computation of equilibrium of coplanar, non-concurrent force system
z Computation of equilibrium of non-coplanar, concurrent force system
z Computation of equilibrium of non-coplanar, non-concurrent force system

1.1 INTRODUCTION
Mechanics is the physical science dealing with the dynamic behaviour of
material bodies subjected to external stimuli. It is one of the core subjects of
all engineering courses. Engineering mechanics is a need-oriented course in
mechanics, especially tailored for engineers. Being one of the oldest branches
of physical sciences, mechanics can be broadly classified as follows:

Kinetics

Kinematics
4 Engineering Mechanics

Statics deals with the conditions of equilibrium of bodies at rest, subjected


to a particular force system. We can see the application of the fundamental
principles of statics in the construction of ancient Egyptian pyramids and of
some temples of Babylon. Archimedes (287–212 BC) is believed to be the first
person who documented the works on statics. Later, Newton, Varignon, Stevinus,
Pappus, St. Venant, and Einstein contributed significantly to the development
of principles which form the framework for modern-day analysis.

1.2 IDEALIZATION OF MATTER


A body is a distinct amount of mass, continuously distributed over a definite
volume enclosed by a proper surface. Depending upon the different types and
nature of analysis, a body may be needed to be idealized in various ways. Dif-
ferent types of idealization of a body have specific nomenclature, and definite
and separate implication.

1.2.1 Continuum
A continuum is an idealization of continuous description of matter and
properties of matter considered as continuous function of space variables.
In the molecular concept, there exists an intermolecular space. But in the
continuum approach, actual conglomeration of separate molecules occurs,
leaving no empty space within the matter or the system. A continuum may
be rigid (solid) or a continuous deformable medium (fluid).

1.2.2 Particle
A particle is defined as a body whose size, within a given physical situation,
is insignificant in the analysis of its response to the forces that act on it. In
other words, if a body is idealized as a point of concentrated mass and has
insignificant rotational motion, it is defined as a particle. Hence, the concept
of dimensional negligibility is not absolute. In a particular reference frame, a
rigid body may also be idealized as a particle. An aeroplane, while standing
on the tarmac, is a rigid body, but is a particle to an observer at ground when
it is in flight.
When two or more bodies are idealized as particles and are dealt with
together, they are called a system of particles. Thus, a system of particles is
basically a congregation of particles, where the property of each particle is
conserved.

1.2.3 Rigid Body


A rigid body is defined as a material body which, when subjected to a system of
external stimuli, does not change its geometry and size. In other words, the dis-
tance between two arbitrarily chosen points in a rigid body is invariant, whether
or not it is subjected to external stimuli. Usually, in a rigid body the generalized
Introduction to Statics 5

deformation (elongation, contraction, and twist) is ignored during analysis,


because either the deformation is too small or it has very little effect on its
motion or equilibrium.

1.2.4 Deformable Body


A material body is defined as a deformable body, which when subjected to a
system of external stimuli, essentially changes its geometry and size. This defor-
mation may be instantaneous or continuous, or temporary or permanent.

1.3 SPACE
Space is a geometric region occupied by a body whose position is described
by the linear and the angular measurements, with respect to the origin of a
chosen coordinate system.

1.4 SCALARS AND VECTORS


Mechanics deals with two types of physical quantities, namely scalars and
vectors. According to Sir Horace Lamb (1849–1934), the term scalar was first
introduced by Sir William Rowan Hamilton (1805–1865). Lamb also credits
Hamilton with the first use of the term vector.
Some physical quantities can only be measured. Thus, the quantities having
only scalable or measurable property are defined as scalar quantities. The sense
of a scalar is denoted by a positive or a negative sign before the magnitude
of the scalar. Volume, mass, density, temperature are all simply measurable
quantities and thus are scalars. Scalar quantities are expressed with numeri-
cal values in association with proper units, for example, 50 kg mass, 98°F
temperature, 30 s time, etc.
The word ‘vector’ literally means carrier. Any physical quantity which car-
ries something from one point to another can be described as a vector. Hence,
a vector quantity has three properties, namely a definite magnitude (also with
specific units), a specific direction, and a specific sense. Some examples of
representation of vector quantities are 15 kN force vertically downwards,
25 N force at clockwise 40° with positive x-axis, 3.5 m/s2 deceleration along
backward tangent to the curve, and 10 Nm moment anticlockwise about
the y-axis. Conventionally, a vector is represented by a continuous thick
line with an arrow (either open or closed) at one end. The length of the line
represents its magnitude (expressed in some suitable scale and with specific
units), the orientation of the line its direction, and the arrowhead its sense.
In addition, a suitably chosen uppercase boldface letter is used as the
symbol for the vector quantity. Sometimes G normal face uppercase letter
with an arrow or a bar above it (e.g., B , F ) is used to indicate a vector.
Fig. 1.1(a) shows a force in the dextral system of coordinates. Generally lower-
case boldface letters are used to indicate unit vectors, as shown in Fig. 1.1(b).
6 Engineering Mechanics

a
reference line
a
Reference line

(a) (b)

Fig 1.1 Representation of a vector

1.5 FORCE
A force can be defined as an external stimulus which tends to change or actu-
ally changes the state of a body (at rest or in motion) or causes deformation
in the body (physical manifestation of stimulus). Force is a vector quantity,
having a definite magnitude, fixed line of action, and a definite sense. Force
is a bound vector or a fixed vector. Sometimes the term generalized force is
used to indicate a force, a torque, and a moment, as a whole.

1.5.1 Force Field


A force may be generated either by virtue of the action of one body on another
body (ies) or due to the existence of a force field. In general, a force field indicates
the presence of a force as a function of Cartesian coordinates (1D, 2D or 3D) and
time. Thus, a force field can be expressed in a generalized way as, F F(x, y, z, t)
or F F(r, q, z, t). If at any point in space, the force field becomes independent
of time, it is defined as time invariant or temporally independent or steady
force field. Thus, F F(x, y, z) or F F(r, q, z). Depending on the dimension
of Cartesian coordinate system, steady force field can be of three types.
Steady linear force field This force field is restricted along a line and is a
function of one dimension only. Hence, F F(x) or F F(s). It is applicable
for uniformly distributed loading also. If the load intensity is w per unit length,
the force generated over the differential length dl is expressed as dF wdl.
Steady planar force field The variation of force occurs over a plane and force
field is a function of two dimensions. Thus, F F(x, y) or F F(r, q). For
uniformly distributed loading over a plane, force field is usually termed as
surface force field. If p is the pressure effective per unit area, the force acting
on the differential area dA is expressed as dF pdA.
Steady spatial force field This variation of force spreads over a space and
force field is a function of three dimensions. Thus, F F(x, y, z) or F F(r, q, z).
Three-dimensional force field for distributed loading is conventionally termed
as body force field. Body force is specifically dependent on the mass or volume
of the body. If in a material body, the differential mass dm has a differential
volume d" and if the weight density of the material is g, the differential
Introduction to Statics 7

force acting on the body is expressed as dF gdm or dF g d", where g is the


gravitational acceleration at the considered place.

1.5.2 Classification of Force


In terms of various differentiating parameters, forces can be classified as
follows.
Tensile or compressive force Depending on the nature of action, forces are
either tensile or compressive. At any point of the body, tensile force acts away
from the point to impart an elongative effect. On the other hand, compressive
force acts towards the point of the body to create a compressive effect. These
forces are shown in Fig. 1.2.

Fig. 1.2 Fig. 1.3

Concentrated or distributed force Depending on the place of action on a body,


forces are classified as concentrated force or distributed force. If the force acts
on a particular point on the body, it is defined as a concentrated force or a
point force. It is possible that the force acts distributed over a definite distance
or definite area on the body. Such forces are distributed force. The actions of
these forces are illustrated in Fig. 1.3.
Contact or body force Depending on the intrinsic nature, forces are either
contact forces or body forces. Contact forces act directly on the surface of the
body or bodies in contact. Body forces act throughout the material body and
are distributed over its entire volume. Forces due to gravitational acceleration
and electro-magnetic field are examples of body forces.

1.5.3 Force System


A force system is a collection of forces acting at specified locations and may
also include couples (Fig. 1.4). Thus, the set of forces shown on any free-body
diagram makes up a force system. Force system is simply a term used to denote
a group of forces. It can be classified into the following types.
Collinear and non-collinear force systems If all the forces have lines of action
lying along a particular line, a collinear force system develops. In a non-collinear
force system, all the forces have separate lines of action.
Coplanar and non-coplanar force systems In a coplanar force system, all
forces acting on a material body lie in the same plane (vertical, horizontal
8 Engineering Mechanics

or inclined). On the other hand, in a non-coplanar force system, there is no


restriction and the forces may exist in any plane. In both these systems, forces
may be either parallel or non-parallel.
Concurrent and non-concurrent force systems If all the forces acting on a
material body either converge at or diverge from a particular point, then the
force system is said to be concurrent. In a non-concurrent force system, the forces
neither converge at nor diverge from a particular point on the body.
We can also find combinations of coplanar, concurrent; coplanar, non-concur-
rent; non-coplanar, concurrent; and non-coplanar, non-concurrent force systems.

Fig. 1.4

1.6 SUPERPOSITION AND TRANSMISSIBILITY OF FORCE


Superposition and transmissibility are the two F1
basic properties of a force vector. Let us con-
sider a rigid body which is subjected to two
forces F1 and F2, as shown in Fig.1.5. If F1 and
F2 are of the same magnitude and act along the _F1_ _F2_
same line but in opposite directions, we can
F2
obviously conclude, without delving into the
details of force equilibrium, that the net force Fig. 1.5
acting on the body is zero. Let us consider oth-
erwise. First, apply force F1 on the body, by virtue of which it will experience
some sort of a pull along the direction of action of F1 (Fig. 1.6 a). Then apply
force F2 on the body, and it will experience a pull along the direction of action
of F2, without hampering the pulling effect on the body due to F1 (Fig. 1.6 b).
Introduction to Statics 9

Here the effect of force F2 has been F1


F1
superimposed on the effect of force F1.
If F1 F2, the body will finally feel no
pushing or pulling effect. Otherwise the
body will feel a pulling or pushing effect
in the direction of the greater force. A
generalized statement explaining this
phenomenon, and a number of such (a) F2 (b)
phenomena, is designated as the prin- Fig. 1.6 Effect of superposition
ciple of superposition. It states that:
The effect or action of a given system of forces on a non-deformable body
will never get altered if another system of forces in equilibrium is added
to or subtracted from it.
The principle of superposition has been found to be very effective in con-
ceiving and discretizing problems where complex force systems are in play.
Refer to Fig. 1.7(a),where the body is subjected to force F1, the point of ac-
tion being A. Now apply F1
two mutually opposite F1
A A A
directional force F2 on the
extended line of action
of F1, the point of action F2 F2
being B, as shown in Fig. B B
1.7(b). If F1 F2 P, we
can conclude that (a) and (a) F2 (b) (c)
(b) are under the effect of Fig. 1.7 Transmissibility of force
the same state of net force
effect. If force F1 acting from point A and force F2 acting from point B shown
in continuous line are removed, the final state of force acting on the body is
shown in Fig. 1.7(c). Keen observation suggests that the remaining force F2
shown in dashed line is the force F2 P F1. This state can be achieved if we
extend the line of action of force F1 backwards, without considering F2.
Thus, the principle of transmissibility of force states that:
The equilibrium or the motion of a rigid body remains unaltered if the
point of application of any force vector acting on the body is displaced
along the line of action of the force.
This principle is applicable to bodies for which internal forces or deformations
are to be considered.
Vectors following the principle of transmissibility are sometimes referred
to as sliding vectors, since they can slide along their lines of action without
changing their physical effects or physical meaning.

1.7 EQUILIBRIUM

The word ‘equilibrium’ carries a number of meanings in the context of mechan-


ics. Here we are specifically interested in ‘state of equilibrium’. It is basically a
kinematic concept and focuses on the state of motion of body(ies). A body at state
10 Engineering Mechanics

of rest is in equilibrium. This equilibrium is called static equilibrium. In contrast,


dynamic equilibrium relates to the body in motion with uniform velocity.
If a material body is in equilibrium, there is no rate of change of linear momen-
tum. This implies that, using Newton’s laws of motion, the total external forces
acting on the body is zero. Again for a body in equilibrium, there is no rate of
change of angular momentum about any point. This implies that, from Euler’s
law, the total externally acting moment is zero. Thus any equilibrium condition
of a rigid body necessarily implies that the net external force and the net external
moment acting on the body is zero. But the reverse statement is not true.
The concept of equilibrium is based on the following axioms:
(i) A rigid body acted upon by two forces is in a state of static equilibrium
if and only if the two forces are of the same intensity, lie along the same
line of action (collinear), and are oriented in opposite directions along
the line.
(ii) If a system of two forces in equilibrium is added to or extracted from a
given system of forces, the way that the system of forces acts on a rigid
body undergoes no change.
(iii) The resultant of two forces acting on the same material point is equal to
the vector sum of the two forces. The line of the resulting force action
contains the material point. This axiom obeys the principle of vector
summation.
(iv) Two interacting bodies react on each other with two forces of equal
intensity, and along the same line of action, but in opposite directions
along the line. This axiom is known as the principle of action and
reaction.
(v) If a deformable body is in a state of static equilibrium, it would also be
in static equilibrium if the body was rigid. This axiom is known as the
principle of solidification.

1.8 RESULTANT OF A FORCE SYSTEM


A generalized force system considers the effect of force (point load, uniformly
distributed load, uniformly varying load, and distributed load varying follow-
ing any geometric or algebraic equation), moment (concentrated moment,
distributed moment), couple, torque, etc.
In general, the term resultant refers to the combined effect of the generalized
force system. If a force system comprises only forces, the vector sum of all these
externally applied forces is considered as the resultant force. However, any force
system can be simplified about a point to a resultant consisting of one force
and one couple (either or both of which may be zero). The resultant couple is
the vector sum of all the couples in the force system plus the vector sum of all
the moments of all the forces about any point in the force system.
If a body is subjected to more than two forces, two situations may appear.
The body, by the action of all forces, may remain in equilibrium and in that
case no resultant force will be available. It is also possible that by the action
of the force system, the body experiences an imbalance in force. In this case,
Introduction to Statics 11

a resultant force will exist which acting in the opposite direction will counter
the imbalance.
Sometimes, it may happen that a body is subjected to externally applied
point loads only, as shown in Fig. 1.8. In this situation, the force resultant
vanishes, as the vector sum of all the forces is zero, but there exists a non-
zero resultant moment.
One immediate consequence of this definition 20N
of resultant is that the resultant about any point 30N
of the external force system acting on any system A
B
in equilibrium must be zero. The statement that
the sum of the external forces and the sum of the D C
external moments acting on a system in equilibrium
30N
is zero is often replaced by the statement that the
20N
resultant acting on a system in equilibrium is zero.
An important attribute of the resultant of a Fig. 1.8 Force resultant zero
force system is that if you apply the resultant of a but moment resultant exists
force system to a rigid body, the effect on that rigid body is exactly the same
as the effect of the original force system. It is in this sense that the resultant
is equivalent to the original force system. Thus as we study the behaviour
of rigid bodies under the action of forces, the resultant of the external force
system will be of vital interest. Care must be taken in replacing force systems
with their resultant when dealing with deformable bodies, as the effects of
the resultant acting on a non-rigid system may be different from the effects
of the actual force system.

1.9 RESULTANT AND EQUILIBRIUM OF A COPLANAR,


CONCURRENT FORCE SYSTEM
In a coplanar, concurrent force system, all the forces are in F1
the same plane, necessarily emerging from or converging
towards a single point. If the force system is composed O
of only two forces, they may be either collinear or non-
collinear. For collinear forces, the vector sum gives the
resultant and to maintain equilibrium, these forces must F2
necessarily act along the mutually opposite direction and
Fig. 1.9 Coplanar,
of same magnitude, F1 F2. It is illustrated in Fig. 1.9. collinear, concur-
For a non-collinear, concurrent force system, we may rent force system
adopt either of the following approaches in order to find
the resultant and the condition of equilibrium therefrom.

1.9.1 Triangular Law of Forces


Referring to Fig. 1.10(a), we observe that the material body is subjected to
two coplanar, non-collinear, concurrent forces P and Q. Starting from a fixed
point, draw lines parallel to the lines of action of original forces, one after
another, and of course to a suitable scale. We find that these two lines form
two arms of a triangle, and thus form an open triangle. The remaining arm
12 Engineering Mechanics

Fig. 1.10 (a) & (b) Equilibrium of two-force systems


(c) equilibrium of three-force systems
to close the triangle will give the magnitude of the resultant force R. The
direction of the resultant force will be of the reverse order of the other two
arms of the triangle. Vectorially we can say that, R P + Q. It is illustrated in
Fig. 1.10(b). To maintain equilibrium, this resultant force must be zero. So, a
three-force system, P, Q, and R, if in equilibrium, will form a closed triangle,
if the forces are plotted successively. Hence, no resultant force will appear. This
is illustrated in Fig. 1.10(c). Thus, the triangular law of forces states that:
If two coplanar, non-collinear, concurrent forces constituting the force system
acting upon a body or a particle, can be represented in magnitude and direc-
tion by two successive sides of a triangle, taken in order, the resultant force
is represented by the third side of the triangle, taken in reverse order.
If three coplanar, non-collinear, concurrent forces acting upon a body or
a particle can be represented in magnitude and direction by the sides of
a triangle, taken in order, it will be in equilibrium under the action of the
said force system.
The converse of the law of triangle is also valid.

1.9.2 Parallelogram Law of Forces


Let us assume that a material body is subjected to two coplanar, non-collinear,
concurrent forces P and Q, as shown in Fig. 1.11 (a). Their lines of action form
an included angle a. The resultant R of this force system can be determined
using the parallelogram law of forces. Draw lines of forces for P and Q from a
common origin, in proper magnitude and parallel to the line of action of the
original force system, maintaining the included angle a. Considering these two
lines as two consecutive arms, complete the parallelogram. The diagonal of this
parallelogram drawn from the point of origin gives the resultant R in correct
magnitude and direction, as shown in Fig. 1.11 (a). The vectorial representa-
tion of the resultant is R P + Q. The magnitude of the resultant is expressed
as | R| = | P |2 + |Q|2 + 2 | P ||Q| cos a , and the inclinations of the resultant R with
|Q|
respect to the lines of action of constitutive forces P and Q are sin b = sin a and
|P| | R|
sin q = sin a , respectively. Now if we apply on the body a force bearing the
| R|
same magnitude as the resultant R but in reverse direction, the body will remain
in equilibrium. Hence, the parallelogram law of forces states that:
If two coplanar, non-collinear, concurrent forces forming the force system
can be represented in magnitude and direction from a common point of
origin as two adjacent sides of a parallelogram, the resultant force is rep-
Introduction to Statics 13

a = 90°

(a) (b)
Fig. 1.11
resented by the diagonal of the parallelogram drawn from that common
point of origin.
In a special case where the magnitude of the included angle a = 90∞, the
relation between the constituent forces and the resultant can be expressed
as, | P | = | R| sin q = | R| cos b and |Q| = | R| sin b = | R| cos q , as shown in Fig.
1.11(b).
It is worth mentioning here that albeit fundamental in nature, this paral-
lelogram law was treated by Newton as Corollary II to his laws of motion.

1.9.3 Polygon Law of Forces


Polygon law of foces is basically the extension of the triangular law of forces. If a
material body is subjected to more than three coplanar, non-collinear, concurrent
forces, polygon law is used to find the resultant force. Starting from a common
point, we have to plot the forces in proper magnitude and direction, in a succes-
sive manner and in order. In this way, we obtain the successive arms of a polygon
through the successive triangulation process. If the force system keeps the body
in equilibrium, a closed polygon will be formed and the resultant R will be zero.
If not so, one side of the polygon will remain open. This side, when joined
properly, gives the magnitude of the resultant R. The direction of the resultant
R will be in the order reverse to that of the constituent forces. Both these cases
are illustrated in Figs 1.12 and 1.13. Thus, the polygon law of forces states that:
If three or more coplanar, non-collinear, concurrent forces constitute the
force system and can be represented in magnitude and direction by the sides

Fig. 1.12 Fig. 1.13


14 Engineering Mechanics

of a polygon, taken in order, then one side will remain open. This side of
the polygon, if joined properly and in reverse order, will give the resultant
of the force system. If the material body remains in equilibrium under the
action of such type of force system, then a closed polygon is obtained.

1.10 RESOLUTION OF A FORCE INTO COMPONENTS


In a general sense, resolution of a vector means splitting the total effect of a
main vector into n number of parts along n directions. Applying this definition
to a force vector and restricting our discussion to two dimensions, resolution
of a force can be defined as:
Resolution of a force vector indicates the computation of its components
along any two specified directions.
Let us consider a force F, represented by line OC, as shown in Fig. 1.14(a),
which has to be resolved along two directions, Om and On, included angle being
q. The inclinations of F with Om and On axes are a and b, respectively, and
q a + b. Draw lines through point C, parallel to Om and On axes, intersect-
ing the axes at points B and A, respectively. We thus obtain a parallelogram
OACB, where the lines OA and OB represent the components of force F and
are represented by Fm and Fn, respectively. Now we have to find the expres-
sions for Fm and Fn in terms of force F.
n
y Fy

B C C F
B
F F
Fn Fy
b b
a q a
O O
Fm A m Fx A x O Fx
(a) (b) (c)
Fig. 1.14 Resolution of forces
In ∆OAC, ‘OCA ‘COB b, ‘OAC p – (a + b ), and BC OA |Fm|
AC OB |Fn | . Applying sine rule in ǻOAC, we obtain:
OC = OA = AC
sin [p - (a + b )] sin b sin a

|F | |F | |F |
or = m = n
sin [p - (a + b )] sin b sin a
sin b sin a
or |Fm | = |F | |F | = |F |
sin [p - (a + b )] and n sin [p - (a + b )]
Now, for a special case where q a + b 90o, Om and On axes are desig-
nated as conventional x-axis and y-axis, respectively, as shown in Fig. 1.14(b).
The force components are hence designated by Fx and Fy, respectively. Here,
|Fx | = |F | cos a = |F | sin b and |Fy | = |F | cos b = |F | sin a .
Introduction to Statics 15

One very important thing to mention here is that the original force and
its resolved components cannot be simultaneously shown with the same type
of lines. As the original force and its resolutions cannot coexist, either the
original force or its resolutions need to be shown by a continuous line and
the other by dashed lines; the reverse convention may also be followed. This
is illustrated in Fig. 1.14(c).

1.11 MOMENT OF A FORCE ABOUT A POINT


Moment of a force about a point on a rigid body
is conceived as the rotational effect that the force
tends to develop on the body having the point
fixed. This moment is a vector quantity. Let us
consider a force F acting along the direction
shown in Fig. 1.15. P is the point of interest about
which the moment needs to be computed. Take
any point Q on the line of action of F and join
Fig. 1.15
PQ. Let the position vector of Q with respect to
P be rQP. So the moment of F about the point P is MP rQP × F, which is
basically the cross product of rQP and F. The direction of the moment will
be normal to the plane containing rQP and F, guided by the right-hand
screw rule.

1.12 MOMENT OF A FORCE ABOUT A LINE


Let us consider a force F having the point of
action at P lying on x–y plane. This force is
inclined by an angle q with the normal to the
x–y plane, as shown in Fig. 1.16. We want
to find out the moment of F with respect
to the line OL. Now the position vector
of P with respect to O is rPO and of L with
respect to O is rLO. It is very important to
keep in mind that in order for MOL to not to
be zero, the line of action of F must neither
intersect OL nor be parallel to OL. Now the
perpendicular distance of OL from point P
is | rPO | sin f, where f is the inclination of rPO Fig. 1.16
with OL. So the moment of force F with
respect to OL is ML F ⋅ (rLO ⋅ rPO). This moment is a scalar quantity. It has
been observed that the moment of a force about a line or an axis becomes
zero if any of the following conditions is satisfied:
(i) The force itself is zero.
(ii) The line of action of the force intersects the line or the axis.
(iii) The force is parallel to the axis.
16 Engineering Mechanics

1.13 MOMENT OF A COUPLE


A system of two equal parallel forces acting in op-
posite directions cannot be reduced to one resultant
force. Hence, two equal parallel forces form a couple,
provided the forces act along the opposite directions.
Referring to Fig. 1.17, we have parallel forces F of
equal magnitude but acting in opposite directions
lying on the same plane. The normal distance be-
tween the lines of action of parallel forces is r. So
the moment of the couple is Fig. 1.17

M r × F | r | | F | sin 90° | r | | F |
This moment is a vector quantity and acts along normal to the plane contain-
ing F and r guided by the right-hand rule. Another important property of the
moment of a couple is that it is a free vector.

1.14 RESULTANT OF A COPLANAR, NON-CONCURRENT


FORCE SYSTEM
F2 F2

F3 F3
FR
FR O d
d2
Fr
b d1 d3
a
M F
r N
F1
F1
(a) (b)
Referring to Fig. 1.18(a), we observe F2
that the body is subjected to three copla-
nar, non-concurrent forces, namely F1, F3
F2, and F3. The law of transmissibility Fr
d
of forces needs to be exhaustively uti- Į
lized here. Extending the lines of action
of these forces either in the forward or
backward direction, try to find a com- F1
mon point from where any two forces (c)
either diverge or converge to. Here the
lines of action of forces F1 and F2 con- Fig. 1.18 Resultant of coplanar non-
verge at the point of concurrence M concurrent forces
with an included angle a. By taking the lines of action of these two forces and
point M, complete the parallelogram. We thus obtain force Fr as the resultant
of forces F1 and F2. Now suitably extend the lines of action of Fr and F3 to
obtain a point of concurrence N. Considering Fr and F3 as two constituent
forces with included angle b, use the parallelogram law to obtain the resultant
force FR. Here FR can be considered as the resultant of the three coplanar,
Introduction to Statics 17

non-concurrent forces, namely F1, F2, and F3, and can be applied at any point
on its line of action. If there are more number of forces, this procedure can
be adopted in a successive manner. Combining the forces in a different order,
the same result will be obtained. Thus, the resultant is invariant to the order
of combination of the forces.
To determine the magnitude of the final resultant, we have to go for suc-
cessive computations of the resultant using the parallelogram approach.
|Fr | = |F1 |2 + |F2 |2 + 2 |F1 ||F2 | cos a

|FR | = |Fr |2 + |F3 |2 + 2 |Fr ||F3 | cos b , etc.


The location of the line of action of resultant FR can be obtained using the
principle of moments. Consider an arbitrary moment centre O. The moment of
FR about O will be equal to the sum of the moments of constituent forces Fr and
F3 about point O. Again, the moment of Fr about point O will be equal to the
sum of moments of the constituent forces F1 and F2. If the lever arm distances
of the forces F1, F2,F3, and FR from the moment centre O are d1, d2, d3, and
d respectively, we can write, FR d = F1 d1 + F2 d 2 + F3 d3 = SM o , where SM o is
the algebraic sum of the moments of the constituent forces. It is illustrated in
Fig. 1.18(b).
If the forces are parallel, the magnitude of the resultant can be computed
by simply considering the algebraic sum of the magnitude of the constituent
forces. The line of action of the resultant force can be obtained by applying
the principle of moments.
A typical force system is shown in Fig. 1.18(c), where the constituent forces
are F1, F2, and F3. The resultant of F1 and F2 is Fr. It is eventually noticed
that Fr and F3 are parallel, non-collinear forces. For attaining equilibrium
the condition |Fr| = |F3| must be satisfied. Hence, their resultant will be only
a moment of a couple of magnitude F3d, the lever arm distance being d.

1.15 RESULTANT OF A NON-COPLANAR, CONCURRENT


FORCE SYSTEM
In a non-coplanar, concurrent force system, the constituent forces of the
system lie in different planes, having a common point of origin or destina-
tion. Being concurrent in nature, the resultant can be computed through a
successive process, using the parallelogram law of forces. Consider we have
three non-coplanar, concurrent forces, namely F1, F2, and F3. If the lines of
action of forces F1 and F2 form an angle a, the resultant Fr of the forces F1
and F2 can be computed as:
|Fr | = |F1 |2 + |F2 |2 + 2 |F1 ||F2 | cos a
Now, the forces Fr and F3 are also concurrent. Let us assume that the
included angle between Fr and F3 is b. Hence, the resultant of Fr and F3 can
be quantified as:
|FR | = |Fr |2 + |F3 |2 + 2 |Fr ||F3 | cos b
18 Engineering Mechanics

In this way, we can successively use the parallelogram law of forces to find the
resultant of non-coplanar, concurrent forces.
Vectorially we can write, FR F1 + F2 + F3. Resolving in rectangular com-
ponents, we have:
FRxi + FRyj + FRzk (F1xi + F1yj + F1zk) + (F2xi + F2yj + F2zk)
+ (F3xi + F3yj + F3zk)
(F1x + F2x + F3x)i + (F1y + F2y + F3y)j + (F1z + F2z + F3z)k
(∑Fx)i + (∑Fy)j +(∑Fz)k
2 2 2
So, |FR | = |( SFx )| + |( SFy )| + | ( S Fz )|
The inclination of the resultant FR with the Cartesian axes of reference
can be expressed as:
|SFx | |SFy | |SFz |
cos q x = , cos q y = , cos q z =
|FR | |FR | |FR |
Conventionally, these inclinations are termed as direction cosines. Moreover,
cos q x = l , cos q y = m, cos q z = n, having the property, l 2 + m 2 + n 2 = 1.

1.16 RESULTANT OF A NON-COPLANAR, NON-CONCURRENT


FORCE SYSTEM
A non-coplanar, non-concurrent force system is, in true sense, a generalized
force system and hence it is not very easy to compute the resultant. If the con-
stituent forces are parallel, the resultant may be a single force or a moment of
a couple in the plane of the system or in a parallel plane. It may also happen
that the resultant is zero. The resultant will be a force when ∑F is a non-zero
quantity and a moment of a couple when ∑F is zero but ∑Mo is non-zero.
The resultant of a non-parallel, non-coplanar, non-concurrent force system,
in general, comprises a force and a couple. We have to resolve each force into
a parallel force through some common point and a couple. Thus, we obtain
a set of concurrent forces and a set of couples. The resultant of concurrent
forces can be obtained using the conventional vector sum approach. The
couples can also be combined vectorially to obtain a resultant couple.

RECAPITULATION

z Statics deals with the conditions of equilibrium of bodies at rest subjected to a


particular force system.
z A body is a distinct amount of mass, continuously distributed over a definite
volume enclosed by a proper surface. If a body is idealized as a point of
concentrated mass and has insignificant rotational motion, it is defined as a
particle. A material body is defined as a rigid body which, when subjected to a
system of external stimuli, does not change its geometry and size. A material
body is defined as a deformable body which, when subjected to a system of
external stimuli, changes its geometry and size.
z A force is an external stimulus which tends to change or actually changes the state
of a body or exhibits physical manifestation of the stimulus on the body.
Introduction to Statics 19

z Static equilibrium is the state of rest of a body in equilibrium. Dynamic equilibrium


relates to the body in motion with uniform velocity.
z Resolution of a force vector involves the computation of its components along
only two specified directions.
z The different types of force systems are coplanar and concurrent; coplanar and non-
concurrent; non-coplanar and concurrent; and non-coplanar and non-concurrent.
The resultants of these force systems are summarized as:
Force system Resultant
Coplanar, concurrent (collinear or non-collinear) Force
Coplanar, non-concurrent Force or couple
Non-coplanar, concurrent Force
Non-coplanar, non-concurrent (parallel) Force or couple
Non-coplanar, non-concurrent (non-parallel) Force or couple; or
force and couple
z The moment of F about point P is M P = rQP ¥ F, and is a vector quantity. The
moment of a force about any line is a scalar quantity, whereas the moment of a
couple is a vector quantity and is a free vector.

NUMERICAL EXAMPLES

Example 1.1 A force of 100 units acts along the line OP, terminating at P. If the
coordinates of points O and P are (3, –1, 2) and (10, 5, 8), respectively, specify the
force in terms of the unit vectors.
Solution
JJJG (10 - 3) i + (5 - (-1)) j + (8 - 2) k
Unit vector along OP
(10 - 3) 2 + (5 + 1) 2 + (8 - 2) 2
7i + 6 j + 6k
11
7i + 6 j + 6k
So, the force vector F 100 × 63.64i + 54.54 j + 54.54k
11
Example 1.2 A point P is located as (–5, 2, 14) with respect to an origin O (0, 0, 0).
Specify its position vector (i) in terms of the rectangular components, (ii) in terms of
its unit vector, and (iii) in terms of its direction cosines.
Solution
Position vector of P with respect to origin O
(–5 – 0)i + (2 – 0)j + (14 – 0)k –5i + 2 j + 14k
Therefore,
JJJG -5 i + 2 j + 14 k -5 i + 2 j + 14 k
Unit vector along OP
( -5) 2 + 22 + 142 15
–0.33i + 0.13j + 0.93k
So, the position vector of P in terms of unit vector
( -5) 2 + 22 + 142 (–0.33i + 0.13j + 0.93k)
–5i + 2j + 14k
20 Engineering Mechanics

Direction cosines, l cos a -5 –0.33


( -5) 2 + 22 + 142
m cos b 2 0.13
15
n cos g 14 0.93
15
So, the position vector of P in terms of direction cosine
( -5) 2 + 22 + 142 (li + mj + nk )
15l i + 15m j + 15n k

Example 1.3 If A 2 i –3 j – k, B i + 4 j – 2 k, and C 2 i, evaluate (i) A ⋅ B, (ii) A ×


B, (iii) (A + B) ⋅ (A – B), (iv) (A + B) × (A – B), (v) (A × B) ⋅ C,(vi) (A + B) × C.
Solution
(i) A ⋅ B (2 i – 3 j – k) ⋅ (i + 4 j – 2k) (2 × 1) + (–3 × 4) + (–1 × –2) –8
i j k

(ii) A × B 2 -3 -1 10 i + 3 j + 11 k
1 4 -2
(iii) A + B (2 + 1) i + (–3 + 4) j + (–1 – 2) k 3i + j – 3k
A–B (2 – 1) i + (–3 – 4) j + (–1 + 2) k i – 7j + k
So, (A + B) ⋅ (A – B) (3 × 1) + (1 × –7) + (–3 × 1) –7
i j k

(iv) (A + B) × (A – B) 3 1 -3 –20 i – 6 j – 22 k
1 -7 1
(v) (A × B) ⋅ C (10 × 2) + (3 × 0) + (11 × 0) 20
i j k

(vi) (A + B) × C 3 1 -3 –6j – 2k
2 0 0
Example 1.4 A force of 210 N forms angles 53°, 77°, 142° with x-, y-, and z-axes,
respectively. Express the force as a vector.
Solution Here, direction cosines are
l cos a cos 53° 0.6018
m cos b cos 77° 0.2249
n cos g cos 142° –0.788
Thus, the vector will be
210(l i + m j + n k )
(210 × 0.6018) i + (210 × 0.2249) j – (210 × 0.788) k
126.378 i + 47.229 j –165.48 k
Example 1.5 Compute the magnitude of the force F, whose components along x-,
y-, and z-directions are 15 kN, –26 kN, and –33 kN, respectively. Also compute the
inclinations with all axes.
Introduction to Statics 21

Solution The force vector is F 15 i – 26 j –33 k. Therefore, magnitude of F


15 + ( -26) + ( -33) 2
2 2 44.609 kN
The inclinations are
Ê 15 ˆ
a cos–1 Á
Ë 44.609 ˜¯
70.351°

Ê ˆ
b cos–1 Á -26 ˜ 125.65°
Ë 44.609 ¯
Ê ˆ
g cos–1 Á -33 ˜ 137.711°
Ë 44.609 ¯

Example 1.6 Forces 30 kN, 40 kN, 50 kN, and 60 kN are concurrent at O(1, 2, 3) and
are directed through M(6, 3, –2), N(–4, –2, 5), P(–3, 2, 4), and Q(4, –3, 6), respectively.
Determine the resultant of the system.
JJJJG (6 - 1) i + (3 - 2) j + ( -2 - 3) k
Solution Unit vector along OM
52 + 12 + ( -5) 2
1 (5i + j – 5k )
51
JJJJG 30
Force vector along OM is F1 (5i + j – 5k)
51
JJJG ( - 4 - 1) i + ( -2 - 2) j + (5 - 3) k
Unit vector along ON
( -5) 2 + ( - 4) 2 + 22
1 (–5i – 4j + 2k)
45
JJJG 40 (–5i – 4j + 2k)
Force vector along ON is F2
45
JJJG ( -3 - 1) i + (2 - 2) j + (4 - 3) k
Unit vector along OP
( - 4) 2 + 12
1 (– 4i + k)
17
JJJG 50
Force vector along OP is F3 (– 4i + k)
17
JJJG (4 - 1) i + ( -3 - 2) j + (6 - 3) k
Unit vector along OQ
32 + ( -5) 2 + 32
1 (3i – 5j + 3k)
43
JJJG 60
Force vector along OQ is F4 (3i – 5j + 3k)
43
Therefore, resultant of the force system
R F1 + F2 + F3 + F4
Ê 30 ¥ 5 40 ¥ 5 50 ¥ 4 60 ¥ 3 ˆ
Á - - + ˜i
ÁË 51 45 17 43 ˜¯
22 Engineering Mechanics

Ê 40 ¥ 4 60 ¥ 5 ˆ
+ Á 30 - - ˜j
ÁË 51 45 43 ˜¯

Ê 30 ¥ 5 40 ¥ 2 60 ¥ 3 ˆ
+ Á- + + 50 + ˜k
ÁË 51 45 17 43 ˜¯

–29.86i – 65.4j + 30.49k


Magnitude of the resultant ( -29.86) 2 + ( - 65.4) 2 + 30.492 kN
78.09 kN
Inclination with x-, y-, and z-axes, respectively,
Ê ˆ
a cos–1 Á -29.86 ˜ 112.48°
Ë 78.09 ¯

b cos–1 ÊÁ -65.4 ˆ˜ 146.87°


Ë 78.09 ¯
Ê ˆ
g cos–1 Á 30.49 ˜ 67.01°
Ë 78.09 ¯
Example 1.7 Given that the forces P 4i – 2 j + 3k, Q 2i + 4 j + 5k, and R 7i – j
+ xk. Determine the value of x for which the forces will be coplanar.
Solution We know that the cross product of vectors lying in a single plane is 0.
Therefore,
4 -2 3
2 4 5 0
7 -1 x
Solving, we get
4(4x + 5) + 2(2x – 35) + 3(–2 – 28) 0
16x + 20 + 4x – 70 – 90 0
20x 140
x 7
Example 1.8 A man pulls a rope attached to the building by a force of 220 kN
(Fig. E1.8). Express the force as a vector
quantity.
Solution From geometry, tan q 7/6.7.
So, q 46.25°.
Now the force vector is
F Fx + Fy |Fx|i + | Fy | j
Here |Fx| 220 cos q 152.13 kN
| Fy | –220 sin q –158.92 kN Fig. E1.8
Hence the force is F 152.13i – 158.92j

Example 1.9 A model of a sail boat is placed inside a test channel, and kept fixed
as shown in Fig. E1.9(a). The dynamometer indicates forces in cables OA and OC as
173 N and 281 N, respectively. Determine the drag force on the hull and the tension
in strip OB.
Introduction to Statics 23

(a) (b) Force diagram


Fig. E1.9
Solution From geometry,
q tan–1 ÊÁ 2.43 ˆ˜ 61.67°
Ë 1.31 ¯
f tan–1 ÊÁ 0.21 ˆ˜ 9.11°
Ë 1.31 ¯
Now,
FOA (–173 sin q)i + (173 cos q ) j –152.28i + 82.09j
FOB (FOB sin f)i + (FOB cos f) j
FD (FD)i
FOC –281j
For the force system in equilibrium,
FOA + FOB + FOC + FD 0
or (–152.28 + FOB sin f + FD)i + (82.09 + FOB cos f – 281)j 0
So, the equations of equilibrium are
FOB sin f + FD 152.28 or, 0.158 FOB + FD 152.28
FOB cos f 198.91 0.987 FOB 198.91
Solving the two equations, we obtain, FOB 201.53 N and FD 120.44 N.
Example 1.10 A block of weight W is suspended by a 600 mm long cord PR and by
two springs PQ and PS, each having an unstretched length of 450 mm [Fig. E1.10(a)].
Determine the (i) tension in the cord and (ii) weight of the block. Take kPQ 1500
N/m, kPS 500 N/m.

a q
P

Fig. E1.10
24 Engineering Mechanics

Solution Length of PS in stretched condition:


3202 + (460 + 140) 2 680 mm
Length of PQ in stretched condition:
3302 + (580 - 140) 2 550 mm
Stretched length of PQ 550 – 450 100 mm
Stretched length of PS 680 – 450 230 mm
Therefore due to stretching, force in spring PQ,
FPQ 1500 ¥ 100 150 N
1000
Force in spring PS,
FPS 500 ¥ 230 115 N
1000
From geometry [Fig. E1.10(b)],
Ê ˆ
a tan–1 Á 330 ˜ 36.87°
Ë 440 ¯
Ê 160 + 320 ˆ
b tan–1 Á 73.74°
Ë 140 ˜¯

q tan–1 ÊÁ 320 ˆ˜ 28.07°


Ë 600 ¯
Now, FPQ (–150 cos a) i + (150 sin a) j –120i + 90 j
FPS (115 cos q) i + (115 sin q) j 101.47i + 54.11 j
FPR (FPR cos b) i + (FPR sin b) j (0.2799FPR) i + (0.96FPR) j
W –W j
The equations of equilibrium are
–120 + 101.47 + 0.2799FPR 0
90 + 54.11 + 0.96FPR – W 0
Solving the above equations, FPR 66.2 N and W 207.66 N.

Example 1.11 From the arrangement shown in Fig. E1.11(a), determine the value of
W and unstretched length of the spring if the spring constant is 800 N/m. Assume the
pulleys as frictionless and strings pass simply over the puleys.

Fig. E1.11
Introduction to Statics 25

Solution From geometry [Fig. E1.11(b)],


Ê ˆ
a tan–1 Á 0.360 ˜ 36.87°
Ë 0.84 - 0.36 ¯
Ê 0.69 + 0.36 ˆ
b tan–1 Á 71.07°
Ë 0.36 ˜¯
Ê 0.36 ˆ
q tan–1 Á 18.92°
Ë 0.69 + 0.36 ˜¯

Length of AB in stretched condition

0.362 + (0.69 + 0.36) 2 1.11 m


As the strings are simply passing over the pulleys at C and D, the force in string AC is
3W and that in AD is W. So, FAC 3W and FAD W. Let us consider the unstretched
length of spring L m.
So, FAB (1.11 – L) × 800 N
Now,
FAB {(1.11 – L) × 800 cos b}i + {(1.11 – L) × 800 sin b}j
259.53(1.11 – L)i + 756.73(1.11 – L) j
FAD (FAD cos q)i + (FAD sin q) j (0.946FAD)i + (0.324FAD) j
FAC – (FAC cos a)i + (FAC sin a) j (– 0.79FAC)i + (0.6FAC) j
The resultant of the force system is
259.53(1.11 – L) + 0.946 × W – 0.79 × 3W 0
756.73(1.11 – L) + 0.324 × W + 0.6 × 3W 400
Solving the above equations of equilibrium, we obtain, W 63.78 N, L 0.76 m.
Example 1.12 The collar A in Fig. E1.12(a) can slide on a frictionless vertical rod and
is attached with a spring. The spring constant is 660 N/m and the spring is unstretched
when H 300 mm. Knowing that the system is in equilibrium when H 400 mm,
determine the weight of the collar.

Fig. E1.12
26 Engineering Mechanics

Solution When H 300 mm, the length of AB 300 2 mm and when H 400 mm,
the length of AB 3002 + 4002 500 mm.
So, stretching of the spring during equilibrium (500 – 300 2 ) mm
75.736 mm
The force induced in the spring
Fs 660 × 75.736 49.986 N
1000
From force diagram in stretched condition, [Fig. E1.12(b)],
Ê ˆ
q tan–1 Á 400 ˜ 53.13°
Ë 300 ¯
Now, Fs (49.986 cos q) i + (49.986 sin q) j 29.992 i + 39.988 j
and W –W j
So from the equations of equilibrium, 39.988 – W 0. Hence, the weight of the collar
is 39.988 N.
Example 1.13 A 1500 N load is supported by the rope and smooth pulley arrangement
as shown in Fig. E1.13(a). Determine the magnitude and the
direction of force P which should be exerted at the free end of
the rope to maintain the equilibrium. P 25°

Solution Here everywhere in the rope, same amount of force P a


will be effective. From the force diagram [Fig. E1.13(b)], we can
write 1500 N
(a)
F2 (2P sin 25°)i + (2P cos 25°) j
F1 (–P cos a)i + (P sin a) j
So, the equations of equilibrium are
2P sin 25° – P cos a 0
2P cos 25° + P sin a – 1500 0
Solving the above two equations, we get
a 32.3° and P 639.12 N Fig. E1.13
Example 1.14 In Fig. E1.14, if the tensions in the pulley cable are equal, i.e., 400 N,
express the resultant force R exerted on the pulley by the two tensions. Determine the
magnitude of R.
Solution Force in the upper part of string
(400 cos 60°)i + (400 sin 60°) j
Force in the horizontal string 400i
So, the resultant force
R (400 cos 60° + 400)i + (400 sin 60°) j
600i + 346.41 j
Magnitude of the resultant 6002 + 346.412 692.82 N
and inclination with x-axis,
Fig. E1.14
Ê ˆ
a cos–1 Á 600 ˜ 29.99°
Ë 692.82 ¯
Example 1.15 Combine the two forces shown acting on the A-frame into a single force
vector R. Determine the amount of this force and its inclination with the horizontal
x-axis (Fig. E1.15).
Introduction to Statics 27

Solution From geometry, we can say that the forces at A and B both are inclined to
horizontal by an amount 30°. So, A 4 kN
Force at A, FA (4 cos 30°) i + (4 sin 30°) j
Force at B, FB (2 cos 30°) i – (2 sin 30°) j 3m
y
Hence, the resultant force
2 kN x
R (4 cos 30° + 2 cos 30°) i + (4 sin 30°
– 2 sin 30°) j B C
1m
5.196i + j 60° 60°
The magnitude of the resultant
D E
5.1962 + 1 5.291 kN
Fig. E1.15
and the inclination with x-axis,
a cos–1 ÊÁ 5.196 ˆ˜ 10.874°
Ë 5.291 ¯
Example 1.16 A force F 3i + 2 j passes through a point P(0, 2) with respect to
an origin O. Determine the moment of the force about the origin and establish its
uniqueness with respect to the arbitrary position vectors.
Solution The position vector r of P with respect to origin O (0 – 0)i + (2 – 0) j 2 j.
So,
i j k
Moment MP r×F 0 2 0 – 6k
3 2 0
Let us consider two more arbitrary position vectors as r1 1.5i + 3j and r2 –3i.
So,
i j k
M P1 1.5 3 0 –6k
3 2 0

i j k
and MP 2 -3 0 0 – 6k
3 2 0
Hence the desired uniqueness is established.
Example 1.17 A vertical pole is guyed by three cables PA, PB, and PC tied at a
common point P 10 m above ground. The base points of the cables are A(–4, –3, 0),
B(5, 1, –1), and C(–1, 5, 0). If the tensile force in the cables are adjusted to be 15, 18,
and 20 kN, find the resultant force on the pole at P.
Solution Here the coordinates of point P (0, 0, 10). The position vector of A,
rA (–4 – 0)i + (–3 – 0)j+ (0 – 10)k –4i – 3j – 10k
JJJG - 4 i - 3 j - 10 k 1 (– 4i – 3j – 10k)
Unit vector along PA
2 2
(- 4) + (- 3) + 10 2 11.18
JJJG
Force vector along PA is

FPA 15 (– 4i – 3j – 10k)
11.18 JJJG JJJG
In the similar fashion, we can write the force vectors along PB and PC as
28 Engineering Mechanics

FPB 18 [(5 – 0)i+ (1 – 0)j + (–1 – 10)k]


52 + 12 + ( -11) 2
18 (5i + j –11k)
12.124
20
and FPC [(–1 – 0)i + (5 – 0)j + (0 – 10)k]
( -1) 2 + 52 + ( -10) 2
20 (– i + 5j – 10k)
11.224
Now, the resultant of FPA, FPB, and FPC is
Ê 15 ¥ 4 18 ¥ 5 20 ˆ Ê 15 ¥ 3 18 20 ¥ 5 ˆ
R ÁË - 11.18 + 12.124 - 11.224 ˜¯ i + ÁË - 11.18 + 12.124 + 11.224 ˜¯ j

Ê ˆ
+ - 15 ¥10 - 18 ¥ 11 - 20 ¥ 10 k
ÁË 11.18 12.124 11.224 ˜¯

0.2747i + 6.369j – 47.567k


Magnitude of the resultant (0.2747) 2 + (6.369) 2 + ( -47.567) 2 47.99 kN

Example 1.18 To stabilize a tree against


storm, two cables AB and AC are attached
to the trunk and fastened on the ground
at B and C, respectively (Fig. E1.18). If
the tensions in AB and AC are 4.2 kN
and 3.6 kN, compute in each case (i)
components of force exerted by cables,
and (ii) a, b, g angles the forces form
with axes at A which are parallel to the
coordinate axes.
Solution For cable AB:
Component of 4.2 kN force on x–z plane
4.2 cos 40° 3.2174 kN
and along y-axis
– 4.2 sin 40° – 2.6997 kN.
The component 3.2174 kN on x–z plane
will have to be resolved to obtain compo-
nents along x- and z-axis. Therefore, Fig. E1.18
Component along x-axis
3.2174 cos 40° 2.465 kN
Component along z-axis 3.2174 sin 40° 2.068 kN
So, the inclinations of components with the three axes are
Ê ˆ
a cos–1 Á 2.465 ˜ 54.06°
Ë 4.2 ¯
Ê ˆ
b cos–1 Á -2.6997 ˜ 129.99°
Ë 4.2 ¯
Ê ˆ
g cos–1 Á 2.068 ˜ 60.5°
Ë 4.2 ¯
Introduction to Statics 29

For cable AC:


Component of 3.6 kN force along y-axis –3.6 sin 45° –2.546 kN
Component of 3.6 kN force on x–z plane 3.6 cos 45° 2.546 kN
Now,
Component of 2.546 kN along z-axis 2.546 cos 25° 2.307 kN
along x-axis –2.546 sin 25° –1.076 kN
So, the inclinations:
a cos–1 ÊÁ -1.076 ˆ˜ 107.39°
Ë 3.6 ¯
Ê ˆ
b cos–1 Á -2.546 ˜ 135°
Ë 3.6 ¯

g cos–1 ÊÁ 2.307 ˆ˜ 50.15°


Ë 3.6 ¯
Example 1.19 A force acts at the origin of a coordinate system in a direction defined
by angle a 64.5° and g 55.9°. If the y-component of the force is (–200) N, determine
the angle b and also other components of the force and the magnitude of the force.
Solution We know, F | F | (l i + m j + n k). Here
l cos a 0.43051 and n cos g 0.5606
Again we know, from the property of direction cosines,
l2 + m2 + n2 1
Solving, we get
m ± 0.70738
and b cos–1(± 0.70738) 44.977° or 135.022°
As the given y-component is negative, so b will be 135.022°.
Now, –200 | F | m
or |F | -200 282.733 N
-0.70738
Therefore,
Magnitude of the force 282.733 N
x-component of force | F | l 282.733 × 0.43051 121.719 N
z-component of force | F | n 282.733 × 0.5606 158.5 N
Example 1.20 A transmission tower is held by three guy wires AB, AC,
and AD anchored by bolts at B, C, and
D, respectively (Fig. E1.20). If the
tension in AB is 2100 N, determine the
components of the force exerted by the
wire on bolt B.
Solution The coordinates of A (0, 20, 0)
and B (–4, 0, 5)JJJG
Position vector PA (0 – (–4))i + (20 –
0)j + (0 – 5)k 4i +JJJ
20j
G – 5k
Unit vector along PA
4 i + 20 j - 5 k
rBA
42 + 202 + ( -5) 2
1 (4i + 20j – 5k)
21 Fig. E1.20
30 Engineering Mechanics

So, the force vector


FBA 2100 (4i + 20j – 5k) 400i + 2000j – 500k
21
Hence, the components along x-, y-, and z-directions are 400 N, 2000 N, and –500
N, respectively.
Example 1.21 A vertical pole is guyed by three cables PA, PB, and PC tied at a
common point P, 8 m above the ground. The base points of the cables are A(4, 0, 0),
B(–1, 4, 0), and C(–2, –3, 0) m. If the tension in PA is 20 kN, calculate the tensions to
be provided in PB and PC so that the resultant force exerted on the pole is vertical.
Find the force exerted on the pole.
Solution As per the given conditions, the coordinate of P is (0, 0, 8) considering ground
as datum plane and origin O(0, 0, 0). So, the position vectors
JJJG
PA (4 – 0)i + (0 – 0)j + (0 – 8)k 4i + 0j – 8k
JJJG
PB (–1 – 0)i + (4 – 0)j + (0 – 8)k –i + 4j – 8k
JJJG
PC (–2 – 0)i + (–3 – 0)j + (0 – 8)k –2i – 3j – 8k
JJJG
PO (0 – 0)i + (0 – 0)j + (0 – 8)k –8k
4i - 8k 1
Unit vectors, rPA (4i – 8k)
2
4 + ( -8) 2 8.9443

- i + 4 j - 8k 1 (–i + 4j – 8k)
rPB
2 2
( -1) + 4 + ( -8) 2 9
-2 i - 3 j - 8 k 1 (–2i – 3j –8k)
rPC
2 2
(- 2) + (- 3) + (-8) 2 8.775

rPO -8 k –k
( -8) 2
Therefore, force vectors,
FPA 20 (4i – 8k) 8.944i – 17.888k
8.9443
FPB
FPB (–i + 4j – 8k)
9
FPC
FPC (–2i – 3j – 8k)
8.775
FPO –FPO k
So, the equations of equilibrium are
8.944 – 1 FPB – 2 FPC 0
9 8.775
4 F – 3 F 0
9 PB 8.775 PC
–17.888 – 8 FPB – 8 FPC –FPO
9 8.775
Solving the above three equations, we obtain FPB 21.9525 kN, FPC 28.534 kN, and
force exerted on pole P, FPO 63.475 kN.
Example 1.22 In a fluid flow, the velocity of a particle is given by V (2xi – 2yj +
xyk) m/s where the distances are measured in metres. Refer to the origin (0, 0, 0) and
compute the cross product r × V for a fluid particle located at (2, 3, 4).
Introduction to Statics 31

Solution Velocity vector at the specified location,


V (2 × 2)i – (2 × 3)j + (2 × 3)k 4i – 6j + 6k
and position vector with respect to origin,
r (2 – 0)i + (3 – 0)j + (4 – 0)k 2i + 3j + 4k
So, the cross product
i j k
r×V 2 3 4 42i + 4j – 24k
4 -6 6

Example 1.23 Three cables DA, DC, and DB are used to tether a balloon as shown
in Fig. E1.23. Determine the vertical force P exerted by the balloon at D when tension
in cable DC is 270 N.
Solution Here the points of interest are A(–4, 0, 0), B(0, 0, –3), C(2, 0, 4), D(0, 5, 0),
and O(0, 0, 0). So, the unit vector along the lines of actions of forces are
( -4 - 0) i + (0 - 5) j + (0 - 0) k
rDA
( -4) 2 + ( -5) 2
1 (–4i – 5j)
6.403
(0 - 0) i + (0 - 5) j + ( -3 - 0) k
rDB
( -5) 2 + ( -3) 2
1 (–5j – 3k)
5.831
(2 - 0) i + (0 - 5) j + (4 - 0) k
rDC
22 + ( -5) 2 + 42
1 (2i – 5j + 4k)
6.708
Now, the force vectors are
FDA Fig. E1.23
FDA (–4i – 5j)
6.403
FDB
FDB (–5j – 3k)
5.831
FDC 270 (2i – 5j + 4k)
6.708
80.5i – 201.25j + 161k
Equations of equilibrium are
–0.6247FDA + 80.5 0
–0.78FDA – 0.8575FDB – 201.25 –P
–0.514FDB + 161 0
Solving the above equations, we obtain,
P 570.355 N (upwards)
Example 1.24 A rectangular plate is supported by three cables as shown in Fig. E1.24.
If tension in the cable AD is 540 N, determine the weight of the plate.
32 Engineering Mechanics

Solution The coordinates of the points of choice are A(0, 1.2, 0), B(0.65, 0, –0.9),
C(1.15, 0, 0.9), and D(–0.8, 0, 0.9). So, the position
vectors are A 0.65 m
JJJG
AD (–0.8 – 0)i + (0 – 1.2)j + (0.9 – 0)k
B
–0.8i – 1.2j + 0.9k
JJJG 1.2 m
AC (1.15 – 0)i + (0 – 1.2)j + (0.9 – 0)k 0.9 m
1.15i – 1.2j + 0.9k O x
JJJG 0.9 m
AB (0.65 – 0)i + (0 – 1.2)j + (–0.9 – 0)k
0.65i –1.2j – 0.9k D C
0.35 m 1.15 m
Force vectors along the direction of these position z
vectors are
Fig. E1.24
F 540 (–0.8i – 1.2j + 0.9k)
AD
( - 0.8) 2 + ( -1.2) 2 + (0.9) 2
–254.11i – 381.18j + 285.88k
FAC
FAC (1.15i – 1.2j + 0.9k)
1.15 + ( -1.2) 2 + 0.92
2

FAC(0.608i – 0.635j + 0.476k)


FAB
FAB (0.65i – 1.2j – 0.9k)
0.65 + ( -1.2) 2 + ( - 0.9) 2
2

FAB(0.398i – 0.73j – 0.551k)


Equations of equilibrium along x- and z-directions:
–254.11 + 0.608FAC + 0.398FAB 0
285.88 + 0.476FAC – 0.551FAB 0
Solving these two equations, we get FAB 562.05 N, FAC 50.02 N.
So, the weight of the plate will be
–381.18 – 0.635FAC – 0.73FAB
–823.24 N
823.24 N (downwards)
Example 1.25 A horizontal circular
plate having a mass of 28 kg is
suspended with the help of three
wires as shown in Fig. E1.25. Each wire
forms 30° with the vertical. Determine
the tension in each wire.
Solution Here the vertical components,
i.e., the components along y-axis, of
FDA, FDB, and FDC are –FDA cos 30°,
– F DB cos 30°, and –F DC cos 30°,
respectively. Components of FDA, FDB,
and FDC on horizontal plane, i.e., on
x–z plane are FDA sin 30°, FDB sin 30°,
and FDC sin 30°, respectively.
Fig. E1.25
Introduction to Statics 33

Now,
x-component of FDA – (FDA sin 30°) sin 50° –0.383FDA
FDB (FDB sin 30°) cos 40° 0.383FDB
FDC (FDC sin 30°) cos 60° 0.25FDC
z-component of FDA (FDA sin 30°) cos 50° 0.321FDA
FDB (FDB sin 30°) sin 40° 0.321FDB
FDC –(FDC sin 30°) sin 60° –0.433FDC
So, the force vectors can be written as
FDA FDA(–0.383i – 0.866j + 0.321k)
FDB FDB(0.383i – 0.866j + 0.321k)
FDC FDC(0.25i – 0.866j – 0.433k)
and the weight vector –28g j –(28 × 9.807)j
So, the equations of equilibrium are
–0.383FDA + 0.383FDB + 0.25FDC 0
–0.866FDA – 0.866FDB – 0.866FDC –274.596
0.321FDA + 0.321FDB – 0.433FDC 0
Solving the above three equations, we obtain,
FDA 135.1 N, FDB 46.9 N, FDC 135.1 N
Example 1.26 The forces F1, F2, and F3
act on the box as shown in Fig. E1.26.
Determine the resultant of the forces. The
magnitude of the given forces are 19 N,
23 N, and 46 N, respectively.
Solution The points of interest are D(0, 2,
3), E(0, 0, 3), H(1, 0, 3), B(3, 2, 0), G(3, 0, 0).
Position vectors of
F1 (1 – 0)i + (0 – 2)j + (3 – 3)k i – 2j
F2 (3 – 0)i + (2 – 0)j + (0 – 3)k
3i + 2j – 3k
F3 (3 – 3)i + (0 – 2)j + (0 – 0)k
–2j
So, the force vectors, Fig. E1.26
i - 2j
F1 19 × 8.497i – 16.994j
1 + ( -2) 2
3i + 2 j - 3 k
F2 23 × 14.71i + 9.807j – 14.71k
32 + 22 + ( -3) 2
-2 j
F3 46 × – 46j
( -2) 2
Therefore,
Resultant R (8.497 + 14.71)i + (–16.994 + 9.807 – 46)j – 14.71k
23.207i – 53.187j –14.71k
Magnitude of resultant R 23.207 2 + ( -53.187) 2 + ( -14.71) 2
59.865 N
34 Engineering Mechanics

Example 1.27 A force F (6i + 8j – 13k) N passes through point P(1, 2, 3).
Compute the moment of the force about point Q(4, 5, 6). The coordinate distances
are measured in metres.
Solution Position vector rQP (1 – 4)i + (2 – 5)j + (3 – 6)k –3i – 3j –3k
Therefore,
i j k
Moment of the force F rQP × F -3 -3 -3 63i – 57j – 6k
6 8 -13
and

Magnitude of the moment 632 + ( -57) 2 + ( - 6) 2 85.17 N m

Example 1.28 A force F (32i – 15j + 50k) N acts at point P(4, –6, 3) m. Determine
the moment of this force about the y-axis.
SolutionJJJConsidering
G origin O(0, 0, 0), the position vector of point P with respect to
origin, OP (4 – 0)i + (–6 – 0)j + (3 – 0)k 4i – 6j + 3k
Therefore,
i j k
Moment of the force about the origin 4 -6 3
32 -15 50
(–255i – 104j + 132k) N m
Moment about the y-axis –104 N m.

Example 1.29 A force of 8 kN acts along OC. Compute the moment of the force
about the line BQ (Fig. E1.29).
Solution The points of interest are O(0,
0, 0), C(4, 3, 3), B(4, 0, 3), and Q(4, 3, 0).
JJJG
Position vector OC (4 – 0)i + (3 – 0)j +
(3 – 0)k 4i + 3j + 3k
JJJG 4 i + 3 j + 3k
Unit vector along OC , rOC
42 + 32 + 32
0.69i + 0.51j +
0.51k

Force vector FOC 8(0.69i + 0.51j Fig. E1.29


+ 0.51k) 5.52i + 4.08j + 4.08k
JJJG
Position vector BQ (4 – 4)i + (3 – 0)j + (0 – 3)k 3j – 3k
3 j - 3k
Unit vector rBQ 0.707j – 0.707k
32 + ( -3) 2
JJJG
Also, position vector BC (4 – 4)i + (3 – 0)j + (3 – 3)k 3j

0 3 0
Moment about BQ 5.52 4.08 4.08 11.708 kN m
0 0.707 - 0.707
Introduction to Statics 35

Example 1.30 Moment of a certain force about a point P(3, 7, –2) is MP (10i – 8j
+ 40k) kN m. Determine the moment of the same force about the line PQ, where the
coordinates of Q are (1, 8, 5).
JJJG
Solution Position vector PQ (1 – 3)i + (8 – 7)j + (5 – (–2))k –2i + j + 7k
-2 i + j + 7 k
Unit vector, rPQ –0.272i + 0.136j + 0.953k
( -2) 2 + 12 + 7 2
Therefore,
JJJG
Moment about PQ MP ◊ rPQ
(10i – 8j + 40k) ◊ (–0.272i + 0.136j + 0.953k)
34.312 kN m
Vector moment 34.312(–0.272i + 0.136j + 0.953k)
–9.333i + 4.666j + 32.7k
Example 1.31 Find the projection of vector F (i – 4j + 3k) on the line joining points
P(–1, 3, 5) and Q(2, –5, –7).
JJJG
Solution Position vector PQ (2 – (–1))i + (–5 – 3)j + (–7 – 5)k
3i – 8j – 12k r (say)
We know, F ◊ r | F | | r | cos q
Giving,
Ê 2 2 2ˆ
(i – 4j + 3k) ⋅ (3i – 8j –12k) | F | Á 3 + ( -8) + (–12) ˜ cos q
Ë ¯
or | F | cos q-1 –0.0678 projection of F on r
217
Example 1.32 Determine the perpendicular distance from the point P(1, 2, 3) to
the line joining the origin O(0, 0, 0) and the point Q(2, 10, 5). The coordinates are
measured in metres.
Solution Position vector of P with respect to O,
rP (1 – 0)i + (2 – 0)j + (3 – 0)k i + 2j + 3k
Position vector of Q with respect to O,
rQ (2 – 0)i + (10 – 0)j + (5 – 0)k 2i + 10j + 5k
Now,
rP × rQ |rP| |rQ| sin q
That is,
i j k
1 2 3 |rP| sin q 22 + 102 + 52
2 10 5

Solving, we get

–20i + j + 6k 11.358|rP| sin q

Now the perpendicular distance from P to line OQ |rP| sin q. Hence,


( -20) + 1 + 62
2 2
Magnitude of this distance 1.8405 m
11.358
36 Engineering Mechanics

Example 1.33 A rectangular block 3 m × 2 m × 1 m is subjected to forces F1, F2, and


F3, the magnitudes of which are 5 kN, 7.5
kN, and 10 kN, respectively (Fig. E1.33).
Replace this force system by an equivalent
force–couple system acting at O.
Solution Coordinates of points of
interest are O(0, 0, 0), R(0, 1, 0), A(0, 1, 2),
D(0, 0, 2), B(3, 1, 2), and Q(3, 1, 0).
Unit vectors
(3 - 0) i + (1 - 1) j + (0 - 0) k
rRQ i
32
(0 - 0) i + (0 - 1) j + (2 - 2) k
rAD –j
( -1) 2 Fig. E1.33

(3 - 0) i + (1 - 1) j + (2 - 0) k 3 i+ 2 k
rRB
32 + 22 13 13

Force vectors F1 5i, F2 –7.5j, and F3 8.32i + 5.547k


Therefore,
Resultant force, FR F1 + F2 + F3 13.32i – 7.5j + 5.547k
The resultant couple is found by taking the summation of moments of F1, F2, F3,
each about origin. So,
Position vectors rOR j and rOD 2k
Summation of moments about O,
MA rOR ¥ F1 + rOD ¥ F2 + rOR ¥ F3

i j k i j k i j k
0 1 0 + 0 0 2 + 0 1 0
5 0 0 0 -7.5 0 8.32 0 5.547

20.547i – 13.32k

Example 1.34 A 0.45 m × 0.25 m plate is subject-


ed to a 13 kN force as shown in Fig. E1.34. Find
the moment of the force about points B and A.
Solution The force vector can be written as
F [(13 cos 63°)i + (13 sin 63°)j] kN
(5.902i + 11.583j) kN
Considering B as origin (0, 0), the position vector
of D, rBD (0.45i + 0.25j) m Fig. E1.34
So, the moment about B, MB rBD × F

i j k
0.45 0.25 0 3.737 kN m
5.902 11.583 0
Introduction to Statics 37

Similarly, position vector of D, rAD (0.45i) m


So, the moment about A, MA rAD × F
i j k
0.45 0 0 5.212 kN m
5.902 11.583 0

Example 1.35 A force F (4i + 5j + 6k) kN acts at a position vector rF (3i – 2j +


4k) m. A couple moment also acts, Mc (2i + 3j) kN m. Replace the total system by
a wrench. Specify the equivalent wrench.
[Note—A force and a couple–moment, equivalent to a force system, directed along
the force is called a wrench.]
4i + 5 j + 6 k
Solution Unit vector along F, rF
4 2 + 52 + 6 2
0.456i + 0.57j + 0.684k
The couple–moment may be resolved into McF, along the direction of force and McN
normal to it, such that, Mc McF + McN. Now, the magnitude of McF
Mc ◊ rF (2i + 3j) ◊ (0.456i + 0.57j + 0.684k) 2.622 kN m
So,
McF 2.622(0.456i + 0.57j + 0.684k)
1.196i + 1.49j + 1.793ks
Therefore,
McN (2i + 3j) – (1.196i + 1.49j + 1.793k)
0.804i + 1.51j – 1.793k
But McN is undesirable and needs to be transferred by means of parallel trans-
fer of force. Let us consider that the force is transferred at a new position vector
r1 xi + yj + zk. So, displacement of position vector for the force rO1
(x – 3)i + (y + 2)j + (z – 4)k
This parallel transfer of force must be accompanied by a moment vector:
i j k
M1 –rO1 × F – ( x - 3) ( y + 2) ( z - 4)
4 5 6
M1 should be –McN –0.804i – 1.51j + 1.793k. The solution of this equation provides
the values of x, y, and z which implies that the equivalent wrench is such that the
force F passes through this point (xi + yj + zk) and the accompanying moment in the
direction of the force is McF.
Example 1.36 Reduce the force system, as shown in Fig. E1.36 into (i) equivalent
single resultant, (ii) equivalent force–moment system at A, and (iii) equivalent force–
moment system at C.

Fig. E1.36
38 Engineering Mechanics

Solution
(i) Resultant force R (10j – 25j + 40j – 70j) kN –45j kN
If the resultant force is to act as equivalent to the force system, it is assumed that
the resultant acts at a distance x m from A, such that the algebraic summation
of moments of all forces about A is equal to the moment of the resultant about
A. So,
1i × (–25j ) + 3i × 40j + 5i × (–70j ) xi × (–45j )
i j k i j k i j k i j k
or 1 0 0 + 3 0 0 + 5 0 0 x 0 0
0 -25 0 0 40 0 0 -70 0 0 -45 0

–25k + 120k + (–350)k (–45x)k


–45x –255
x 5.67 m
(ii) If the force of –45j is to act at A, the accompanying moment must be MA
(–255k) kN m, as determined earlier.
(iii) If the force of –45j is to act at C, the accompanying moment can be computed as
MC –3i × 10j + (–2i ) × (–25j ) + 2i × (–70j )

i j k i j k i j k
-3 0 0 + -2 0 0 + 2 0 0
0 10 0 0 -25 0 0 -70 0
–30k + 50k –140k
(–120k) kN m
Example 1.37 A 4 m × 5 m concrete slab is z
subjected to the forces as shown in Fig. E1.37. 3 kN y
O
Determine the equivalent action which can be
5 kN
applied at O. Also determine the single resultant (1, 4)
7 kN
of the force system. 4m
6 kN (2, 2)
Solution The resultant force
(3, 4)
R –6k + 5k – 7k – 3k (–11k) kN
5m
The accompanying moment at O for equiva- x
lence,
Fig. E1.37
MO 4i × (– 6k) + (2i + 2j ) × 5k + (i + 4j ) × (–3k) + (3i + 4j ) × (–7k )
i j k i j k i j k i j k
4 0 0 + 2 2 0 + 1 4 0 + 3 4 0
0 0 -6 0 0 5 0 0 -3 0 0 -7

(24j + 10i – 10j –12i + 3j –28i + 21j) kN m


(–30i + 38j) kN m
To determine the single resultant of the force system, we have to determine the point
of application of the resultant force, of which the moment about O will balance MO.
Let us assume the point of application as G(xi + yj). So,
Introduction to Statics 39

(–30i + 38j ) (xi + yj ) × (–11k)


i j k
x y 0
0 0 -11
(–11y)i + (11x)j
By comparing, we get
11x 38 and 11y 30
or x 3.454 m and y 2.727 m
So, if the force of 11 kN acts downwards at coordinates (3.454, 2.727), it will act as
single resultant of the given force system.

Example 1.38 Find the moment of the force F (2i + 3j – k) N acting through the
point P (3, 1, 1) m with respect to the line passing from Q (2, 5, –2) m through
R (3, –1, 1) m.
JJJG
Solution The position vector QP (3 – 2)i + (1 – 5)j + (1 – (–2))k
i – 4j + 3k
The moment of F about point Q
i j k
JJJG
MQ QP × F 1 -4 3 –5i + 7j + 11k
2 3 -1
Now, unit vector of line QR,
(3 - 2) i + ( -1 - 5) j + (1 - ( -2)) k
rQR
12 + ( -6) 2 + 32
1 (i – 6j + 3k)
6.7823 JJJG
Therefore, moment of force F about the line QR,

MQR MQ ◊ rQR (–5i + 7j + 11k) ◊ 1 (i – 6j + 3k)


6.7823
1 (–5 – 42 + 33) N m
6.7823
–2.0642 N m
In this example QP is considered as the lever arm. The result would be the same if
RP is the lever arm.
Example 1.39 Determine the general force–couple resultant of the non-coplanar, non-
concurrent force system consisting of a 180 N force along the line from A(2, 0, 0) m
to B(0, 0, 1) m, a 110 N force along the line from C(0, –2, –1) m through D(–1, 0, –1)
m and a 230 N m couple lying in the x–y plane.
Solution Unit vectors
(0 - 2) i + (0 - 0) j + (1 - 0) k 1
rAB (–2i + k)
( -2) 2 + 12 5

( -1 - 0) i + (0 - ( -2)) j + ( -1 - ( -1)) k 1
rCD (–i + 2j)
2
( -1) + (2) 2 5
40 Engineering Mechanics

Therefore, force vectors


180
FAB (–2i + k) –160.996i + 80.498k
5
110
FCD (–i + 2j) –49.193i + 98.387j
5
Considering origin as O(0, 0, 0), position vector
JJJG
OA (2 – 0)i + (0 – 0)j + (0 – 0)k 2i
JJJG
OC (0 – 0)i + (–2 – 0)j + (–1 – 0)k –2j – k
JJJG JJJG
Here, MAB OA × FAB, MCD OC × FCD, MC 230k and resultant couple,
MR MAB + MCD + MC
i j k i j k
2 0 0 + -2 0 -1 + 230k
-160.996 0 80.498 -49.193 98.387 0
–160.996j + 98.387i + 49.193j – 196.774k + 230k
(98.387i – 111.803j + 33.226k) N m
Example 1.40 A surveyor needs to determine the minimum distance between two lines
LAB and LCD. The first line passes through points A(50, –75, 25) m and B(25, 0, –50) m.
The second line passes through C(100, 50, 25) m and D(–25, –50, 25) m.
Solution Position vector of B with respect to A,
rBA (25 – 50)i + (0 – (–75))j + (–50 – 25)k
–25i + 75j – 75k
Position vector of D with respect to C,
rDC (–25 – 100)i + (–50 – 50)j + (25 – 25)k
–125i – 100j
The vector perpendicular to the plane containing rBA and rDC is
R rBA × rDC
i j k
-25 75 -75 –7500i + 9375j + 11875k
-125 -100 0
The unit vector along R,
-7500 i + 9375 j + 11875 k
rR
( -7500) 2 + 93752 + 118752
–0.4441i + 0.5552j + 0.7032k
Now, the position vector of B with respect to C,
rBC (25 – 100)i + (0 – 50)j + (–50 – 25)k
–75i – 50j –75k
To determine the minimum distance, we have to take the dot product of rR and rBC.
So, the minimum distance | rR ◊ rBC |
|(–0.4441i + 0.5552j + 0.7032k) ◊ (–75i – 50j – 75k)|
|33.3075 – 27.76 – 52.74|
47.1925 m
Introduction to Statics 41

Example 1.41 A rigid T bar is subjected to a force system, as shown in Fig. E1.41.
Determine the resultant force and the resultant couple transmitted to the structure.
Take the magnitude of forces as F1 2 kN, F2 3 kN, and F3 1.5 kN.
Solution The entire force system is equivalent to three couple–moments. Force F1
composes a couple–moment M1, where M1 +(3.5 × 2)i 7i. Similarly, M2 –(3 × 7)j
–21j, and M3 –(1.5 × 7)k –10.5k. So, resultant vector
MR M1 + M2 + M3
7i – 21j – 10.5k

Fig. E1.41
Therefore,
Magnitude of the resultant vector 7 2 + ( -21) 2 + ( -10.5) 2
24.5 N m
Inclinations with respect to the corresponding axes are

a cos–1 ÊÁ 7 ˆ˜ 73.398°
Ë 24.5 ¯

b cos–1 ÊÁ -21 ˆ˜ 148.99°


Ë 24.5 ¯
g cos–1 ÊÁ -10.5 ˆ˜ 115.37°
Ë 24.5 ¯
Example 1.42 In the crank handle shown in Fig. E1.42, determine the moment of
force F about O and A. The magnitude of force is 120 N.

Fig. E1.42
42 Engineering Mechanics

Solution From the geometry, we can obtain the coordinates of the salient points as
O(0, 0, 0) m, A(10, 0, 0) m, C(10, 4, –8) m, and F(18, 8, –12) m. Now, the position
vector of F with respect to C:
(18 – 10)i + (8 – 4)j + (–12 – (–8))k 8i + 4j – 4k
Therefore,
8i + 4 j - 4 k
Unit vector rFC 0.816i + 0.408j – 0.408k
82 + 42 + ( - 4) 2
Force vector FCF 120(0.816i + 0.408j – 48.96k
97.92i + 48.96j – 48.96k
Also, position vector
JJJG
OC (10 – 0)i + (4 – 0)j + (–8 – 0)k 10i + 4j – 8k
So, moment about point O,
JJJG
MO OC ¥ FCF
i j k
10 4 -8 195.84i – 293.76j + 97.92k
97.92 48.96 -48.96

Magnitude of moment 195.842 + 97.922 + (- 293.76) 2 366.38 N m


Now, the position vector
JJJG
AC (10 – 10)i + (4 – 0)j + (–8 – 0)k 4j – 8k
So, moment about point A,
JJJG
MA AC × FCF
i j k
0 4 -8 195.84i – 783.36j – 391.68k
97.92 48.96 - 48.96

Magnitude of moment 195.842 + ( -783.36) 2 + ( -391.68) 2

897.45 N m

EXERCISES

Multiple Choice Questions


Choose the correct or most appropriate option(s).
1.1 Two essential properties of force are
(a) magnitude and sense (b) sense and direction
(c) magnitude and direction (d) none of these
1.2 A point force is an idealization of a real force that is distributed over a small
area of application.
(a) true (b) false
1.3 Concurrent forces may be
(a) coplanar (b) non-coplanar
(c) either coplanar or (d) none of these
non-coplanar
Introduction to Statics 43

1.4 Coplanar forces may be


(a) concurrent (b) non-concurrent
(c) either concurrent or (d) none of these
non-concurrent
1.5 Collinear forces are
(a) concurrent (b) coplanar
(c) concurrent and coplanar (d) none of these
1.6 The sum of two vectors is
(a) a scalar (b) a vector
(c) a pure number (d) none of these
1.7 The product of two vectors may be
(a) a vector (b) a scalar
(c) either a vector or a scalar (d) none of these
1.8 A vector having zero magnitude is called
(a) a unit vector (b) a bound vector
(c) a free vector (d) a null vector
1.9 The value of a unit vector dotted into itself is
(a) one (b) zero
(c) both (d) none of these
1.10 The value of a unit vector dotted into a different unit vector is
(a) one (b) zero
(c) both (d) none of these
1.11 A scalar triple product is
(a) a vector (b) a scalar
(c) both (d) none of these
1.12 A vector triple product is
(a) a vector (b) a scalar
(c) both (d) none of these
1.13 If the scalar triple product of three vectors is equal to zero, then the vectors are
(a) coplanar (b) non-coplanar
(c) either coplanar or (d) none of these
non-coplanar
1.14 According to the law of triangle of forces
(a) three concurrent forces will be in equilibrium
(b) three concurrent forces can be represented by a triangle each side being
proportional to the force
(c) if three forces acting upon a particle are represented in magnitude and direc-
tion by the sides of a triangle taken in order, they will be in equilibrium
(d) if three concurrent forces are in equilibrium, each force is proportional to
the sine of the angle between the other two
1.15 According to the principle of transmissibility of forces, the effect of a force
upon a body is
(a) maximum when it acts at the centre of gravity of a body
(b) same at every point in its line of action
(c) minimum when it acts at the centre of gravity of a body
(d) different at different points in its line of action
1.16 Two vectors are said to be equal if their
(a) magnitude and direction are the same
(b) magnitude is the same
(c) magnitude, direction, and sense are the same irrespective of their location
in space
(d) direction is the same
44 Engineering Mechanics

1.17 Magnitude of a vector quantity is


(a) its dot product with unit vector along its direction
(b) its cross product with unit vector along its direction
(c) both (a) and (b)
(d) none of these
1.18 If the dot product of two vectors is zero, then we can conclude that
(a) vectors are parallel (b) vectors are mutually perpendicular
(c) vectors are collinear (d) none of these
1.19 If the cross product of two vectors is zero, then we can conclude that
(a) vectors are parallel (b) vectors are mutually perpendicular
(c) vectors are collinear (d) none of these
1.20 The simplest possible resultant of a coplanar force system is
(a) a single moment (b) a single force
(c) a single force or (d) none of these
a single moment
1.21 The moment of a force about a point is
(a) a vector quantity (b) a scalar quantity
(c) zero (d) none of these
1.22 The moment of a force about a line is
(a) a vector quantity (b) a scalar quantity
(c) zero (d) none of these
1.23 The moment of a couple is
(a) a free vector (b) a bound vector
(c) zero (d) none of these
1.24 The term ‘direction cosine’ in vector analysis is
(a) the cosine of the angles made by the components of the vector with respec-
tive coordinate axis
(b) the inverse of the cosine of the angles made by the components of the vec-
tor with respective coordinate axis
(c) zero
(d) none of these
1.25 The values of ‘direction cosine’ lies within the range
(a) 0 to +1 (b) +1 to –1
(c) 0 to –1 (d) none of these
1.26 A force and a couple–moment directed along the force, equivalent to a system
of forces is called
(a) a momentum (b) a torque
(c) a wrench (d) none of these
1.27 In the right-hand triad convention of vector direction, the resultant vector
(a) acts along the reverse direction of emergent normal
(b) acts along the same direction of emergent normal
(c) zero
(d) none of these

Review Questions
1.1 Define (i) particle, (ii) continuum, (iii) rigid body, and (iv) deformable body.
1.2 Define force. Explain the different types of force fields.
1.3 Write a brief note on the different types of forces.
Introduction to Statics 45

1.4 Explain the different types of force systems.


1.5 Write short notes on (i) polygon law of forces, (ii) parallelogram law of forces,
and (iii) triangular law of forces.
1.6 Explain resolution of forces.
1.7 Establish the expression for the moment of a force about a line.
1.8 Discuss the moment of a couple.
1.9 How will you obtain the resultant of a coplanar, non-concurrent force
system?
1.10 Explain the procedure for finding the resultant of a non-coplanar, concurrent
force system.

Numerical Problems
1.1 A force of 500 N forms angles of 60°, 45°, and 120°, respectively, with the x-,
y-, and z-axes. Find the components of the force along the coordinate axes.
1.2 A force of 400 N acts along AB, where A (3, 2, –4) and B (8, –5, 6). Write
the force vector.
1.3 The point of application of a force F (5i + 10j –15k) N is displaced from the
point P (1, 0, 3) m to the point Q (3, –1, –6) m. Find the work done by the
force.
1.4 Two vectors P and Q are given by P (2i –6j –3k) and Q (4i + 3j – k). Deter-
mine their dot product and the angle between them.
1.5 A force F (8i + 5j – 6k) N passes through the point Q (6, 2, 5) m. Calculate
the moment about the point P (3, 1, 1) m.
1.6 Moment of a certain force about the point A (3, –1, –6) m is (10i – 8j + 40k)
kN m. Find the moment of the same force about the line AB, where the coor-
dinates of B are (5, 8, 1) m.
1.7 A vertical beam AE is supported by three guy wires AB, AC, and AD
where these are anchored at points B, C, and D, respectively. The height of
the peak point of the beam is 12 m from ground. If the tensile force in the
wire AD is 252 N, determine the forces in the wires AC and AB so that the
resultant force on A is vertical. Consider the coordinates as A (0, 12, 0),
B (0, 0, –9), C (–4, 0, 3), and D (6, 0, 4).
1.8 Determine the component of the vector (3i + 2j – 5k) along the vector
(4i – 3j).
1.9 A force of 1 kN in a particular direction must be applied to tow a boat. Due to
some reason, it is not possible to apply the force in that direction, but two forces
can be applied at 30° and 45° on either side of it in the same plane containing
the given force. Determine the magnitudes of the forces required along these
directions.
1.10 Determine the unit vector for a line S that originates at point (2, 3, 0) and
passes through point (–2, 4, 6). Also determine the projection of vector
(2i + 3j – k) along the line S.
2

C H A P T E R
Equilibrium of Forces

Key Concepts
z Lami’s theorem
z Theorem of Varignon
z Resultant of two like and unlike parallel forces
z Parallel shifting of forces
z Resolution of a force system into an equivalent force–couple system
z General equations of equilibrium
z Active and reactive forces
z Different types of support
z Concept of free-body diagrams

2.1 LAMI’S THEOREM


Bernard Lamy or Lami (1640–1715), the French oratorian mathematician
talked on equilibrium of forces in a different way.
Lami’s theorem is applicable for the case where the force system in equilib-
rium consists of three coplanar concurrent forces. The theorem states that:
If three coplanar concurrent forces are in equilibrium, then ratio of each
force and the sine of the angle included between the other two forces is
constant.
Figure 2.1(a) shows a coplanar concurrent force system, where the angle
included between F1 and F2 is g, F2 and F3 is a, and F3 and F1 is b. So, the
mathematical expression of the theorem is
| F1| |F | |F |
= 2 = 3 (2.1)
sin a sin b sin g
We can prove the theorem very easily. All these three forces can be repre-
sented in a vector triangle ABC as shown in Fig. 2.1(b). Now the included
angles are ·ABC p – b, ·BCA p – g, and ·CAB p – a. Applying the
rule of sine, we can write,
| F1| | F2| | F3|
= =
sin –CAB sin –ABC sin –BCA
Equilibrium of Forces 47

Fig. 2.1

| F1| | F2| | F3|


or = =
sin(p - a ) sin(p - b ) sin(p - g )
| F1| |F | |F |
or = 2 = 3
sin a sin b sin g

2.2 THEOREM OF VARIGNON


A very useful property of a concurrent force system, established by the French
mathematician Pierre Varignon (1654 –1722) long before the introduction of
vector algebra, states that:
The moment of the resultant of a number of concurrent forces, about any
given point, is equal to the algebraic summation of the moments of all
contributing forces about that same point.
Let us consider forces P and Q have resultant R, and the force system is
represented in x–y coordinate system as shown in Fig. 2.2. O is an arbitrarily
chosen point to be considered as moment centre. The perpendicular distances
from moment

Fig. 2.2
48 Engineering Mechanics

centre on the line of action of the respective forces are rP, rQ, and rR and they
subtend angles aP, aQ, and aR with y-axis. The resolution of the forces along
x-axis are Px = P cos a P , Qx = Q cos a Q , and Rx R cos aR. Now, the sum-
mation of moments of forces P and Q about point O is
MP + MQ P ¥ rP + Q ¥ rQ P × (OA cos aP) + Q × (OA cos aQ)
( P cos a P )OA + ( Q cos a Q )OA Px ¥ OA + Qx ¥ OA
( Px + Qx ) ¥ OA Rx ¥ OA
( R cos a R ) ¥ OA R ¥ (OA cos a R ) R ¥ rR
M R moment of the resultant R
Though the theorem was originally proposed for a force having two resolved
components, it is equally valid for any force having more than two resolved
components.
Based on the principle established here, some special cases are discussed
here.
2.2.1 Resultant of Two Like Parallel Forces
Let us consider F1 and F2 as two like parallel forces acting normal to any incline
at points A and B, respectively, as shown in Fig. 2.3. Also assume a point C
on the incline which serves as the point of action of resultant force FR. From
the concept of resultant, FR F1 + F2. Consider an arbitrarily chosen moment
centre O. Now applying the theorem of Varignon to the force system,
F1 ¥ OA + F2 ¥ OB FR ¥ OC
( F1 + F2 ) ¥ OC
or F1 ¥ (OC - OA) F2 ¥ (OB - OC)
or F1 ¥ AC F2 ¥ BC
F1 BC
Hence, (2.2)
F2 AC
So it can be said that for two like parallel forces,
Fig. 2.3
the ratio of magnitude of two forces is the re-
ciprocal of the ratio of the normal distances of their lines of action from the
line of action of their resultant force.

2.2.2 Resultant of Two Unlike Parallel Forces


Let us consider F1 and F2 as two unlike parallel
forces acting normal to any incline at points A
and B, respectively, as shown in Fig. 2.4. Also
assume a point C on the incline which serves
as the point of action of resultant force FR. If
it is assumed that |F1| > |F2 |, from the concept
of resultant, FR F1 – F2. Consider an arbitrar-
ily chosen moment centre O. Now applying the
Fig. 2.4
theorem of Varignon to the force system,
Equilibrium of Forces 49

F1 ¥ OA - F2 ¥ OB FR ¥ OC ( F1 - F2 ) ¥ OC

or F1 ¥ (OC - OA) - F2 ¥ (OB - OC)


or F1 ¥ AC - F2 ¥ BC
F1 BC
Hence, (2.3)
- F2 AC
So it can be said that for two unlike parallel forces, the ratio of magnitude of
two forces is the reciprocal of the ratio of the normal distances of their lines
of action from the line of action of their resultant force.
If | F1| = | F2| = | F | , the resultant force becomes zero and the algebraic sum-
mation of the moment of the forces yields,
F1 × OA – F2 × OB F × (OA – OB) –F × AB MC (2.4)
The negative sign indicates the reversal of the direction of line of action of F2
only. This MC is defined as the moment of a couple.

2.3 PARALLEL SHIFTING OF FORCES


Sometimes, in order to meet the demand of computational ease, the line of
application of a force needs to be shifted parallel. As the new line of action
of force is parallel to the previous one, this approach is generally termed as
parallel shifting of forces.
Referring to Fig. 2.5, the body is subjected to a tensile force F and the
point of application is A. We will discuss how the line of action of this

Fig. 2.5

force F can be shifted parallel, so that the new line of action passes through
the new point of application B. We should always be careful so that the
effect of statical equivalence of forces is maintained. Let us place two col-
linear and opposite directional forces of magnitude |F | at point B, so that
their lines of action are parallel to the line of action of force F at point A.
The perpendicular distance between the lines of action of forces at points A
and B is d. Now observation suggests that upward force F at A and down-
ward force F at B constitute a couple and the moment of this couple is MC
F × d, acting at point B. Hence only an upward force F remains alone at
B. Maintaining the static equivalence, it can be finally said that the effect
of upward force F at A is equivalent to an upward force at B along with a
clockwise moment MC.
50 Engineering Mechanics

2.4 EQUIVALENCE OF FORCE AND FORCE–COUPLE SYSTEM


Using the principle of parallel shifting
of forces and extending further, we can
comfortably handle a generalized force
system, where a body is subjected to a
number of non-concurrent forces and
moments of couple. Such a general-
ized force system in three-dimensional
Cartesian coordinate system is shown
in Fig. 2.6, where F1, F2, ... are the
forces and MC1, MC2, ... etc., are the
moments of couple. Now we are inter-
ested to obtain a statical equivalence Fig. 2.6
of this force system at point O. Let the respective position vectors of the forces
be d1, d2, ... etc. Now the statical equivalence of the forces F1, F2, ... at point
O will be the vector sum of the forces FR F1, + F2, + ∙∙∙ along with the vec-
tor sum of the moments MO F1 × d1 + F2 × d2 + ∙∙∙. In addition, we have to
consider the vector sum of the moments of couple. Hence finally the statical
equivalence of the given force system at any arbitrary point O will be
FR F1 + F2 + ∙∙∙
and
MC (F1 × d1 + F2 × d2 + ∙∙∙) + (MC1 + MC2 + ∙∙∙)
(M1 + M2 + ∙∙∙) + (MC1 + MC2 + ∙∙∙) (2.5)
If necessary, we can resolve the above equations along the standard co-
ordinate axes. A force–moment system consisting of a force FR and a
moment MC which are parallel to each other is conventionally called a
wrench. Perhaps screwdriver would be a more appropriate term because
it is physically equivalent to push/pull along an axis along with a twist
about that axis. The line of action of FR is called the axis of the wrench,
MC
and the ratio is the pitch of the wrench. If FR and MC are of the same
FR
sense, the wrench is then called positive and viceversa. Inserting a screw with
a screwdriver is an example of positive wrench and pulling out a screw is an
example of negative wrench. When both FR and a moment MC are zero, we
obtain a null wrench or zero wrench.

2.5 GENERALIZED EQUATIONS OF EQUILIBRIUM


Let us consider a force system as shown in Fig. 2.6. Now if the body is to
maintain its static equilibrium, the necessary conditions are
(i) The resultant force FR should be zero, that is,
FR F1 + F2 + ∙∙∙ 0 (2.6)
(ii) The resultant moment of couple should be zero, that is,
MC (F1 × d1 + F2 × d2 + ∙∙∙) + (MC1 + MC2 + ∙∙∙)
(M1 + M2 + ∙∙∙) + (MC1 + MC2 + ∙∙∙) 0 (2.7)
Equilibrium of Forces 51

To attain this condition, summation of moments of forces and summation of


moments of couples should each be zero, that is,
M1 + M2 + ∙∙∙ 0 and MC1 + MC2 + ∙∙∙ 0
From these considerations, we can write the generalized equations of equi-
librium in three-dimensional coordinate system as,
i =n ¸ i =n ¸ i =n ¸
Â
i =1
( Fx )i = 0 Ô
Ô Â
i =1
( M x )i = 0 Ô
Ô Â
i =1
( M Cx )i = 0 Ô
Ô
i =n
Ô i =n
Ô i =n
Ô
Ô Ô Ô
Â
i =1
( Fy )i = 0 ˝ (2.8a)
Ô
Â
i =1
( M y )i = 0 ˝ (2.8b)
Ô
Â
i =1
( M Cy )i = 0 ˝ (2.8c)
Ô
i =n Ô i =n Ô i =n Ô
 ( Fz )i = 0 Ô
Ô Â ( M z )i = 0 Ô
Ô Â ( M Cz )i = 0 Ô
Ô
i =1 ˛Ô i =1 ˛Ô i =1 ˛Ô
The first set of equations is termed as force equilibrium equations along
the three mutually perpendicular directions, second set as equilibrium equa-
tions of moment of forces about axes, and third set as equilibrium equations
of moment of couples about axes.

2.6 ACTIVE AND REACTIVE FORCES


A force system acting on a particle or a rigid body consists of generalized
forces. Some of them are found to act externally and some are generated in-
ternally due to the effect of external stimuli. External forces are active force
components and internally built-up forces are reactive force components. Here
the term force is used in generalized sense which also includes moment, torque,
etc. In Fig. 2.7(a), a block rests on an incline and it is pulled by a force P. The
block is not weightless and so will be pulled by earth’s gravity. So here the ac-
tive forces are pull P and self-weight of the block W. Now we have to find the
reactive forces. If any real active force exists in the system, then reactive forces
would also exist, one or more and in some form or other. To resist the effect of
self-weight of the block, a resistive force R
will be generated from the inclined plane.
Again, the inclined plane being rough,
due to the pull P, a resistive force in the
form of friction will come to play at the
interface of the block and inclined plane. (a) (b)
Hence the reactive forces are R and fric- Fig. 2.7
tional force mR, as shown in Fig. 2.7(b).

2.7 DIFFERENT TYPES OF SUPPORTS


Theoretically we deal mainly with three types of supports, namely roller
support, hinge support, and fixed or clamped support. Roller has a natural
tendency to roll over the plane on which it is resting and it cannot move per-
pendicular to the plane. Hence reaction force generated from roller support
has only one component and acts normal to the plane on which the roller
52 Engineering Mechanics

rests. A hinge can move neither horizontally nor vertically but can rotate along
vertical axis. The attachment of a door plank with a door frame is a practical

(a)

(b)
Fig. 2.8
example of hinge support. The reaction force generated from a hinge or pin
support has two components, one along the plane and the other normal to the
plane on which the hinge rests. Sometimes, instead of showing the reactions
by resolution, the direction of hinge reaction is shown by an arbitrary zigzag
line, as if representing the resultant of horizontal and vertical reactions. It is
illustrated in Fig. 2.8(a). Reaction force generated from a clamped or fixed
support has three components, two forces along and normal to the plane
of fixity and one bending moment. The direction of moment (clockwise or
counter-clockwise) may be user-defined. It is shown in Fig. 2.8(b).

2.8 FREE-BODY DIAGRAMS


To solve any unknown component in a system of forces acting on a particle
or a rigid body maintaining equilibrium, it is mandatory to include the effects
of all active and reactive force components contributing to the system. It is
important to notice that there may exist some force components which are not
applied directly to the body. During analysis, those forces must be excluded.
To avoid erroneous analysis and results, we must be cautious to not include
any irrelevant force or exclude any relevant force. Now to draw the free-body
diagram (FBD), we should remove the rigid body(ies) from all sorts of sup-
ports and attachments and show the relevant forces along their proper lines
of action, if known. The term ‘free-body’ has been coined due to its detach-
ment from all sorts of supports and attachments. As a system may have two
or three or more components, we may have to draw the FBD of entire system
or FBD of all or relevant components. To draw the free-body diagram, the
following rules are to be kept in mind:
Equilibrium of Forces 53

(i) Self-weight of a particle or of a rigid body, irrespective of its position


(horizontal or inclined), always acts through its centre of gravity and
vertically downwards normal to the horizontal plane.
(ii) Axial forces in any member may be universally considered to be tensile.
If the result becomes negative, the force will have to be considered
compressive.
(iii) Support reaction always acts normal to the support.
(iv) Reaction force generated from a roller support has only one component
and acts vertically normal to the support on which the roller rests.
(v) Reaction force generated from a hinge or pin support has two
components, one along the plane and the other normal to the plane
on which the hinge rests. Sometimes, instead of showing the reactions
by resolution, the direction of hinge reaction is shown by an arbitrary
zigzag line.
(vi) Reaction force generated from a clamped or fixed support has three
components, two forces along and normal to the plane of fixity and
one bending moment. The direction of moment (clockwise or counter-
clockwise) may be user-defined.
(vii) Frictional resistance is shown by half-arrow lines, parallel to the plane
of motion or impending motion, in opposite direction. Being a fictitious
force, frictional resistance may be shown by dashed lines.
Finally a simplified and practical definition of the FBD can be stated as
follows
FBD Space diagram of the body with pertinent weight and applied indicated
loads – Supports and connections of the body + Reactions from sup-
ports and connections existing just before their removal
Following the above rules, some FBD of the force systems are shown in
Figs 2.7 and 2.8.

RECAPITULATION

z For a coplanar concurrent force system, if the angle included between F1 and F2
is g, F2 and F3 is a, and F3 and F1 is b, Lami’s theorem is expressed as
| F1| |F | |F |
= 2 = 3
sin a sin b sing
z Varignon’s theorem states that, the moment of the resultant of a number of concurrent
forces, about any given point, is equal to the algebraic summation of the moments of
all contributing forces about that same point.
z For two like or unlike parallel unequal forces, the ratio of magnitude of two forces
is the reciprocal of the ratio of the normal distances of their lines of action from
the line of action of their resultant force. But if the unlike parallel forces are equal
in magnitude, there exists no resultant force but a resultant moment of couple.
z A force–moment system consisting of a force FR and a moment MC which are
parallel to each other is conventionally called a wrench.
54 Engineering Mechanics

NUMERICAL EXAMPLES

Example 2.1 A traffic signal of mass 50 kg is hung with the help of two strings, as
shown in Fig. E2.1(a). Find the forces induced in the strings.

Fig. E2.1

Solution Forces in strings PR and PQ are designated as SPR and SPQ. From FBD of
the system [Fig. E2.1(b)], we obtain the equations of equilibrium
Æ≈

ÂF x 0: SPR cos 40° SPQ cos 65°

or SPR 0.55168 SPQ (1)



≠ ÂF y 0: SPR sin 40° + SPQ sin 65° 50 × 9.807
Substituting from Eq. (1), we get
0.55168 SPQ sin 40° + SPQ sin 65° 50 × 9.807
or SPQ 388.88 N
From Eq. (1),
SPR 214.54 N
Otherwise, we can solve this problem using Lami’s theorem
SPR SPQ 50 ¥ 9.807
sin (90∞ + 65∞) sin (90∞+ 40∞) sin (180∞- 40∞- 65∞)

sin 155∞
Hence, SPR (50 × 9.807) 214.54 N
sin 75∞
sin 130∞
and SPQ (50 × 9.807) 388.88 N
sin 75∞
Example 2.2 A cylindrical wooden log of 1200 kg is kept within inclined planes,
which are mutually perpendicular [Fig. E2.2(a)]. Considering smooth contact surfaces,
determine the forces of reaction.
Solution Use Lami’s theorem, referring to FBD of log [Fig. E2.2(b)]
RP RQ 1200 ¥ 9.807
=
sin (90∞ + 38∞) sin (90∞+ 52∞) sin (180∞- 38∞- 52∞)
Equilibrium of Forces  55

Fig. E2.2
Hence, RP = 9273.63 N
RQ = 7245.35 N
Example 2.3  A ball is supported on a
smooth wall by tying a rod, as in Fig.
E2.3(a). Find the tension in the tie rod
and support of the wall. Take radius of
ball 5 cm, length of the rod 13 cm, and
weight 120 N. 5 cm
Solution  From geometry [Fig. E2.3(b)],
Ê ˆ
q = sin–1 Á 5 ˜ = 22.62°
Ë 13 ¯
Use Lami’s theorem, following the FBD Fig. E2.3
of the ball
RC S = 120
=
sin (180∞ - q ) sin 90∞ sin[180∞ - (90∞ - q )]
RC S = 120
or =
sin157.38∞ sin 90∞ sin112.62∞

Hence, RC = 50 N  and  S = 130 N


Example 2.4  An electric light fixture is held with the arrangement shown in
Fig. E2.4(a). If the weight of the fixture is 20 kg and the hinge is an ideal one, determine
the axial force in the bar and the weight W suspended from pulley Q required to hold
the string stable.
Q

W RH

RV

22º M

R P
FBD of point M FBD of pulley Q
(a)

Fig. E2.4
56 Engineering Mechanics

Solution Consider FBD of pulley and find W S1.


Consider FBD of point M [Fig. E2.4(b)] and therefore the equations of equilibrium are
Æ≈

ÂF x 0: S2 –S1 cos 22° (1)


≠ ÂF y
0: S1 sin 22° 20 × 9.807
Solving, we get S1 523.59 N.
From Eq. (1), S2 – 485.46 N 485.46 N compressive.
Thus W S1 523.59 N.
Example 2.5 Two identical iron spheres, each of radius 5 cm and weight 150 N are
connected with a string of length 16 cm, and rest on a horizontal smooth floor. Another
sphere of radius 6 cm and weight 200 N rest over them. Determine the tension in the
string and reaction at all contact surfaces [Fig. E2.5(a)].

Fig. E2.5
Solution As the bottom spheres are identical, the reactive forces with the top sphere
will be identical. Also floor reactions will be identical. From FBD of the left bottom
sphere [Fig. E2.5(b)], the equations of equilibrium are
Æ≈

ÂF x 0: S R cos q (1)


≠ ÂF y 0: R1 150 + R sin q (2)
From FBD of top sphere [Fig. E2.5(c)], the equation of equilibrium is

≠ ÂF y 0: 2R sin q 200
100
or R (3)
sin q
Equilibrium of Forces  57

From an auxiliary diagram of right-angled DCC1C2 [Fig. E2.5(d)], C1C2 = 16 cm, CC1
= CC2 = 5 + 6 = 11 cm. So,
16 / 2 = 8
cos q =
11 11
or  q = 43.34°

From Eqs (3), (2), and (1), we get R = 145.7 N, R1 = 250 N, and S = 105.97 N,
respectively.
Example 2.6  Two spheres are kept within a conical channel, as shown in Fig. E2.6(a).
All contact surfaces are smooth. Determine all contact reactions. Size of spheres are
same but have different weights.

Fig. E2.6

Solution  Considering FBD of top sphere [Fig. E2.6(b)], apply Lami’s theorem
200 R RM
= =
sin 90∞ sin (90∞ + 63∞) sin (90∞ + 27∞)

Solving, we get R = 90.8 N  and  RM = 178.2 N


Consider FBD of bottom sphere [Fig. E2.6(c)], equations of equilibrium are

≠ ÂF y = 0: RQ sin 63° – 150 – R sin 27° = 0

or RQ = 214.61 N
Æ≈

  ÂF x = 0: R cos 27° + RQ cos 63° = RP

Hence, RP = 178.33 N

Example 2.7  A block of 3 kN weight is suspended from a framework as shown in


Fig. E2.7(a). Determine the forces induced in guy wires AB and PR. Also determine
the string forces at CB and CP.

Solution  From the geometry of right-angled triangle BCP [Fig. E2.7(b)], the sag of point
C = 1.9 m and a + b = 90°. Therefore,

x = 1.9 and (6.5 – x) = 1.9


tan a tan b
58 Engineering Mechanics

Fig. E2.7
1.9 1.9
Hence, 65 tan a + tan b
Ê ˆ Ê ˆ
1.9 Á 1 + 1 ˜ 1.9 Á tan a + 1 ˜
Ë tan a tan b ¯ Ë tan a ¯
1 + tan 2 a
or 3.421 or tan2 a – 3.421 tan a + 1 0
tan a
or tan a 3.098 or 0.3227
or a 72.11° or 17.88°
Both values of a are real and possible. But following geometry, as a > b, so we take
a 72.11° and b 17.88°.
Consider FBD of joint C [Fig. E2.7(c)],
Æ≈

ÂF x 0: SCB cos a SCP cos b

cos 17.88∞
So, SCB SCP 3.098 SCP
cos 72.11∞

≠ ÂF y 0: SCB sin a + SCP sin b 3000

Hence, 3.098 SCP sin 72.11° + SCP sin 17.88° 3000


or SCP 921.59 N
and SCB 3.098 SCP 2855.09 N
Consider FBD of joint B [Fig. E2.7(d)],
Æ≈

ÂF x 0: SCB cos a SAB sin 25°

So, SAB 2075.29 N


Equilibrium of Forces 59

Consider FBD of joint P [Fig. E2.7(e)],


Æ≈

ÂF x 0: SCP cos b SPR sin 30°

or SPR 1754.16 N
Example 2.8 Two weights are suspended from B and C points of a rope as shown
in Fig. E2.8(a). If distance AD is 6 m, how much will be the magnitude of W to
maintain the equilibrium?

Fig. E2.8

Solution Let AE x and BE CF h.


So, FD (6 – x –3.1) m (2.9 – x) m
From right DABE,
x2 + h2 2.32 5.29 (1)
From right DCFD,
(2.9 – x)2 + h2 1.72
or 8.41 – 5.8x + x2 + h2 2.89
Substituting the value of (x2 + h2) from Eq. (1):
8.41 – 5.8x + 5.29 2.89 or x 1.864 m
Ê ˆ
So, a cos–1 ÊÁ 1.864 ˆ˜ 35.86° and b cos–1 6 - 1.864 - 3.1 52.45°
Ë 2.3 ¯ ÁË 1.7 ˜¯

Consider FBD of joint B [Fig. E2.8(b)],



≠ ÂF y
0: SAB sin a 3.2

So, SAB 5.463 kN


Æ≈

ÂF x 0: SBC SAB cos a 4.427 kN


60  Engineering Mechanics

Consider FBD of joint D [Fig. E2.8(c)],


Æ≈

ÂF x = 0: SBC = SCD cos b

So,   SCD = 7.264 kN



≠ ÂF y
= 0:  W = SCD sin b = 5.76 kN

Example 2.9  Two weights, 20 kN and 30 kN are suspended as shown in Fig. E2.9(a).
Determine the balancing weight W, if the pulleys are assumed to be smooth.
R
HQ HR

Q 65° a R VQ SPQ VR
SPR
W P
30 kN
20 kN
(a)
Fig. E2.9
SPQ SPR
Solution  FBD of pulley Q gives W = SPQ. PQ PR

FBD of pulley R gives SPQ = 30 kN. 65° a


Now consider FBD of joint P [Fig. E2.9(c)]
P
Æ≈
  ÂF x = 0:  SPR sin a = SPQ sin 65°(1)
20 kN
≈ (c) FBD of joint P
≠ Â
Fy = 0: SPQ cos 65° + SPR cos a = 20 or SPQ cos 65° = 20 – SPR cos a
(2)
Dividing Eq. (1) by Eq. (2), we obtain
30sin a 3sin a
tan 65° = or  2.1445 =
20 - 30cos a 2 - 3cos a
Solving the above equations by trial and error method, we get a = 77.83°.
Now, from Eq. (1),
SPQ = 32.357 kN
Thus W = SPQ = 32.357 kN
Example 2.10  Two spheres weighing 60 N and 100 N are connected by a flexible
string AB and rest on two mutually perpendicular planes PQ and QR [Fig. E2.10(a)].
Find the tension in the string which passes freely through slots in smooth inclined
planes PQ and QR.

Solution  Consider FBD of sphere A [Fig. E2.10(b)], and apply Lami’s theorem,
SAB RA 60
= =
sin120∞ sin (90∞ + f ) sin (180 - 30∞ - f )
Equilibrium of Forces  61

Fig. E2.10

Hence, SAB = 51.962 (1)


cos(60∞ - f )
Now consider FBD of sphere B [Fig. E2.10(c)], and apply Lami’s theorem,
SAB RB 100
= =
sin150∞ sin (90∞ - f ) sin (90∞ + 30∞ + f )

Hence, SAB = 50 (2)


cos (30∞ + f )
Comparing Eqs (1) and (2), we get
51.962 = 50
cos(60∞ - f ) cos(30∞ + f )
On solving, we get f = 16.102°
Now from Eq. (2), SAB = 72.11 N.
Example 2.11  In the arrangement shown in Fig. E2.11(a), all pulleys are assumed to
be frictionless. Determine the angle a and the reactive force on the ball from the floor.

HA HB

VA VB

Fig. E2.11
Solution  Considering FBD of pulley A [Fig. E2.11(b)], SAC = 40g
Considering FBD of pulley B [Fig. E2.11(c)], SBC = 30g
62  Engineering Mechanics

Now consider FBD of ball [Fig. E2.11(d)], the equations of equilibrium are
Æ≈

ÂF x = 0: SBC = SAC  cos a

By putting the values, we get


30g = 40g × cos a
3
or cos a = or  a = 41.41°
4

Again, ≠ ÂF y
= 0:  SAC sin a + R = 85g
or 40g sin 41.41° + R = 85g
or R = 58.54g = 574.12 N
Example 2.12  A rigid bar AB with balls of weights W1 = 75 N and W2 = 125 N at
ends is supported inside a circular ring as shown in Fig. E2.12(a). Radius of the ring
and AB are such that radii AC and BC make right angle at the centre of the ring C.
Neglecting friction and weight of AB, ascertain equilibrium configuration defined by
angle [(a – b)/2] that AB makes with the horizontal. Find the contact reaction at A
and B, and axial force in rod AB.

Fig. E2.12

Solution  From the condition of inclination of AB,


a -b
q= or  q + b = a – q
2
Consider FBD of left ball [Fig. E2.12(b)]
Æ≈

ÂF x = 0:    SAB cos q = R1 sin a (1)


≠ ÂF y = 0:  R1 cos a + SAB sin q = W1 = 75 (2)
From Eqs (1) and (2),
Equilibrium of Forces 63

cos q cos(a - q )
75 SAB sin q +SAB cos a SAB (3)
sin a sin a
Consider FBD of right ball [Fig. E2.12(c)]
Æ≈

ÂF x 0: SAB cos q R2 sin b (4)


≠ ÂF y 0: R2 cos b – SAB sin q 125 (5)

From Eqs (4) and (5),


cos q cos(q + b )
125 SAB cos b – SAB sin q SAB (6)
sin b sin b
From Eqs (3) and (6),

125 cos(q + b ) sin a sin a sin a sin a


¥ = =
75 sin b cos(a - q ) sin b sin (90∞ - a ) cos a

or tan a 5 or a 59.04°
3
So, b 30.96° and q 14.04°
From Eqs (3), (1), and (4), SAB 90.95 N, R1 102.89 N, R2 171.51 N, respec-
tively.
Example 2.13 A ladder of weight 30 kg is supported at wall and floor as shown in
Fig. E2.13(a). A man of weight 72 kg rides on a rung 8 m above floor level. Considering
all contact surfaces smooth, determine the reactions at P and Q.

Fig. E2.13

Solution Here ∆PQC and ∆MQF are similar [see Fig. E2.13(b)]. Therefore,
PC QC
MF QF
Putting the values, we get
10 m 5m
or QF 4m
8m QF
64 Engineering Mechanics

Again, ∆PQC and ∆BQE are similar, self-weight of ladder PQ acts at B, the middle
point. So, QE QC/2 2.5 m.
Consider moment equilibrium at Q,

ÂM Q
†
0: 30g × 2.5 + 72g × 4 – RP × 10 0

So, RP 36.3g 355.99 N


Now from equations of force equilibrium,
Æ

ÂF x 0: HQ RP 355.99 N


≠ ÂF y 0: VQ 30g + 72g 1000.31 N

So, reaction at Q, RQ H Q2 + VQ2 1061.77 N and inclination with horizontal, q


–1
tan (VQ/HQ) 70.41°.

Example 2.14 A light rod AB 200 mm long rests on two pegs C and D 100 mm apart.
How must it be placed so that the reactions of the pegs may be equal when weights
of 2W and 3W are suspended from A and B, respectively?
A C D B A C D B
100 mm
2W 3W
2W RC R 3W
x
200 mm 100 mm
100–x
(a) (b)

Fig. E2.14

Solution Total loads 2W + 3W 5W


As per given condition, reaction of each peg RC 5W 2.5 W RD
2
To achieve equal reaction, let the peg C is placed at a distance x from the left end and
thus AC x and DB 100 – x
Consider moment equilibrium at C

ÂM at C
†
0: 3W (100 + 100 – x) – RD × 100 – 2Wx 0

or 3W (200 - x ) - 2.5 W ¥ 100 - 2Wx = 0


or 600 – 3x – 250 – 2x 0
or 5x 350 or x 70
So, the required distance is 70 mm.
Example 2.15 A weightless beam BCD is held with the help of a tie rod AC
[Fig. E2.15(a)]. Determine the reaction at B and tension in the tie rod AC.
Solution From geometry [Fig. E2.15(b)],
Ê ˆ
tan -1 Á 2.3 ˜ 47.6°
q
Ë 2.1 ¯
Consider moment equilibrium at B,
ÂM at C
†
0: 2 × 4.4 – SAC sin q × 2.1 0
Equilibrium of Forces 65

Fig. E2.15

Hence, SAC 5.67 kN


From equations of force equilibrium,

≠ ÂF y 0: VB + SAC sin q – 2 0
So, VB –2.187 kN 2.187 kN downwards
Æ≈
and ÂF x 0: HB SAC cos q 3.823 kN

So, reaction at B, RB 2.187 2 + 3.8232 4.40 kN and inclination with the horizontal
q tan–1 (VB/HB) 29.77°.

Example 2.16 A heavy cylinder of mass 280 kg is to be pulled over a curb of height
5 cm by a horizontal force F applied by means of a rope wound around the cylinder
[Fig. E2.16(a)]. Determine the magnitude of pull for impending motion over the curb,
while the radius of the cylinder is 13 cm.

Fig. E2.16

Solution At the instant of start of movement of the cylinder, it will lose contact with
floor. Only the reaction R acting at peak of curb will be effective. In this situation R,
F, and self-weight of the cylinder will be concurrent.
From geometry [Fig. E2.16(b)],
13 - 5
sin q or q 37.98°
13
Consider moment equilibrium at curb-peak,
280g × 13 cos q F × (13 + 13 – 5)
Hence, F 1339.89 N
66  Engineering Mechanics

Example 2.17  A prismatic bar of length 11 m and mass 21 kg is hinged with vertical
wall at B and is tied at the other end with a strut AC [Fig. E2.17(a)]. Compute the
compressive force induced in the strut and the reaction at hinge.

Fig. E2.17

Solution  From geometry [Fig. E2.17(b)],


AC = 11 cos 23° = 10.126 m  and  AB = 11 sin 23° = 4.298 m
Consider moment equilibrium at A,

ÂM ⊕
= 0:  HB × 4.298 = 21g × 10.126
A
2
Therefore, HB = 242.6 N
From equations of force equilibrium,
Æ≈

ÂF x = 0: SAC = HB = 242.6 N


≠ ÂF y = 0: VB = 21g = 205.947 N

So, the reaction at hinge RB = 205.947 2 + 242.62 = 318.23 N and inclination with
horizontal, q = tan–1 (205.947/242.6) = 40.328°.

Example 2.18  A rod ABC is hinged at C and is supported by a strut BD. If 50 kN pull
is applied at A [Fig. E2.18(a)], determine the reaction at C and reaction at strut.

Fig. E2.18

Solution  From geometry [Fig. E2.18(b)],


Ê ˆ
q = tan -1 Á 3 ˜ = 36.87°
Ë 4¯
Equilibrium of Forces 67

If the system is to maintain equilibrium, RC, SDB, and 50 kN forces must be concur-
rent at P. Applying Lami’s theorem,
50 SDB RC
=
sin (90∞ - q ) sin (180∞ - 2q ) sin (90° - q )
Hence,
RC 50 kN and SDB 60 kN
Example 2.19 A crane is hinged at P and is supported by a guide at Q. Determine the
reaction produced at P and Q, if W 7.5 kN is applied at point R [Fig. E2.19(a)].

Fig. E2.19

Solution Consider moment equilibrium at P [Fig. E2.19(b)],


RQ × 1.7 7.5 × 2.8 12.35 kN
Applying equations of force equilibrium,
Æ≈

ÂF x 0: HP RQ 12.35 kN


≠ ÂF y 0: VP W 7.5 kN

So, reaction at P, RP 12.352 + 7.52 14.45 kN and inclination with the horizontal
q tan–1 (7.5/12.35) 31.27°.

Example 2.20 A weightless 13 m long bar BA is supported as shown in Fig. E2.20(a).


Anywhere along its length, 2.1 kN vertical load is applied. Determine the position
of this load along the bar for which tension in the cable AC becomes maximum and
also find the value of maximum tension.
Solution Consider FBD of pulley, we see, T SAC.
Let us assume the vertical load acts x away from B, along the length of the bar
[Fig. E2.20(b)]. Now, consider moment equilibrium at point B,
2.1 × x cos 55° – SAC sin 75° × 13 cos 55° + SAC cos 75° × 13 sin 55° 0
or SAC 0.2709x
To attain a maximum value of SAC, x must possess a maximum value. The maximum
possible value of x is 13 m. So, in this condition, SAC 3.5217 kN.
68  Engineering Mechanics

D
HC

VC
2.1 kN

20°
55°
A
(a)
Fig. E2.20
Example 2.21  A nut cracker is shown in Fig. E2.21. If 210 N force is applied at the
handle, how much force will be available to break betel nuts?

Fig. E2.21
Solution  Consider moment equilibrium at the hinge of the cracker. If the force at
the tip is F, we get
210 × 103 = 18 × F
Hence, F = 1201.67 N

Example 2.22  A nail remover is used with a force of 180 N applied at handle as
shown in Fig. E2.22(a). Determine the magnitude of pull acting on the nail.

RB

Fig. E2.22
Equilibrium of Forces 69

Solution Consider moment equilibrium at contact point B [Fig. E2.22(b)]. Let us


assume the pull on the nail is R. Therefore,
180 × 35 R cos 18° × 4
Hence, R 1656.05
Example 2.23 In the L-bracket, as shown in Fig. E2.23(a), the bolt A fits loosely in
a vertical slot. Determine the reaction at A and B.
HA
A C

A C
380 N
17cm
21cm 380 N HB
B
B
VB
FBD of L-bracket
Fig. E2.23
Solution As bolt A can make up-down movement, the only restraining force will be
a horizontal one, HA [Fig. E2.23(b)]. Take moment equilibrium at B,
HA × 21 –380 × 17
or HA –307.62 N 307.62 N leftwards
Consider force equilibrium equation on bracket,

ÂF
≠ y
0: VB 380 N
Æ≈

ÂF x 0: HB HA –307.62 N 307.62 N rightwards

So, reaction at B, RB 3802 + 307.622 488.9 N and inclination with the horizontal,
q tan–1 (380/307.62) 51°.
Example 2.24 In the spanner wrench, as shown in Fig. E2.24(a), a force of 250 N
is applied at handle. How much force will be exerted on the cylinder at points M and
N? Consider the radius of cylinder to be 7 cm.

Fig. E2.24
70 Engineering Mechanics

Solution Consider moment equilibrium at M,


RN × 7 35 × 250 or RN 1250 N
Use equations of force equilibrium of the wrench [Fig. E2.24(b)]
Æ≈

ÂF x 0: HM RN 125 N


≠ ÂF y 0: VM 250 N

So, reaction at M, RM 12502 + 2502 1274.75 N and inclination with the hori-
–1 (250/1250)
zontal q tan 11.31°.
Example 2.25 Two beams AD and BE are arranged and supported as shown in
Fig. E2.25(a). Determine the reaction at A.
270 N

E 37
B
D C A

55°

1.1 m 1.2 m 1.3 m 1.4 m

Fig. E2.25
Solution Consider moment equilibrium at E of beam BE [Fig. E2.25(b)],
RC × (1.1 + 1.2) 270 sin 37° × (1.1 + 1.2 + 1.3)
RC 254.33 N
Consider moment equilibrium at D of beam AD [Fig. E2.25(c)],
RC × 1.2 RA sin 35° × (1.2 + 1.3 + 1.4) or RA 136.43 N
Example 2.26 A smooth right circular cylinder of radius 16 cm rests on the horizontal
plane and is kept from rolling by an inclined string PC of length 32 cm. A prismatic bar
PQM of length 48 cm and weight 530 N is hinged at P and leans against the cylinder,
as shown in Fig. E2.26(a). Determine the tension in the string PC.
Equilibrium of Forces 71

Fig. E2.26
Solution From geometry of the system [Fig. E2.26(b)],
CQ 16
sin q 0.5 or q 30°
PC 32
Here,
48
PN NM 24 cm
2

and PQ PC2 - CQ 2 322 - 162 27.71 cm


Considering FBD of bar, take the moment equilibrium at P [Fig. E2.26(c)]
530 cos 2q × PN RQ × PQ
Putting the value of q, we get,
530 cos 60° × 24 RQ × 27.71 or RQ 229.52 N
Again from horizontal force equilibrium of the cylinder,
Æ≈

ÂF x 0: RQ cos q SPC cos q

or SPC RQ 229.52 N
Example 2.27 Two halves of a round homogeneous cylinder are held together by a
thread wrapped around the cylinder with two weights P, each attached to its ends
as shown in Fig. E2.27(a). The complete cylinder weighs W Newton. The plane of
contact of both of its halves is vertical. Determine the minimum value of P for which
both halves of the cylinder will be in equilibrium on a horizontal plane.
Solution If the two halves remain on the verge of separation, then only the holding force
(weight) will be minimum. At this stage, either left or right half can be analysed. The free
body diagram of the left half is shown in Fig. E2.27(b). As both the halves have separate
entity, self-weight of each will be 0.5 W N and will act at the centroid of hemisphere.
72 Engineering Mechanics

A
P

G
O
0.5W
4r
3p
C
P P
P RC
(a) (b) RF

Fig. E2.27
At the stage of separation, the two halves will maintain the contact at bottom-most
point and thus appears RC in the free body diagram. Assume the radius of cylinder as r.
Consider

ÂM at C
†
0
4r
P × 2r – P × r – 0.5W × 0
3p
or P = 2W = 0.2122W N
3p
Example 2.28 A circular log of weight 1200 N and radius 18 cm is supported by a
pair of bracket, one of which is shown in Fig. E2.28(a). Bar PN hinged at P and held
by a string MN is 57 cm long. To induce minimum tension at MN, determine the value
of 2q, as shown, for equilibrium. Consider all contact surfaces smooth. Also find the
value of minimum tension.

Fig. E2.28
Equilibrium of Forces 73

Solution On each bracket, the effective weight of log will be 600 N. Consider FBD
of log [Fig. E2.28(b)],

≠ ÂF y 0: RJ sin 2q 600
600
or RJ
sin 2q
Consider FBD of bar PN [Fig. E2.28(c)]. From simple geometry,
PJ 18 cot q and PM 57 cos 2q
Using moment equilibrium equation at P, we obtain
T × PM RJ × PJ
600
or T × 57 cos 2q × 18 cot q
sin 2q
600 ¥ 18 cot q
Hence, T ¥
57 sin2q cos 2q
600 ¥ 18 cosq
¥
57 sinq ¥ 2sinq cosq ¥ cos 2q
Ê 600 ¥ 18 ˆ 1
ÁË 57 ˜¯ ¥ 2sin2 q cos 2q (1)

If T is to attain a minimum value, (2 sin2 q cos 2q) must posses a maximum value.
Hence, d (2 sin2 q cos 2q) 0
dq
or d [(1 – cos 2q) cos 2q] 0 or –2 sin 2q + 4 cos 2q sin 2q 0
dq
or sin 2q × (2 cos 2q – 1) 0
As, sin 2q ≠ 0,
2 cos 2q – 1 0 or q cos–1 1 60°
2
600 ¥ 18 1
Now from Eq. (1), Tmin ¥ 757.89 N
57 2sin 2 30∞ cos60∞
Example 2.29 ABCD is a square of side 6 m. Forces acting along AB, BC, CD, DA,
AC and DB are 1.5 kN, 3 kN, 12 kN, 7.5 kN, 7.5 2 kN, and 3 2 kN, respectively.
Show that the resultant of this force system is not a force, but a moment of 72 kNm
anticlockwise.
A 1.5 kN B

7.5—2 kN

7.5 kN O 3 kN

6m 3—2 kN

D 6m 12 kN C
Fig. E2.29
Solution Considering
Æ≈

ÂF x =0: 1.5 – 12 + 7.5 2 cos 45∞ + 3 2 cos 45∞ = 0


74 Engineering Mechanics


≠ ÂF y
0: 7.5 - 3 - 7.5 2 sin 45∞ + 3 2 sin 45∞ = 0
From the above results, we can infer that no resultant force exists for this force system.
Let the point of intersection of diagonals AC and DB be O. Total moment of all
forces at O is computed as
MO 1.5 × 3+3 × 3 + 12 × 3 + 7.5 × 3 72 kN m clockwise
Therefore the resultant will be a moment of 72 kN m anticlockwise (Proved).
Example 2.30 Figure E2.30(a) shows a body under the action of coplanar forces.
Determine the magnitude, direction, and position of a single force which will keep
the body in equilibrium.

10 N 20 N RF
10 N
20 N
45° D C 30° D C
45° 30°

90°
60°
E
12 m F
60°
d
30°
30° 45° 30°
A B A B
20 N 15 N 20 N (b)
(a) 15 N
Fig. E2.30

Solution From the arrangement of the forces, it is evident that 20 N forces form
a couple with lever arm distance CE and acting clockwise. In addition, a force of
(15 N – 10 N) 5 N acts along line DB. So, the square ABCD is subjected to a 5 N
force along DB and a couple (20N × CE) acting clockwise. Our intention is to find
out a single force which will keep the square in equilibrium. Let that force be RF
and its line of action be parallel to line DB and at a normal distance d. To obtain
equilibrium, the magnitude of RF should be 5 N, such that the governing condition
for equilibrium becomes
5N ¥ d = 20 N ¥ CE (1)

From ∆ABF, BF = ABtan 30∞ = 12 tan 30∞

Thus, CF = BC - BF = 12 - 12 tan 30∞ = 12(1 - tan 30∞)

So, CE = CFsin 60∞ = 12(1 - tan 30∞)sin 60∞ = 4.392m


From Eq. (1), we obtain, 5 × d 20 × 4.392 or d = 17.569 m

Example 2.31 Two gears with pitch diameters 20 cm and 15 cm are connected with
an idler [Fig. E2.31(a)]. If moment M1 1334 N m is applied on top gear, how much
moment will be required to apply at bottom gear so that equilibrium condition is
established?
Solution From FBD of top gear [Fig. E2.31(b)], we can determine the force P causing the
moment M1 to be equal to 1334/0.2 N.
Again from FBD of bottom gear, the active moment generated is
Ê 1334 ˆ
ÁË 0.2 ¥ 0.15˜¯ N m 1000.5 N m
Equilibrium of Forces 75

M1 15 cm

C1 C2 C3
(i)

20 cm (ii)
(a)
Fig. E2.31
and the sense is counterclockwise. So, the external moment necessary will be of amount
1000.5 N m and sense clockwise.
Example 2.32 Beam MJN supports beam PQR with the help of a suitable truss as shown
in Fig. E2.32(a). Determine the reactions at supports M and N of the lower beam.
150 N

P Q

D E

M
N
J

2.2 m 2.2 m 1.8 m

(a)
150 N

(150 × 1.8) Nm
P
Q
C

D E

M J

2.2 m 2.2 m

(b)
150 N

(150 × 1.8) Nm
M

RM RN
(c)
Fig. E2.32
76 Engineering Mechanics

Solution The given loaded system can be reduced to the system shown in Fig. E2.32(b).
Corresponding FBD of beam MN is shown in Fig. E2.32(c). Consider moment
equilibrium at M,
RN × 4.4 – 150 × 4.4 – 150 × 1.8 0
or RN 211.36 N

≠ ÂF y
0: RM + RN 150

RM 150 – RN –61.36 N or 61.36 N downwards


Example 2.33 A rigid T is made out of metal bars MN and PQ, each 1.4 m long
and weigh 40 kg and 30 kg, respectively. It is suspended in a vertical plane. Compute
angle f for equilibrant, subjected to a load of 500 N [Fig. E2.33(a)].

Fig. E2.33

Solution ∆From FBD of T shown in Fig. E2.33(b), PB BQ 0.7 m. So,


BE BQ sin f 0.7 sin f
PA PQ sin f 1.4 sin f
Again from ∆PCN, PC PN cos f 0.7 cos f.
Therefore,
AC PC – PA 0.7(cos f – 2 sin f)
Consider moment equilibrium of frame T at Q.
40g × PA + 30g × BE 500 × AC
or 40g × 1.4 sin f + 30g × 0.7 sin f 500 × 0.7(cos f – 2 sin f)
Solving, we get
80g sin f + 30g sin f 500 cos f – 1000 sin f
or (110g + 1000) sin f 500 cos f
or tan f 0.2405
Hence, f 13.52°
Example 2.34 A uniform rod BC of length l and weight W is suspended from ceiling
point A with the help of two equally long chords AC and AB. If a concentrated
moment Mo is applied on the rod, determine its inclination b with the horizontal. If
both the chords are to remain taut, prove that Mo < 0.5 Wl sin a.
Solution From the free body diagram of rod BC, as shown in Fig. E2.34(b), we
obtain
Equilibrium of Forces 77

A S2

b a M0
S1
a M0 G
b l/ a
2
G b
W C
a l/
W 2
b C
(a) (b)

Fig. E2.34
Æ≈

ÂF x 0: S1cos(a + b) S2 cos (a – b) (1)


≠ ÂF y 0 S1sin(a + b) + S2 sin (a – b) W
S1 cos(a + b )
Thus, S1 sin(a + b ) + sin(a - b ) = W [substituting from Eq. (1)]
cos(a - b )

or S1 ÈÎsin(a + b )cos(a - b ) + cos(a + b )sin(a - b ) ˘˚ = W cos(a - b )

or S1 sin 2a W cos(a – b)
W cos(a - b )
or S1 = (2)
sin 2a
S1 cos(a + b ) W cos(a - b ) cos(a + b ) W cos(a + b )
So, S2 = = ¥ = (3)
cos(a - b ) sin 2a cos(a - b ) sin 2a

Again, ÂM at G
†
0: S2 sin a ¥ l - S1 sin a ¥ l + M o = 0
2 2
2M o
or S1 - S2 = (4)
l sin a
W cos(a - b ) W cos(a + b ) 2 M o
or - =
sin 2a sin 2a l sin a

or W È cos(a - b ) - cos(a + b ) ˘ = 2 M o
sin 2a Î ˚ l sin a

W 2M o
or 2sin a sin b =
2sin a cos a l sin a
2M o
or sin b = (5)
Wl tan a
Ê 2M o ˆ
or b = sin -1 Á ˜
Ë Wl tan a ¯
Equation (4) reveals that if Mo increases, S2 must have to decrease, assuming S1 as
invariant. If we consider the lowest value of S2, i.e., S2 0, the remaining forces are S1
and W. In that condition, to maintain equilibrium, both S1 and W have to be collinear.
Hence, a + b 90° and thus b 90° –a. Applying this value in Eq. (5), we obtain
78 Engineering Mechanics

2M o 2M o
sin(90∞ - a ) = or cos a =
Wl tan a Wl tan a
or M o = 0.5Wl sin a (6)

Therefore, it is quite obvious from Eq. (6) that to maintain both the chords taut the
necessary condition is Mo< 0.5 Wl sina.
Example 2.35 Three beams, hinged together at their ends, are supported and loaded,
as shown in Fig. E2.35(a). Determine the reactions at M, N, V, W.
700 N 800 N 900 N

M N P T Q V W

1.0 m 0.8 m 1.3 m 0.7 m 0.8 m 0.9 m

(a)
800 N 900 N 700 N 800 N 900 N
V W P T Q V W
Q

RV RN RV RW
FBD of beam QW (b) FBD of beam PQ and QW
700 N 800 N 900 N

M N P T Q V W

RM RN RV RW
(c)
FBD of all three beams
Fig. E2.35

Solution From FBD of beam QW [Fig. E2.35(b)], consider moment equilibrium at Q


0.8 × 900 – 0.8 × RV – 1.7 × RW 0
8RV + 17RW 7200 (1)
Considering FBD of beams PQ and QW [Fig. E2.35(c)], and taking moment equi-
librium at P
1.3 × 700 + 2.0 × 800 + 2.8 × 900 2.8 RV + 3.7 RW
28 RV + 37 RW 50300 (2)
Solving Eqs (1) and (2), we obtain, RV 3270.56 N, RW –1115.56 N.
Now consider FBD of all three beams [Fig. E2.35(d)]. Take moment equilibrium at M.
–1 × RN + 3.1 × 700 + 3.8 × 800 + 4.6 × 900 – 4.6 × RV – 5.5 × RW 0
or RN 441.004 N
Equilibrium of Forces  79

Considering vertical force equilibrium equation,



≠ ÂF y
= 0: RM + 441.004 + 3270.56 – 1115.56 = 700 + 800 + 900

or RM = –196.004 N
From the result, it is obvious that the assumed directions of RM and RW are opposite.
Example 2.36  Two metallic rods PQ and QR are fused within internal angle 55°,
and hung as shown in Fig. E2.36(a), such that QR makes an angle q in equilibrium
condition. Determine the angle q.
Solution  Consider the FBD of the entire frame and take moment equilibrium at P
[Fig. E2.36(b)]
PN × 115 = PM × 250 (1)

Fig. E2.36
From geometry, PB = BQ = 1.15/2 m,
RC = CQ = 2.5 m, ÐBQD = ÐBPN = 55° – q
2
From ∆PBN,  PN = PB cos (55° – q) and
from ∆CDQ,  PM = QD – PN = CQ cos q – PN
Substituting in Eq. (1)
PB cos (55° – q) × 115 = [CQ cos q – PB cos (55° – q)] × 250

or 1.15 cos (55° – q) × 115 = È 2.5 cos q - 1.15 cos(55∞ - q ) ˘ × 250


2 ÍÎ 2 2 ˙˚
or 1.679 cos (55° – q) = 2.5 cos q
cos(55∞ - q )
or = 1.4889
cos q
or tan q = 1.1174  or  q = 48.17°
Example 2.37 A ladle is lifted by means of three sling-chains O
each 0.9144 m in length. The upper ends of the chains are
attached to a ring, while the lower ends are attached to three a
hooks fixed to the ladle, forming an equilateral triangle of
A
1.2192 m side. If the weight of the ladle and its content is
45 kN, find the load taken up by each chain. C D
G
Solution  Let us assume A, B, and C are the hooks attached
to the ladle and O the ring. So, OA = OB = OC = 0.9144 m
B
and AB = BC = CA = 1.2192 m.
Fig. E2.37
80 Engineering Mechanics

Draw OG perpendicular to the plane ABC. By virtue of symmetry, G must be the


centroid of the base triangle ABC. If C and G are joined and extended to meet AB
at D, CD will be the median.
Now, CD = ACsin 60∞ and CG = 2 CD = 2 AC sin 60∞
3 3
2 ¥ 1.2192sin 60∞
From ∆COG, we obtain, sin a = CG = 3 = 0.7698
OC 0.9144
So, a 50.336°

If S be the load taken by each chain, by considering vertical force equilibrium, we


can obtain
3S cos a 45 or S = 45 = 23.5kN
3cos50.336∞
Example 2.38 Three identical wooden planks AN, CM, and BP are arranged in a
horizontal plane as shown in Fig. E2.38(a). Each plank is suitably supported and
ABC forms an equilateral triangle. External load acts midway of AB. Find all support
reactions if each of the plank is 3.2 m long.

Fig. E2.38

Solution Here, AM AC AB BN BC CP 3.2/2 1.6 m and DB AB/2


0.8 m.
Consider FBD of AN and take moment equilibrium at N and vertical force equilib-
rium [Fig. E2.38(b)],

ÂM N
†
0: RB × BN + 2100 × DN RA × AN
Hence, 1.6RB + 2.4 × 2100 3.2RA
Equilibrium of Forces 81

or 2RA – RB 3150 (1)



≠ ÂF y 0: RA + RN 2100 + RB (2)

Consider FBD of CM [Fig. E2.38(c)]. Consider moment equilibrium at M and verti-


cal force equilibrium.

ÂM M
†
0: RA × AM RC × CM
Hence, RA 2RC (3)

≠ ÂF y
0: RM + RC RA 2 RC

So, RM RC (4)
Take FBD of BP [Fig. E2.38(d)]. Consider moment equilibrium at P and vertical
force equilibrium,

ÂM P
†
0: RC × CP RB × BP

Thus RC 2RB (5)



≠ ÂF y 0: RB + RP RC 2RB

Hence, RP RB (6)
Solving Eqs (1), (3), and (5), we obtain, RA 1800 N, RB 450 N, and RC 900 N. Again
from Eqs (2), (4), and (6), we obtain, RN 750 N, RM 900 N, and RP 450 N.
Example 2.39 A horizontal platform PQ carries a block of mass 732 kg, as shown
in Fig. E2.39(a). In the equilibrium condition of the whole system, a force F is to be
applied at M. Determine this equilibriant force.

Fig. E2.39
82  Engineering Mechanics

Solution  Consider FBD of the platform [Fig. E2.39(b)]. From the equilibrium of
vertical forces, we can say, FN = 732g.
Now use FBD of the balancing rod and consider moment equilibrium at O [Fig.
E2.39(c)]:
FN × ON = F × OM
Putting the values, we get
732 × g × 0.2 = F × 3  or  F = 478.58 N
Example 2.40  Determine the reaction at A and the axial force in the bar CB of
the crane, shown in Fig. E2.40(a). Neglect self-weight of the crane and assume ideal
hinges at A, B, and C.
Solution  From geometry [Fig. E2.40(b)],

tan q = 1.9 = 1  or  q = 45°


1.9
1.9 m 1.9 m
F
E G

250 N 150 N

D H

1.9 m
A B 

1.9 m
(a)
Fig. E2.40
Consider moment equilibrium of the entire frame at A,
250 × 1.9 + 150 × 3.8 = S sin q × 1.9 or S = 777.82 N
Now consider the horizontal and vertical force at equilibrium,
Æ≈

ÂF x = 0: HA = S cos q = 550 N

≠ ÂF y = 0: VA = 250 + 150 – S sin q = –150 N = 150 N downwards

So, reaction at A = 5502 + ( -150) 2 = 570.088 N

Example 2.41  A 75 kg man stands on the middle rung of ladder AB of weight 25 kg,
which is supported on smooth wall and smooth floor. A string OC holds the ladder
in position preventing it from slipping [Fig. E2.41(a)]. Determine the tension in the
string and the reaction at the supports.
Equilibrium of Forces 83

Fig. E2.41
Solution Consider FBD of the ladder [Fig. E2.41(b)] and use force equilibrium
equations and moment equilibrium at O
Æ≈

ÂF x 0: RA S cos 30° (1)



≠ F y 0: RB (75 + 25)g + S sin 30° (2)

ÂM †
O 0: RA × 3 – RB × 1.5 + (75 + 25)g × 1.5
2
0

or S cos 30° × 3 – (75 + 25)g × 1.5 – S sin 30° × 1.5 + (75 + 25)g × 0.75 0
or (2 cos 30° – sin 30°) S 50g or S 397.99 N
From Eqs (1) and (2), RA 344.67 N and RB 1179.69 N

Example 2.42 Two identical rods MP and MN, each of mass m are hinged together
at M and supported by two pegs at B and C, such that each of the bars make an
angle a with the horizontal in equilibrium position. The pegs are in the same level
horizontally and x-distance apart. If the bars are each of length l, express a as a
function of x and l [Fig. E2.42(a)].
Solution In the FBD of bar MN [Fig. E2.42(b)], the peg reaction at C will act normal
to the bar and self-weight will act downwards at D.
From geometry,
x /2
MC and AD l cos a
cos a 2
Taking moment equilibrium at M,
RC × MC mg × AD
Putting the values of MC and AD:
x /2 l
RC × mg × cos a (1)
cos a 2
Now consider FBD of entire frame, and the equations of equilibrium [Fig. E2.42(c)]
84 Engineering Mechanics

Fig. E2.42
Æ≈
are ÂF x 0: RC sin a RB sin a

or RC RB (2)

≠ ÂF y 0: RB cos a + RC cos a 2mg (3)
or 2RC cos a 2mg or mg RC cos a
On substitution in Eq. (1), we obtain,
x /2 l
RC × RC cos a × cos a
cos a 2
x
cos3 a
l
1/ 3
Hence, a cos–1 ÊÁ x ˆ˜
Ël¯

Example 2.43 A rod of weight W is bent in the shape of a semicircle OAB of radius
R and is made hung at point O with a hinge, as shown in Fig. E2.43. A weight Wo is
suspended from point B. Derive an expression for a, the inclination of diameter OCB
with the vertical.
Solution From ∆COE, CE = R tan a . The CG of the semicircular rod OAB lies at

point G, where CG = 2 R .
p
So, GE = CG - CE = 2 R - R tan a
p

From DGEN , GN = GE cos a = R cos a ÊÁ 2 - tan a ˆ˜ and DB = 2 R sin a


Ëp ¯
Equilibrium of Forces  85

2R/

Wo

Fig. E2.43

For attaining equilibrium condition,


ÂM at O

= 0: W ¥ GN = Wo ¥ DB

Ê ˆ
or W ¥ R cos a Á 2 - tan a ˜ = Wo ¥ 2 R sin a
Ëp ¯

or W Ê 2 - tan a ˆ = tan a
Wo ÁË p ˜¯

Ê ˆ
or tan a Á1 + W ˜ = 2W
Ë Wo ¯ pWo

or tan a (W + Wo ) = 2W or  tan a = 2W


p p (W + Wo )

Example 2.44  A prismatic bar of length 2.1 m and weight 365 N rests within a
hemispherical bowl of radius 0.7 m, as shown in Fig. E2.44(a). Determine the angle
a for the position of equilibrium.

Fig. E2.44

Solution  The bar will remain in equilibrium under the action of reactions at M and
N, and self-weight of the bar acting at G [Fig. E2.44(b)]. Hence, these three forces
must be concurrent. From ∆MDN,
MN = MD cos a = 2 × 0.7 cos a
DN = MD sin a = 2 × 0.7 sin a
So, GN = MN – MG = 1.4 cos a – 2.1
2
86 Engineering Mechanics

Now from 'GDN,


GN = 1.4cos a - 1.05
tan a
DN 1.4sin a
or 1.4 sin2 a 1.4 cos2 a – 1.05 cos a
or 1.4 – 1.4 cos2 a 1.4 cos2 a – 1.05 cos a
or cos2 a – 0.375 cos a – 0.5 0 or cos a 0.919
Hence, a 23.21°
Example 2.45 A heavy metal rod of weight 73 kg and length l, is supported at one
end N by a guy wire NP and rests at M vertically below P. If the length of wire is a
and the wall is smooth, express the distance PM as a function of a and l, when the
system will attain equilibrium [Fig. E2.45(a)].
Solution For attaining equilibrium RM, S, and 73g weight must be concurrent at D
[Fig. E2.45(b)]. Here DNGD and DNMP are similar and MG GN. Therefore,
DG PM = x and PD a
2 2 2

Fig. E2.45
From 'MGD,
DG = x /2 x
sin a
MG l /2 l
1/ 2
Ê x2 ˆ l 2 - x2
Therefore, cos a ÁË1 - l 2 ˜¯ =
l2
1/ 2
l Êl - x ˆ
2 2
Again, MD MG cos a Á ˜
2Ë l 2
¯
Now from DPMD, PM2 PD2 – MD2
Putting the values, we get

l2 Ê l - x ˆ
2 2 2
Ê aˆ a2 - l 2 + x2
x2 ÁË 2 ˜¯ - 4 Á ˜ 4 4 4
Ë l2 ¯

or 3x2 a2 - l 2 or x ( a 2 - l 2 )/3
4 4 4
Equilibrium of Forces  87

Example 2.46  Two balls of weight 100 kg and 150 kg having radius 10 cm and 15 cm,
respectively, are placed one above another inside a hollow circular cylinder of radius
18 cm, open at both ends, as shown in Fig. E2.46(a). Neglecting the effect of friction,
determine the self-weight of cylinder so that it will not tip over.
Solution  Consider FBD of balls [Fig. E2.46(b)],
Æ≈
ÂF x = 0: R1 = R2 (1)

Take moment equilibrium at C2,


100g × x = R1 × y (2)
Now consider FBD of the cylinder [Fig. E2.46(c)].
At the instant of overturning, the cylinder will lose contact with floor at point N and
only reaction at M will be effective. In this condition, we can say from Eqs (1) and (2)
that R1 and R2 will form a couple. Consider the moment equilibrium at M,
18 × W = R1 × y
Hence, 18W = 100g × x (3)

Fig. E2.46
Now, from geometry,
r1 + x + r2 = 18 + 18  or  10 + x + 15 = 36
or x = 11 cm
Substituting in Eq. (3), we obtain
18W = 100g × 11 or W = 599.317 N
88 Engineering Mechanics

Example 2.47 (N + 1) number of bricks having the same size are piled one above
another in a vertical plane so that they rest, each one overhanging the one below by
as much as possible. Prove that if 2a is the length of each brick, the lowest but one
overhangs the lowest by a length ÊÁ a ˆ˜ . Also show that if each brick overhangs the
ËN¯
one next below by a length a , the largest number of bricks that can be piled up is (2N – 1).
N

W (I)
B1
A1
a
a W
(II)
B2
A2
W (III)
x (a – x) B3
A3
W (IV)
y (a – y)
A4
(a)
Fig. E2.47
Solution Let us consider the weight of each brick W. Arrangement of four bricks is
shown in Fig. E2.47(a) as a representative one. The bottom-left points of the bricks
successively from top to bottom are designated as A1, A2, A3, and A4. Topmost contact
points between bricks (I) and (II) is B1, (II) and (III) is B2, and (III) and (IV) is B3.
If the brick (I) is to stay over brick (II) with maximum projection, then the maximum
horizontal distance between A1 and B1 will be a. So the CG of brick (I) will be along line
B1A2. Similarly the CG of bricks (I) and (II) combined will be along line B2A3. Let us assume
the projection for brick (II), A2B2  x. Considering moment about B2, we obtain
W(a – x)  Wx or x = a .
2
Again the CG of bricks (I), (II), and (III) combined will be along line B3A4. Let us
consider the projection for brick (III), A3B3  y. Considering moment about B3, we
obtain
2Wy  W(a – y) or y = a .
3
Proceeding in the identical manner, we can thus infer that N-th brick of (N + 1)
Ê ˆ
numbered brick pile will have a maximum projection value Á a ˜ .
ËN¯
For the second case, let us consider the largest number of bricks is k. Now, the distance
of CG of lowest but one brick from the edge of the lowest brick = a - a .
N
The distance CG of the second lowest brick from the edge of the lowest brick = a - 2a .
N
Thus the distance of CG of the topmost brick from the edge of the lowest brick = a - ka .
N
Hence, W ÊÁ a - a ˆ˜ + W ÊÁ a - 2 a ˆ˜ + ............ + W ÊÁ a - ka ˆ˜ = 0
Ë N¯ Ë N¯ Ë N¯
Equilibrium of Forces 89

Ê 1ˆ Ê 2ˆ Ê kˆ
or ÁË1 - N ˜¯ + ÁË1 - N ˜¯ + ......... + ÁË1 - N ˜¯ = 0

or (1 + 1 + 1 + .....) = 1 (1 + 2 + ...... + k )
N
k ( k + 1)
or k= 1 or k = (2 N - 1)
N 2
Example 2.48 A block of mass 180 kg is suspended from the end of a 20 cm long
lever NM, as shown in Fig. E2.48(a). The spring becomes unstretched when the lever
moves in vertical position. Determine the angle a for equilibrium of the system. Take
spring constant 40 N/mm.

Fig. E2.48

Solution Let us assume that at an angle a, the system will maintain equilibrium. Due
to this angular displacement, linear displacement of spring will be S 75a mm. So
the tension induced in the spring will be F (40 × 75a) N. Considering FBD of the
system [Fig. E2.48(b)] and taking moment equilibrium at O, we have
F × 75 180g × 220 sin a
40 × 75a × 75 180g × 220 sin a or sin a 0.57936a
By trial and error method, the value of a will be determined.
Example 2.49 A model aeroplane is tested in a wind tunnel. Under test condition,
lift force and drag force are 12.5 N and 18 N, respectively. If the auto recorder at C
records a moment of 34.5 N m [Fig. E2.49(a)], determine the pitching moment MP.
90  Engineering Mechanics

Fig. E2.49

Solution  Consider FBD of bracket GBC [Fig. E2.49(b)], where lift force is made
equivalent by applying a force and moment. Take moment equilibrium at C
MP + (12.5 × 0.6) + (18 × 0.33) = 34.5
Therefore, MP = 21.06 N m
Example 2.50  The arrangement shown in Fig. E2.50(a) is a lever attached with two
rollers, used for compressing bonding of lamination. The rollers are of radius 22 mm.
If 100 N force is applied to the end of the lever, how much force will be exerted on
the top and bottom surfaces of the laminates.
P P
mm 100 N
150
100 N N
N 150° cos 45°
45°
RN
35 mm RM (35 + 22 + 22 mm)
45°
M M
(a) (b)FBD of lever MNP
Fig. E2.50
Solution  Consider FBD of lever MNP [Fig. E2.50(b)]. Take moment equilibrium
at N,
100 × 150 cos 45° = RM × (35 + 22 + 22)
Hence, RM = 134.26 N
Take moment equilibrium at M,
RN × (35 + 22 + 22) = 100 × (35 + 22 + 22 + 150 cos 45°)
or RN = 234.26 N

Example 2.51  A moment of 24 N m is required to turn the bolt about the axis
[Fig. E2.51(a)]. Determine the force F. If the wrench fits easily on the bolt, find the
reactions at two corners P and Q of the bolt.
Equilibrium of Forces 91

Fig. E2.51

Solution If the moment of 24 N m is applied at C [Fig. E2.51(b)], we can say


110 × F 24 × 1000
or F 218.19 N
Now from geometry of the hexagon of height 16 mm [Fig. E2.51(c)], the side length
of hexagon can be calculated as,
x sin 60° 8 or x 9.24 mm
Now from Fig. E2.51(d), take moment equilibrium at P
Ê 9.24 ˆ
9.24 × RQ ÁË110 - 2 ˜¯ × 218.19

or RQ 2488.405 N
Again take moment equilibrium at Q,
Ê 9.24 ˆ
9.24 × RP ÁË110 + 2 ˜¯ × 218.19

or RP 2706.595 N

Example 2.52 A lawn mower shown in Fig. E2.52(a) is pushed at handle A by a force
F and it moves with uniform speed. The mass of the machine with attached grass bag
is 50 kg. If q 15°, determine the reaction force under each pair of wheels. Compare
the result with reaction forces when q 0 P. Neglect frictional effect.

Solution Taking equilibrium of forces along the incline [Fig. E2.52(b)], we can write,
F mg sin q.
Now, consider moment equilibrium equation about point B,
F × 0.9 + mg cos q × 0.2 – mg sin q × 0.215 – RC × 0.7 0

Hence, RC 1 [mg sin q × 0.9 – mg sin q × 0.215 + mg cos q × 0.2]


0.7
mg
(sin q × 0.685 + cos q × 0.2) 259.52 N
0.7
Again, consider force equilibrium equation normal to the incline
RB + RC mg cos q
or RB mg cos q – RC 214.12 N
92  Engineering Mechanics

s
Grags
ba

Fig. E2.52
Example 2.53  Determine the reaction at supports for the bracket shown in
Fig. E2.53(a). Neglect self-weight.
Solution  From the FBD of the idealized bracket [Fig. E2.53(b)], consider horizontal
force of equilibrium, RAH = 250 N
Consider moment equilibrium at A,
RB × 5 = 750 + 750 or RB = 300 N
Using the equation of vertical force equilibrium,
RAV = –RB = –300 N

Fig. E2.53
Example 2.54  Blocks A and B have masses 400 kg and 200 kg, respectively and rest
on 27° incline. Blocks are attached to a post by cords and the post is held fixed by
action of force F [Fig. E2.54(a)]. Assuming all contact surfaces smooth and cords
parallel to incline, determine the value of F.
Equilibrium of Forces 93

Fig. E2.54

Solution Consider FBD of block B [Fig. E2.54(b)] and from the force equilibrium
equation along specified x-and y-directions,
SB 200g sin 27° and RB 200g cos 27°
Now analysing FBD of block A [Fig. E2.54(c)],
SA 400g sin 27°
and RA RB + 400g cos 27°
600g cos 27° [Substituting the value of RB]
Finally, consider FBD of post [Fig. E2.54(d)] and take moment equilibrium at O
SA × 0.18 + SB × (0.16 + 0.18) F × (0.38 + 0.16 + 0.18)
400g sin 27° × 0.18 + 200g sin 27° × 0.34 F × 0.72
or F 865.72 N
Example 2.55 The rigid beam ABD is supported and loaded as shown in
Fig. E2.55(a). If the spring constant is 20 N/mm for both the springs, determine the
reaction at A and force in each spring.
Solution The beam is assumed to get deflected as shown in Fig. E2.55(b). If the angle
of deflection be q, CC1 10q and DD1 16q. Now, the uniformly distributed load is
assumed to act at its centroid which is 5 m away from D. Here force F (20g × 10 × 100) N.
94 Engineering Mechanics

Fig. E2.55

The restoring forces at springs C and D will be RC (20 × 103 × 10q) N and RD
(20 × 103 × 16q ) N. Consider the equation of vertical equilibrium,
RAV + RC + RD F
or RAV + 20q × 104 + 32q × 104 2g × 104 (1)
Consider moment equilibrium at A,
RC × 10 + RD × 16 F × 11
or 20q × 105 + 32q × 104 × 16 2g × 104 × 11 or 712q 215.754
or q 0.303025 rad
Hence, RC 20q × 104 60.605 kN and RD 32q × 104 96.968 kN.
Substituting in Eq. (1), RAV 38.567 kN. From observation it is obvious, RAH 0.

EXERCISES

Multiple Choice Questions


Choose the correct or most appropriate option(s).
2.1 Which of the following conditions is/are valid for the equilibrium of a rigid
body subjected to three coplanar forces?
(a) all forces are parallel (b) all forces are concurrent
(c) at least two forces are concurrent (d) all of these
2.2 Reactive component(s) of a roller support on a horizontal plane
(a) only vertical force (b) only horizontal force
(c) both vertical and horizontal force (d) none of these
2.3 Reactive component(s) of a hinge joint supported on a horizontal plane
(a) only vertical force (b) only horizontal force
(c) both vertical and horizontal force (d) none of these
Equilibrium of Forces 95

2.4 Reactive component(s) of a fixed support


(a) only vertical force and fixity moment
(b) only horizontal force and fixity moment
(c) both vertical and horizontal force and fixity moment
(d) none of these
2.5 A coplanar force and a coplanar couple acting on a rigid body
(a) balance each other (b) cannot balance each other
(c) produce moment of a couple (d) none of these
2.6 A transversely loaded beam will be unstable, if the end supports are
(a) one fixed, other hinge (b) one fixed, other roller
(c) one roller, other hinge (d) both roller
2.7 Self-weight of a block resting on an incline will act
(a) vertically downwards (b) horizontally
(c) along the incline (d) normal to the incline
2.8 For two like or unlike parallel unequal forces, the ratio of magnitude of two
forces is
(a) the ratio of the normal distances of their line of action from the line of
action of their resultant force
(b) zero
(c) the reciprocal of the ratio of the normal distances of their line of action
from the line of action of their resultant force
(d) none of these
2.9 For two unlike equal parallel forces, there exists
(a) a resultant force (b) a resultant moment
(c) a resultant force and a moment (d) none of these
2.10 When trying to turn a key into a lock, the following is applied
(a) coplanar force (b) non-coplanar force
(c) lever (d) couple

Review Questions
2.1 State and prove Lami’s theorem.
2.2 State and prove the theorem of Varignon.
2.3 Using the theorem of Varignon, derive the expression for resultant of (i) two
like parallel forces and (ii) two unlike parallel forces.
2.4 Explain parallel shifting of a force.
2.5 Starting from fundamentals, derive the expression of a wrench.
2.6 Explain (i) positive wrench, (ii) negative wrench, and (iii) null wrench.
2.7 Discuss the various types of supports and their characteristics.
2.8 What is a free-body diagram? Discuss the procedure for drawing free-body
diagrams.

Numerical Problems
2.1 A vertical force F of 50 N is applied to the bell crank at point B as shown in
Fig. P2.1. A force P is applied at A to prevent rotation of the crank about point
O. Compute the force P and bearing reaction R at O.
96 Engineering Mechanics

Fig. P2.1
2.2 Neglecting thickness and mass of the beam, determine the support reactions
on the beam loaded as shown in Fig. P2.2.

Fig. P2.2
2.3 To turn left, the steering wheel of a Zen Estillo is to rotate by applying a force
of 10 N. If the diameter of the wheel is 0.4 m, compute the required moment.
2.4 A dam is subjected to 50 kN force on u/s vertical face and 40 kN on d/s inclined
face [Fig. P2.4]. If the self-weight of the dam is 140 kN, assuming all forces
coplanar, determine a single equivalent force and locate its point of intersection
with the base.

Fig. P2.4
2.5 A 2 m × 4 m plate is subjected to a system of two coplanar forces as shown in
Fig. P2.5. Determine the equivalent action at centroid of the plate that may
replace the force system.

Fig. P2.5

2.6 A triangular lamina ABC having obtuse angle at C rests on side AC in con-
tact with a table, as shown in Fig. P2.6. If M is the mass of the lamina, prove
that the least weight P suspended from B causing overturning of lamina is
Equilibrium of Forces 97

Mg Ê a 2 + 3b 2 - c 2 ˆ
, where a, b, c are lengths of the sides of the triangle. What
3 ÁË c 2 - a 2 - b 2 ˜¯
would happen for c2 > a2 + 3b2? [FBD is drawn as hint]
B

B
P

c
P
a Mg
G

D C F E bA
C A
(a) (b)
Fig. P2.6
2.7 ABCD is a rectangle such that AB CD a and BC DA b (Fig. P2.7).
Force P acts along AD and CB, and force Q acts along AB and CD. Prove that
the perpendicular distance between the resultant of the forces P and Q at A, and
Pa - Qb
resultant of the forces P and Q at C is .
P2 + Q2

A q B
Re
b
P P
d

D q C
a
Fig. P2.7
3
ac
YI
c
n
a
I
0
Truss, Frames, and
Cables

Key Concepts
0 Definition of truss, its components, and basic assumptions for truss analysis
0 Determinacy, stability, and redundancy
0 Methods of analysis (joint, section, and hybrid) of truss structure
0 Frame structure and its method of analysis
0 Analysis of suspension cables subjected to point loading
0 Analysis of suspension cables subjected to uniform loading (both parabolic
and catenary)

3.1 INTRODUCTION
Italian architect Andrea Palladio (1508–1580) is thought to have first used
modern trusses, although his design basis is not known. He may have revived
some old Roman designs and probably sized the truss component by some
rules of thumb (perhaps old Roman rules). Palladio’s extensive writing in
architecture included detailed descriptions and drawings of wooden trusses
quite similar to those used today. After his time, trusses were forgotten for 200
years, until they were reintroduced by Swiss designer Ulric Grubermann.

3.2 TRUSS
A truss is a load-bearing structure formed by a group of members arranged
in the shape of one or more triangles. The members are assumed to be con-
nected with frictionless pins; the triangle is the only stable shape. Figure 3.1
shows that it is impossible for a triangle to change the shape under load, except
through the deformation of the members unless one or more of the sides are
bent or broken. It also shows that any structure or its sub-elements, compris-
ing four or more sides are not stable and may collapse under load. Therefore,
the sub-element of an ideal truss will necessarily be a triangle.
Truss, Frames, and Cables 99

A A′

F A F A A′ B B′ F B
B′
E E′

B C D C D C

(a) (b) (c)


Fig. 3.1
Thus an idealized truss can be defined as an assemblage of component
load-bearing parts, which are all straight and two force bodies.
For the analysis of truss, the term joint invariably comes into picture. The
joint of a structure is defined as a connection where two or more members
are fastened together.

3.3 ELEMENTS OF A TRUSS


A simple truss is constructed of different elements, as shown in Fig. 3.2.
Chords These are members which form the outline of the truss. The members
like CD, LK, and EF are chords.
Verticals These are vertical members of the truss, like CK, DJ, FH, etc.
Diagonals These are inclined members inside the truss, like BK, EJ, etc.
End posts These are members at the ends of the truss, may be vertical or
inclined. Here AB and FG are the end posts.

Fig. 3.2

3.4 TYPES OF TRUSS


Considering the basic characteristics of the structure along with the geometry,
trusses can be categorized into the following types:
Simple truss It is a rigid plane truss which can be formed from a basic
triangular truss by appropriately repeating the addition of two new members
and a one new joint at a time. The sequential illustration is provided in Fig.
3.3. In Fig. 3.3(b), new members BD and CD are fastened to existing joints B
and C, respectively, whereas the joint C (not lying on member BC) connects
members BD and CD. It is further developed by adding joint E and members
AE and DE to obtain Fig. 3.3(c).
100 Engineering Mechanics

B
B D
B
D

A C A C A C

(a) (b) (c)


Fig. 3.3 Simple truss and its development

Compound truss It is a rigid plane truss which is composed of two or more


simple trusses directly connected together by either linking joints or linking
members, or both, but cannot be formed with the process of building a simple
truss. It is illustrated in Fig. 3.4, where the linking members and linking joints
are symbolized by Ml and Jl, respectively.
Ml F
F F

JL Ml
JL
Ml
Ml

JL
(a) (b) (c)
Fig. 3.4 Different types of compound truss formation
Complex truss The characteristics of these trusses are such that they do not
fall within the category of either simple truss or compound truss. These are
shown in Fig. 3.5
F

(a) (b)
Fig. 3.5 Different types of complex truss

3.5 ASSUMPTIONS FOR TRUSS ANALYSIS


To analyse a multi-member truss, properly loaded and supported, following
assumptions are made:
Truss members are connected with frictionless pins. In reality, pin connections
are used for very few trusses, and no pins are frictionless. A heavy bolted or
welded joint is very different from a frictionless pin.
Truss members are straight. If they were not straight, the axial forces would
cause them to have bending moments.
Truss, Frames, and Cables 101

The displacement of the truss is small. The applied loads cause the members
to change length, which in turn causes the truss to deform. The deformations
of a truss are not of sufficient magnitude to cause appreciable changes in the
overall shape and dimensions of the truss. Special consideration may have to
be given to some very long and flexible trusses.
Loads are applied only at the joints. Members of the truss are so arranged
that the loads and reactions are applied only at the truss joints.
With these foregoing assumptions, we obtain an ideal truss, whose members
have only axial forces. Following the illustration in Fig. 3.6, we can say that
a member with only axial force is subjected to axial tension or compression
and that no bending is present.

Fig. 3.6

3.6 DETERMINACY, STABILITY, AND REDUNDANCY


The terms ‘determinacy’, ‘stability’, and ‘redundancy’ have a very important
significance in the analysis of any structure, whether a truss, frame, or anything.
In case of a truss, if we can compute all the unknown forces in the members
as well as the reaction forces from the available equations of equilibrium, then
we can define the truss as a statically determinate or simply a determinate truss.
If m is the number of members in a plane truss and j is the number of joints,
then we can write the necessary condition for statical determinacy as m + r
2j, where r is the number of reactive components. Generally a plane truss is
supported by one hinge bearing and one roller bearing. So the total number
of unknown reactive components is three, one for roller and two for hinge.
Now for a plane two-dimensional truss we can write the condition of statical
determinacy as m + 3 2j. Sometimes a determinate truss is also called just-
rigid truss. If in a structure, the number of unknown forces of member is higher
than the number of available equations of equilibrium, then we cannot solve
for the unknown forces. In that case we will designate that structure as statically
indeterminate internally. Hence, the condition for statical indeterminacy for
a plane truss is m + 3 > 2j, and the degree of internal indeterminacy is [(m +
3) – 2j]. Actually in this case the number of members present is more than the
number required to achieve just-rigid condition. A statically internally inde-
terminate structure is hence sometimes called over-rigid truss. Indeterminacy
may occur externally also. In that case the number of components of support
reactions will be more (more than three, in case of plane truss) and the number
of members may be just the required or more than the required. The condition
for statical indeterminacy externally is r – 3. Statical indeterminacy may occur
internally and externally simultaneously. In Fig. 3.7(e), the total degree of
determinacy, as governed by m + r – 2j is 4. Out of that, the degree of external
102 Engineering Mechanics

indeterminacy is found as r – 3 5 – 3 2. Hence the degree of internal inde-


terminacy total determinacy – external determinacy 4 – 2 2. All the de-
terminate and indeterminate structures are stable. But if it is found that a plane
truss is satisfying the relation m + 3 < 2j, then we can say that surely some
members are missing to form triangular sub-element of the structure. So that
truss will naturally be an unstable truss or under-rigid truss. All these probable
cases are illustrated in Fig. 3.7.

Fig. 3.7
Redundancy generally refers to the degree of indeterminacy. Referring to
Fig. 3.7(c) and (d), the degree of indeterminacy is 2, hence the redundancy is
2, but in the first case redundancy occurs in member force while in the second
it occurs in support reaction. So just like indeterminacy, redundancy is either
internal or external or both, as shown in Fig. 3.7(e). Redundant members are
those members whose removal in no way affects the determinacy and stability
of a just-rigid simple truss. Similarly removal of redundant support does not
affect the determinacy and stability of a just-rigid simple truss. Redundant
members and supports are evoked purposefully adjudicating the sensitiveness
and applicability of the structure.

3.7 METHODS OF ANALYSIS OF A TRUSS


The basic approach for analysing a properly supported and loaded truss is
to separate it into pieces. We can either separate a single joint of the truss or
segregate the entire truss into one or more pieces. Our next job is to construct
free-body diagrams (FBD) of the joints or segments. This is illustrated in
Fig. 3.8.
Truss, Frames, and Cables 103

Fig. 3.8

From these free-body diagrams, we can determine the forces of the members
of the truss. Depending on the type of separation, the method of analysis can
be broadly classified into two types.

3.7.1 Method of Joints


We can follow a systematic approach to determine the axial forces of the
members.
(i) Find the components of support reactions. Of course, we may encounter
some cases where computation of unknown reactive components is not
essential.
(ii) Select the joint where the maximum number of unknown forces is two.
(iii) Construct the FBD of the joint.
(iv) Compute the unknown axial forces of the joint considering equilibrium
equations of forces along two mutually perpendicular directions. Univer-
sally the forces in all the members are considered tensile. If after analysis
the value of any force comes out negative, the nature of the force will be
considered as compressive.
(v) Proceed successively to other joints having two unknowns to solve the
unknown forces of members until all members are analysed.
In Fig. 3.9, first we have to determine the components of reactions at A and
D. Now select joint D having two unknown member forces and one known
component of reaction. Solve the unknown forces in members CD and DE.
Now proceed to joint C. Here the force in member CD is already known, and
the remaining unknown forces are two, in members BC and CE. Follow the
same procedure to solve the forces in BC and CE. After that proceed to joint
B. Then iterate the process for joint E.
From meticulous observation of the joints, we can reduce the labour of
computation to some extent. In Fig. 3.10, we can say that in each case, the
force in the member BD is zero and SAB SBC. Again in Fig. 3.11, SAB SBC
and SBD SBE in all the cases. In Fig. 3.12, the joint remains in equilibrium
by the action of two members AB and BC. But from the concept of static
equilibrium, we know that the present case is impossible to happen until AB
and BC are collinear. Hence SAB 0 SBC.
104 Engineering Mechanics

Fig. 3.9 Fig. 3.10

Fig. 3.11 Fig. 3.12

3.7.2 Method of Sections (Ritter’s Method)


We can analyse a truss structure through the method of sections as per the
procedure given below.
(i) Identify the location of the section which passes through all or maximum
number of members under consideration. It is generally desirable that
after separation there should be not more than three unknown member
forces. We have shown this in Fig. 3.13 where the section is intersecting
three members with unknown forces. There are of course some special
cases where we can tackle the section passing through more than three
members.

Fig. 3.13
In Fig. 3.14, the section intersects four members. It may happen that
one section is not sufficient for analysis. Referring to Fig. 3.15, to de-
termine the forces in the members BD, BE, and DE, one section is not
sufficient. Hence we have taken two sections. Finally we can conclude
that selection of section is entirely case-specific.
Truss, Frames, and Cables 105

Fig. 3.14 Fig. 3.15


(ii) Now find the reactions at support(s). Knowing the reactive components
at support may not be always essential. It is purely dependent on choice
of section.
(iii) Select either top or bottom part of the section or left or right part of the
section or both the parts of the section.
(iv) Draw the FBD of the selected portion of the section.
(v) To determine the unknown forces, analyse the segment(s) using either
the equation(s) of force equilibrium or moment equilibrium or both. In
method of sections, all the unknown forces are assumed tensile. After
analysis, if the member forces are found to be negative, the assigned
nature of those forces will get reversed, that is, will be of compressive
nature.

3.7.3 Hybrid Method


It is not actually a separate method but a combination of the method of joints
and method of sections. In this approach follow the guidelines given below
to solve the unknown forces of the members.
(i) First identify the location of the desired section. If you find that analysing
the FBD of both sides or either sides of the section, all the unknown
forces cannot be computed, in that case before cutting the truss into
segments and analysing, you have to select the desired joint(s) to solve
some of the unknown forces.
(ii) Now proceed following the method of sections to compute the unknown
forces of members.
In Fig. 3.16, the choice of sections is shown by chain-dotted section lines.
But you will notice that analysis of either top part of section or bottom part
of section or of both will not yield all the unknown forces. To tackle this, first
analyse the equilibrium of joints H and G. Then proceed with the equilibrium
of the bottom part of the section and find the desired unknowns.
106 Engineering Mechanics

Fig. 3.16

3.8 FRAMES
A frame is a load-bearing structure comprising several interconnected rigid
bodies (members) and forming a definite geometrical shape, where at least one
member will be a multi-force member. It means unlike members of a truss,
external loading can be applied not only at joints but at anywhere along the
length of the members of a frame. A frame is designed to support applied
loads and is usually fixed in position. Figure 3.17 shows some representative
frame structures.

Fig. 3.17

3.9 METHOD OF ANALYSIS OF A FRAME


Though there is no fixed rule for analysing loaded frames, the below procedure
can be followed:
(i) Dissociate the entire frame from its supports to determine the support
reactions. Normally determination of the support reactions is neces-
sary to proceed further. But there are some special cases where you may
proceed without computing the support reactions.
(ii) Next step is disassembling of members. Depending on the nature of
problem, you have to disassemble each and every member of the frame
and analyse its equilibrium. Sometimes you may find that disassembling
only a few members yields the desired result. In some cases partial disas-
sembling of the entire frame will be beneficial to find the unknowns. So
the process of disassembling is entirely case-specific.
Truss, Frames, and Cables  107

3.10  Suspension Cables


Suspension cables are special structures which are used in suspension bridges,
power transmission lines, aerial trolley movement paths, etc. Following are
the assumptions for analysis of loaded suspension cables:
(i) The cable is idealized as of uniform cross-section and perfectly flexible.
(ii) Bending resistance offered by the cable is negligibly small.
(iii) Internal force induced in the cable is always along the direction of
cable.
(iv) If the cable is subjected to concentrated loads, the self-weight of the
cable can be neglected.
Suspension cables may be subjected to either concentrated load or distrib-
uted load.

3.11 Elements of Suspension Cables


The general parameters associated with the structure of suspension cables
have been illustrated below with help of Fig. 3.18(a) and (b).
Span  This is the horizontal distance between two end supports holding the
cable. If the supports for a cable lie at different elevations, the inclined line
joining these two end points is
called inclined span.
Length  This is the actual length
of the cable between two end
supports.
Sag  It is the vertical distance of
any particular point of the cable
from the imaginary horizontal
(a)(a) line joining two supports.
  If the end supports stay at
span
different levels, sag of a particular
ined
l
Inc B point can be measured from
an imaginary horizontal line
dBC
UC
from any of the supports, and
A the reference support should
dAC be mentioned. In Fig. 3.18(b),
C when the sag of point C is
(b)
(b)
measured with respect to the
horizontal through support A,
Fig. 3.18
it is designated as dAC, and when
it is with respect to support B, it is dBC. Sometimes sag is also termed as dip.
Tension  It is the axial force induced in the cable. The magnitude of tension
may vary from point to point within the cable. For cables subjected to uniform
loading, the direction of tension at any point in the cable is determined by the
outward tangent drawn at that point.
108 Engineering Mechanics

3.12 SUSPENSION CABLES SUBJECTED TO POINT LOADING


When the suspension cables are subjected to only point loading, the analysis
becomes very easy. Conventionally the self-weight of the cable is not taken
into account. For example, if we want to find the axial tension in the cable as
shown in Fig. 3.18(a), we can simply adopt the principle of method of joints
for a truss. It is illustrated in Fig. 3.19. To solve joint B, we have two unknown
forces, namely SBA and SBC and the known force of 1 kN. Considering force
equilibrium equations, we can easily obtain the magnitudes of SBA and SBC.
Then shift to point C, where there is only one unknown force SCD. After solv-
ing point C, we can finally reach point D, where the remaining unknown is
SDE. Thus by means of successive selection of loading points, we can easily
determine the tension in cables subjected to concentrated loads only.
SBC SCD SDE
SBA
B C
D

SBC

SCD
1 kN 2 kN 3 kN
(a) (b) (c)
Fig. 3.19

3.13 SUSPENSION CABLES SUBJECTED TO UNIFORMLY


DISTRIBUTED LOADING
The uniformly distributed loading acting on a suspension cable may be of two
types. When the self-weight of the cable is only taken into account, loading
is considered uniformly distributed along the length of the cable. Sometimes
the cable is loaded with the help of hangers. In that situation, the loading is
considered to be distributed along the horizontal span of the cable. In the
first case, the curve in equilibrium condition takes the shape following the
equation of a catenary and hence is termed as catenary cable. In the second
case, it follows the path of a parabola and hence is termed as parabolic cable.
For a generalized analysis, refer to Fig. 3.20(a), where A and B are the end
supports, O is the lowest point through which the reference coordinate axes
are assumed, C is an arbitrarily chosen point on the cable having coordinates
(x, y), and W is the resultant of uniformly distributed loading between points
O and C. The FBD of the segment OC is shown in Fig. 3.20(b), where T is the
tension at point C and H at point O. For attaining equilibrium condition, W,
H, and T must be concurrent. The slope at point C can be computed as
dy W
= (3.1)
dx H
This is the governing differential equation of equilibrium of a cable subjected
to uniformly distributed loading, and can be suitably applied for cable of any
shape in equilibrium. The tension induced in the cable can be computed as
T = W2 + H2
Truss, Frames, and Cables 109

T
y
B
C

A
C (x, y) x
H
O

x
O W
(a) (b)

Fig. 3.20
3.13.1 Catenary Cables y

In this case the cable is subjected to self- B (xB,yB)


weight only and thus freely hangs under (xA,yA)
gravity field. If for the self-weight, the A h2
S
loading intensity per unit length of the C (x, y)
h1
cable is w, referring to Fig. 3.21, the O
total load acting for the OC span is ws. a b
Following the governing differential l
equation, as derived earlier, we obtain Fig. 3.21
dy ws
= (3.2)
dx H
Again from geometry, we can write
2 È 2˘
Ê dy ˆ
ds = dy2 + dx 2 = dx 1 + Á ˜ = Í 1 + ÊÁ ws ˆ˜ ˙ dx
Ë dx ¯ Í ËH¯ ˙
Î ˚
Inverting the above equation and integrating within limits,
-1
x È
s 2˘

Ú dx = Í 1 + ÊÁ ws ˆ˜ ˙ ds or
Ú x = H sinh -1 ws
Í ËH¯ ˙ w H
0 0Î ˚
or H
s = sinh wx (3.3)
w H
Substituting in Eq. (3.2), we obtain

dy = sinh wx dx
H
Integrating the avove within limits, we obtain
y x

Ú dy = Ú sinh wx
0 0
H
dx

or y = H ÊÁ cosh wx - 1ˆ˜ (3.4)


wË H ¯
110 Engineering Mechanics

For simplicity of analysis, let us introduce the constant, c = H . Thus the


above equation yields w

y = c ÊÁ cosh x - 1ˆ˜ (3.5)


Ë c ¯
Thus the shape of a suspended cable subjected to only its self-weight can be
defined by the equation of a vertical hyperbolic cosine curve shifted a dis-
tance c downwards and is commonly called a catenary. If reference x-axis is
assumed c distance below the lowest point of the cable, its equation will then
get modified to
y = c cosh x (3.6)
c
dy ws s
Again substituting = = , we obtain another form of the above
dx H c
equation,

s = c sinh x (3.7)
c
Now, x = sinh -1 Ê s ˆ = sinh -1 Ê dy ˆ = sinh -1 (tan q ) = ln(tan q + sec q )
c ÁË c ˜¯ ÁË dx ˜¯

Thus, x = c ln (tan q + sec q ) (3.8)


Now total downward loading in OC span
= ws = H sinh wx
H
Hence, tension at any point on the curve
2
T= Ê H sinh wx ˆ + H 2 = H cosh wx = H + wy (3.9)
ÁË H ˜¯ H
Other form of tension is:
2
T= ( ws )2 + H 2 = Ê H sinh x ˆ + H 2 = H cosh s (3.10)
ÁË c ˜¯ c
So, the tension at both support ends A and B are
TA H + wh1 and TB H + wh2 (3.11)
If we can determine the value of H, the magnitude of tension could easily be
found out. Using Eqs (3.10) and (3.11), we thus obtain
H Ê cosh wb - 1ˆ
h1 = H ÊÁ cosh wa - 1ˆ˜ and h2 =
wË H ¯ w ÁË H ˜¯

On slight reorientation of the above equations, we obtain


Ê wh ˆ Ê wh ˆ
a = H cosh -1 Á 1 + 1˜ and b = H cosh -1 Á 2 + 1˜
w Ë H ¯ w Ë H ¯
Adding the above equations and knowing that a + b l, we finally get

wl = cosh -1 Ê wh1 + 1ˆ + cosh -1 Ê wh2 + 1ˆ (3.12)


H ÁË H ˜¯ ÁË H ˜¯
Truss, Frames, and Cables 111

Mostly the geometric parameters like l, h1, and h2 are provided and loading
intensity is known. Thus we can compute the value of H and thereby the
tension T.

3.13.2 Parabolic Cables


y In this type of cable, the loading in-
tensity is assumed per unit length of
B (xB, yB) the span of the cable. In Fig. 3.22, a
parabolic cable is shown, where the
A (xA, yA) h2 coordinate axes have been assumed
C (x, y)
h1 through lowest point O and the supports
x
O are at A (xA, yA) and B (xB, yB), C be-
a b ing any arbitrarily chosen point on the
l
parabola. The total load acting on the
Fig. 3.22 OC span is wx. Following the governing
differential equation, as derived earlier, we obtain
dy wx
= (3.13)
dx H
Integrating within limits, we have
y x
2
Ú
y = dy = w
0
H Ú xdx = wx
0
2H
(3.14)

This is the equation of a parabola and hence the name parabolic cable. So,
the tension at both support ends A and B are
TA = H 2 + w 2 a 2 and TB = H 2 + w 2 b2
Using Eq. (3.14), we find
2 wb2
h1 = wa and h2 = (3.14a)
2H 2H
Introducing h1 - h2 h, we find,
2hH w(b2 - a2) (3.15)
As a + b l, we obtain, a = l - hH and b = l + hH (3.15a)
2 wl 2 wl
Substituting the above expression for b into the expression of h2 in (3.14a),
we find
H 2 - 2wl2 H ÊÁ h2 - h ˆ˜ + w l2 = 0
2 2 4

h Ë 2 ¯ 4h
Solving the above equation, we have

H = wl2 ÊÁ h2 - h ± h1h2 ˆ˜
2
(3.16)
h Ë 2 ¯
When the end supports are at the same vertical plane, h1 = h2 = h and
a = b = l , then the above equation reduces to
2
112 Engineering Mechanics

2
H = wl (3.17)
8h
Now we can determine the length of the cable. Let us assume the length from
O to A is lSA and from O to B is lSB. Now
2 È 2˘
Ê dy ˆ
ds =dy2 + dx 2 = dx 1 + Á ˜ = Í 1 + ÊÁ wx ˆ˜ ˙ dx
Ë dx ¯ Í ËH¯ ˙
Î ˚
Integrating the above equation within limits for the range O to A, we obtain
-1
lSA xA È 2˘
Í 1 + ÁÊ wx ˜ˆ ˙ dx
Ú ds = Ú Í ËH¯ ˙
0 0 Î ˚
xA -1
È 2 4 ˘
or lSA = Ú Í1 + 1 ÊÁ wx ˆ˜ + 1 Ê 1 - 1ˆ 1 ÊÁ wx ˆ˜ + "˙ dx
ÍÎ 2 Ë H ¯ 2 Ë 2 ¯ 2! Ë H ¯ ˙˚
0

w 2 xA3 w 4 xA5
= xA + 2
- +"
6H 40 H 4

wxA2
Substituting yA = , we obtain
2H

È Êy ˆ
2
Êy ˆ
4 ˘
lSA = xA Í1 + 2 Á A ˜ - 2 Á A ˜ + "˙ (3.18a)
Í 3 Ë xA ¯ 5 Ë xA ¯ ˙
Î ˚
Similarly for span O to B, we have

È Ê yB ˆ
2
Ê yB ˆ
4 ˘
lSB = xB 1 + Á ˜ - Á ˜ + "˙
Í 2 2 (3.18b)
Í 3 Ë xB ¯ 5 Ë xB ¯ ˙
Î ˚
When supports A and B are at the same level, xA xB x0 and yA yB y0,
total length of the cable
È Ê y0 ˆ
2
Ê y0 ˆ
4 ˘
lS = lSA + lSB = 2 x0 1 + Á ˜ - Á ˜ + "˙
Í 2 2 (3.19)
Í 3 Ë x0 ¯ 5 Ë x0 ¯ ˙
Î ˚

RECAPITULATION

z A truss is a load-bearing structure formed by a group of members arranged in one


or more triangles.
z Truss members are straight and connected with frictionless pins. The displacement
of the truss is small. Loads are applied only at the joints.
Truss, Frames, and Cables 113

z If m is the number of members in a plane truss, j is the number of joints, and r is


the number of reactive components, then the necessary condition for (i) statical
determinacy: m + r 2j, (ii) statical indeterminacy internally: m + r > 2j, and (iii)
instability: m + r < 2j.
z In the method of joints for analysis of truss, find the reaction and then choose the
joint having not more than two unknowns. Solve the joint for unknown member
forces. Proceed to next available joint with less than three unknowns.
z In the method of sections for analysis of truss, choose one or more section planes
passing through the desired members of unknown forces. Take either part of sec-
tion or both to find the unknown forces. However, the choice of section is case
specific.
z In mixed method of analysis of truss, first, one or some joints are solved for one or
more unknown forces. Then a suitable section is selected to determine the remaining
unknowns.
z Frame is a load-bearing structure comprising several interconnected rigid bod-
ies (members) forming a definite geometrical shape, with at least one multi-force
member. To analyse a frame structure, first find the reactions, then disassemble
all or some of the members of the frame. Consider their equilibrium analysis and
obtain the desired unknowns.
z Suspension cable is idealized as of uniform cross-section, perfectly flexible, negligible
bending resistance, and of negligible self-weight while subjected to concentrated
loads. Internal force induced in the cable is always in the direction of cables.
z The governing equation of a suspension cable subjected to uniform loading is
dy W
= and tension in the cable is T = W 2 + H 2 .
dx H
z In catenary cables, loading is per unit length of the cable, and the equation of the
Ê ˆ
cable is y = c Á cosh x - 1˜ or y = c cosh x , being dependent on the choice of x-axis.
Ë c ¯ c
Other form is s = c sinh x .
c
z In parabolic cables, loading is per unit length of the span, and the equation of
wx 2 .
the cable is y =
2H

NUMERICAL EXAMPLES

A. Truss
Example 3.1 Calculate the axial force in the members BE and EC as shown in
Fig. E3.1(a).

Solution From observation, SBE 0. From 'AEB,


q tan–1 ÊÁ 2 ˆ˜ 45°
Ë 2¯
Consider equilibrium of joint A from its FBD [Fig. E3.1(b)]

ÂF
SAE
≠ y 0: 6
2
114  Engineering Mechanics

SAE

θ
A E
SAB SED
θ
θ
6 kN SAE SEC
(b) FBD of joint A (c) FBD of joint E
Fig. E3.1

So, SAE = 8.49


Consider the equilibrium of joint E [Fig. E3.1(c)]
≈ SAE SEC
≠ ÂFy = 0: 
2
=-
2
So, SEC = –8.49 kN

Example 3.2  Find the forces in the members CB, CD, and DE. Restraining link
AB is horizontal [Fig. E3.2(a)].

C
D

Fig. E3.2

Solution  From geometry, CE = 3 2m and q = 45°.


Truss, Frames, and Cables  115

Consider FBD of the whole truss and taking moment equilibrium at C


[Fig. E3.2(b)]:
S1 ¥ 3 = 3 ¥ 3 2 S1 = 4.24
Consider equilibrium of joint B Fig. E3.2(c):
Æ≈
S2
ÂF x
= 0: 
2
= S1 S2 = 6


S2
≠ ÂF y = 0: S3 = –
2
= – 4.24

Consider equilibrium of joint D [Fig. E3.2(d)]:



S2
≠ ÂF y = 0: S6 =
2
= 4.24

Æ≈
S2
ÂF x = 0: S4 = –
2
= – 4.24

Result: Force Magnitude Nature


SCB 4.24 kN C
SCD 4.24 kN C
SDE 4.24 kN T
Example 3.3  Determine the axial forces in all the members of the truss shown in
Fig. E3.3(a).

Fig. E3.3

Solution  From Fig. E3.3(a),

tan q = 0.7 and tan a = 0.7


1.8
or q = 21.26°  and  a = 35°
Consider FBD of the whole truss and taking moment equilibrium about joint A
[Fig. E3.3(b)]
900 ¥ 1.8 = RC ¥ 0.8
So, RC = 202.5
116 Engineering Mechanics

Consider FBD of joint C [Fig. E3.3(c)]:



≠ ÂF y 0: SCB sin 35° + RC 0

So, SCB –3530.48


Æ≈

ÂF x 0: SCA SCB cos 35° –2892

Consider FBD of joint B [Fig. E3.3(d)]:


Æ≈

ÂF x 0: SBA cos 21.26° + SCB cos 35° 0

Hence, SBA 3103.19


Result: Force Magnitude Nature
SCA 2892 N C
SCB 3530.48 N C
SBA 3103.19 N T
Example 3.4 Determine the axial forces of all the members of the loaded truss
[Fig. E3.4(a)].
Solution Joint C remains in equilibrium by the action of SCB and SCD which are
non-collinear.
Hence, SCB 0 SD

Fig. E3.4
Consider FBD of joint D [Fig. E3.4(b)]:

≠ ÂF y 0: SDE sin 45° –1300

So, SDE –1838.48


Æ≈

ÂF
SDE
x 0: SDB – 1300
2
Consider FBD of joint B [Fig. E3.4(c)]:
Æ≈

ÂF x 0:
SBA
2
SDB
Truss, Frames, and Cables  117

So, SBA = 1838.48



≠ ÂF y = 0:  SBE = –SBA cos 45° = –1300
Result: Force Magnitude Nature
SCB 0
SCD 0
SBA 1838.48 N T
SDB 1300 N T
SBE 1300 N C
SDE 1838.48 N C
Example 3.5  Determine the axial forces in the central member BD of the truss
shown in Fig. E3.5(a).
Solution  From geometry,
BD = 3 ¥ 3 / 2 = 1.5 3 m
and h = 1.5 sin 60° = 0.75 3 m.
Consider FBD of whole truss [Fig. E3.5(b)]:
Æ≈

ÂF x = 0:  RAH = 800

Take moment equilibrium at C,


800 ¥ h = –RAV ¥ 3
Therefore, RAV = –200 3

Fig. E3.5

Consider equilibrium of joint A [Fig. E3.5(c)]:



≠ ÂF y = 0: SAB sin 60° = –RAV
118  Engineering Mechanics

So, SAB = 400


Consider equilibrium of joint B [Fig. E3.5(d)]:
Æ≈

ÂF x = 0: SAB cos 60° = SBE cos 60°

So, SAB = SBE



≠ ÂF y = 0: SBD + (SAB + SBE) cos 30° = 0

SBD = –692.82 N

Example 3.6  Find the axial force induced in the member AD of the truss shown
in Fig. E3.6(a).
Solution  From DCED, DE = DC sin 60° = 0.75 3 m and CE = DC cos 60° = 0.75 m.
Hence, AE = 3 – 0.75 = 2.25 m. Consider equilibrium of whole truss, and moment
equilibrium at A yields [Fig. E3.6(b)],
P ¥ 1.5 + 400 ¥ AE + 800 ¥ DE = RC ¥ AC
So, RC = 0.5P + 646.41
Consider equilibrium of joint C [Fig. E3.6(c)]:

≠ ÂF y = 0: SCD sin 60° + RC = 0

Hence, SCD = –0.577P – 746.41


Æ≈

ÂF x = 0:  SCE = –SCD cos 60° = 0.289P + 373.21

Fig. E3.6
From observation of joint E, SED = 400. Now consider the equilibrium of joint D
[Fig. E3.6(d)]:
Truss, Frames, and Cables 119

Æ≈

ÂF x 0: SDB cos 60°+ SAD sin 60° 800 + SCD sin 30°

SDB + 3 SAD 853.6 – 0.577P (1)



≠ ÂF y 0: SDB sin 60° SAD cos 60° + 400 + SCD cos 30°

3 SDB – SAD –492.82 – P (2)


Solving Eqs (1) and (2), we obtain, SAD 492.82 N.
Example 3.7 For the plane truss AEDCB shown in Fig. E3.7(a), determine the
induced axial forces in members AE, BE, and BC.

Fig. E3.7

Solution Consider equilibrium of joint D [Fig. E3.7(b)]:



≠ ÂF y 0: SDE sin 30° 1000

So, SDE 2000


Æ≈

ÂF x 0: SDC –SDE cos 30° –1732.05

From observation at joint C, SCE 1000 N and SBC SDC –1732.05 N


Now consider equilibrium of joint E [Fig. E3.7(c)]
Æ≈

ÂF x 0: (SAE + SBE) cos 30° SE cos 30°

Therefore, SAE + SBE 2000 (1)



≠ ÂF y
0: SCE + SDE sin 30° + SBE sin 30° SAE sin 30°

So, SAE – SBE 4000 (2)


Solving Eqs (1) and (2), we have SAE 3000, SBE –1000
Result: Force Magnitude Nature
SAE 3000 N T
SBE 1000 N C
SBC 1732.05 N C

Example 3.8 Compute the force induced in the bar BD of the simply supported
loaded truss as shown in Fig. E3.8(a).
120  Engineering Mechanics

Fig. E3.8

Solution  From DFDC, h = 3a/2 . Considering FBD of the whole truss [Fig. E3.8(b)],
take moment equilibrium at joint A:
RC ¥ 2a = h ¥ 800
RC = 200 3
Now consider equilibrium of joint C [Fig. E3.8(c)]:

≠ ÂF y = 0: SCF sin 60° + RC = 0

So, SCF = –400


Æ≈

ÂF x = 0: SCD + SCF cos 60° = 0


So, SCD = 200
Consider equilibrium of joint F [Fig. E3.8(d)]:
Æ≈

ÂF x = 0: SCF cos 60° + 800 = (SBF+ SFD) cos 60°

So, SBF + SFD = 1200 (1)



≠ ÂF y = 0: SBF sin 60° = (SFD + SCF) sin 60°

So, SBF – SFD = –400 (2)


Solving Eqs (1) and (2), SBF = 400, SFD = 800.
Consider equilibrium of joint B [Fig. E3.8(e)]:
Æ≈

ÂF x = 0: SBE sn 30° = SBF sin 30°

So, SBE = SBF = 400



≠ ÂF y = 0: SBD = –(SBE + SBF) cos 30°
= –692.82 N
Truss, Frames, and Cables  121

Example 3.9  Determine the axial forces induced in each of the members of the truss
loaded and supported as shown in Fig. E3.9(a).

Fig. E3.9

Solution  Here DABC is isosceles and –A = 30° = –C. Let AB = a = BC. From DABD,

AD = AB = 2 a
sin 60∞ 3
BD = AD cos 60° = a
3
Again DBDC is isosceles and BD = DC. Consider FBD of the whole truss
[Fig. E3.9(b)], and moment equilibrium at A yields,
500 ¥ AD = RC (AD + DC)
Hence, RC = 1000
3
Consider equilibrium of joint C [Fig. E3.9(c)]:

≠ ÂF y = 0: SCB sin 30° = –RC

2000
So, SCB = -
3
Æ≈

ÂF x = 0: SDC + SCB cos 30° = 0

1000
So, SDC =
3
Now consider equilibrium of joint B [Fig. E3.9(d)]:
Æ≈

ÂF x = 0: SCB cos 30° + SBD cos 60° – SBA cos 30° = 0

So, SBD – SBA 3 = 2000 (1)



3
≠ ÂF y
= 0: SCB sin 30° + SBD sin 60° + SBA sin 30° = 0
122  Engineering Mechanics

So, SBD 3 + SBA = 2000 (2)


3
Solving Eqs (1) and (2), we get

SBD = 1000 , SBA = - 1000


3 3
Consider equilibrium of joint D [Fig. E3.9(e)]:
Æ≈

ÂF x = 0: SBD cos 60° + SDA = SDC

Hence, SDA = 500


3
Result: Force Magnitude Nature
SBA 333.34 N C
SBD 577.35 N T
SDC 577.35 N T
SDA 288.68 N T
SCB 666.67 N C
Example 3.10  Determine the induced axial force in the bottom tie member DC of
the loaded truss [Fig. E3.10(a)].

Fig. E3.10

Solution  From geometry, h = a tan 45° = a. Consider equilibrium of the whole truss


[Fig. E3.10(b)]. From moment equilibrium at D, we obtain
RC ¥ 2a = 2h ¥ 1000
Hence, RC = 1000 N
Consider equilibrium of joint F [Fig. E3.10(c)]:

≠ ÂF y = 0: SAF sin 45° + SBF sin 45° = 0
Truss, Frames, and Cables  123

So, SAF = –SBF (1)


Æ≈

ÂF x = 0: SBF cos 45° + 1000 = SAF cos 45°

So, SBF = –707.106 [Substituting Eq. (1)]


Therefore, SAF = 707.106
Consider equilibrium of joint B [Fig. E3.10(d)]:
Æ≈

ÂF x = 0: SBF cos 45° + SBE = 0

Hence, SBE = 500



≠ ÂF y = 0: SBC = SBF sin 45° = –500

Consider equilibrium of joint C [Fig. E3.10(e)]:



≠ ÂF y = 0: SCE sin 45° + SBC + RC = 0

Hence, SCE = –707.106


Æ≈

ÂF x = 0: SDC + SCE cos 45° = 0

So, SDC = 500 N

Example 3.11  Compute the axial forces induced in all the members of the loaded
truss shown in Fig. E3.11(a). ABCD is square and the diagonal AC is horizontal.
Assume B and C are roller supports.
D

A
C

Fig. E3.11

Solution  Consider equilibrium of the whole truss [Fig. E3.11(b)]:


124 Engineering Mechanics


≠ ÂF y 0: RB 1000
Now consider equilibrium of joint B [Fig. E3.11(c)]:
Æ≈

ÂF
SBA SBC
x 0: =
2 2

So, SBA SBC


ÂF
SBC
≠ y 0: 2 –1000
2
Hence, SBC –707.1 SBA
Consider equilibrium of joint A [Fig. E3.11(d)]:

ÂF
SAD SBA
≠ y 0: = + 1000
2 2
Therefore, SAD 707.1
Æ≈

ÂF x 0: (SAD + SBA) 1 + SAC


2
0

Hence, SAC 0
Consider equilibrium of joint D [Fig. E3.11(e)]:

≠ ÂF y 0: SDC –SAD –707.1
Æ≈

ÂF
SAD SDC
x 0: = + SDE
2 2
So, SDE 1000
Result: Force Magnitude Nature
SAD 707.1 N T
SBA 707.1 N C
SBC 707.1 N C
SDC 707.1 N C
SAC 0
SDE 1000 N T
Example 3.12 Determine the axial forces in the members of the plane truss as shown
in Fig. E3.12(a).
Truss, Frames, and Cables 125

Fig. E3.12
Solution Here,
Ê ˆ
q tan–1 Á 0.5 ˜ 26.57°
Ë 1 ¯
Consider equilibrium of joint C [Fig. E3.12(b)]:

≠ ÂF y
0: SCD sin q + 500 0

So, SCD –1117.84


Æ≈

ÂF x 0: SCB + SCD cos q 0

So, SCB 1000


Consider equilibrium of joint D [Fig. E3.12(c)]:
Æ≈

ÂF x 0: SDE SCD cos q –1000



≠ F y
0: SDB –SCD sin q 500

Consider equilibrium of joint B [Fig. E3.12(d)]:


Æ≈

ÂF x 0: (SBA + SBE) cos q SCB

Therefore, SBA + SBE 1118.08 (1)



≠ ÂF y 0: SDB + SBE sin q SBA sin q

Therefore, SBA – SBE 1117.84 (2)


Solving Eqs (1) and (2), we obtain SBA 1117.96 and SBE –0.12. It is worth men-
tioning here that the magnitude of the force SBE is practically zero. But a very small
value is obtained here due to successive approximation effect.
Result: Force Magnitude Nature
SBA 1117.96 N T
SBE 0
SCB 1000 N T
SDE 1000 N C
SCD 1117.84 N C
Example 3.13 Determine the axial forces in the members CF, FD, DG, GE, and EB
of the simply supported loaded truss shown in Fig. E3.13(a).
Solution From geometry,

AB AE 4 3m
cos 30∞
126  Engineering Mechanics

Fig. E3.13
and from DDGE,
4 tan 30∞
tan a = DG =
DE 2
a = 49.1°
Consider equilibrium of the whole truss [Fig. E3.13(b)].

ÂM at A = 0: RB ¥ AB = 900 ¥ 2

Hence, RB = 150 3
Truss, Frames, and Cables 127

Consider equilibrium of joint B [Fig. E3.13(c)]:


Æ≈

ÂF x 0: SBG cos 30° RB sin 30°

So, SBG 150



≠ ÂF y 0: SBE + SBG sin 30° + RB cos 30° 0

So, SBE –300


Consider equilibrium of joint E [Fig. E3.13(d)]:

≠ ÂF y 0: SEG sin 49.1° + SBE 0

Hence, SEG 396.9


Æ≈

ÂF x 0: SDE + SEG cos 49.1° 0

So, SDE –259.87


Consider equilibrium of joint G [Fig. E3.13(e)]
Æ≈

ÂF x 0: SEG cos 49.1°  SBG cos 30° SGF sin 60°

So, SGF 450



≠ ÂF y 0: SDG + SEG sin 49.1° + SGF cos 60° – SBG sin 30° 0

Hence, SDG – 450


Consider equilibrium of joint D [Fig. E3.13(f)]:

≠ ÂF y 0: SDF sin 30° + SDG 0

So, SDF 900


From observation of joint C, we obtain SCF –900:
Result: Force Magnitude Nature
SCF 900 N C
SDF 900 N T
SDG 450 N C
SEG 396 N T
SBE 300 N C
Example 3.14 A truss is loaded and supported as shown in Fig. E3.14(a). Compute
the axial force in the members DE, EF, and FC.
128 Engineering Mechanics

Fig. E3.14

Solution From geometry, h 3/2 – 4 2 3m .


Consider equilibrium of the whole truss [Fig. E3.14(b)]:

ÂM at B 0: RC – 4 600 – h + 600 – 2

So, RC 819.62
Consider equilibrium of joint C [Fig. E3.14(c)]:

≠ ÂF y 0: SEC sin 60° –RC

So, Æ≈
SEC –946.41

ÂF x 0: SFC –SEC cos 60° 473.2

Consider equilibrium of joint A [Fig. E3.14(d)]:



≠ ÂF y 0: SAE cos 30° –SAD cos 30°

So, SAE –SAD


Æ≈

ÂF x 0: SAE sin 30° + 600 SAD sin 30°

So, SAD 600


Therefore, SAE –600
Consider equilibrium of joint E [Fig. E3.14(e)]:

≠ ÂF y
0: SAE sin 60° SEF sin 60° + SEC sin 60°

Hence, SEF 346.41


Æ≈

ÂF x 0: SEC cos 60° SDE + SAE cos 60° + SEF cos 60°

So, SDE –346.41


Result: Force Magnitude Nature
SDE 346.41 N C
SEF 346.41 N T
SFC 473.2 N T

Example 3.15 Find the forces induced in each member of the loaded truss
[Fig. E3.15(a)].
Truss, Frames, and Cables  129

Fig. E3.15
Solution  Here, q = sin–1 (3/6) = 30° and a = tan–1 (3/3) = 45°. From geometry:

CD = AB = 62 - 32 = 3 3 m
Consider equilibrium of the whole truss, and moment equilibrium at D yields
[Fig. E3.15(b)],
S6 ¥ 3 3 = 6 ¥ (3 + 3 3 )

Hence, S6 = 9.46

≠ ÂF y = 0: RDV = 6

Æ≈
ÂF x = 0: RDH = S6 = 9.46
130 Engineering Mechanics

Consider equilibrium of joint A [Fig. E3.15(c)]:



≠ ÂF y 0: 6 + S2 sin T 0

So, S2 –12
Æ≈

ÂF x 0: S1 + S2 cos T 0

Hence, S1 10.39
From observation of joint F, we find, S8 0 and S3 S2 –12.
Consider equilibrium of joint B [Fig. E3.15(d)]:
Æ≈

ÂF x 0: S7 cos a S1

So, S7 14.7

≠ ÂF y 0: S7 sin a + S9 0

So, S9 –10.39
Consider equilibrium of joint E [Fig. E3.15(e)]:

≠ ÂF y 0: S9 + S3 sin q S4 cos q

Hence, S4 –18.93
Æ≈

ÂF x 0: S3 cos q S10 + S4 sin q

So, S10 –0.93


Consider equilibrium of joint D [Fig. E3.15(f)]:

≠ ÂF y 0: S5 + RDV + S4 cos q 0

Therefore, S5 10.39
Result: Force Magnitude Nature Force Magnitude Nature
S1 10.39 N T S6 9.46 N T
S2 12 N C S7 14.7 N T
S3 12 N C S8 0
S4 18.93 N C S9 10.39 N C
S5 10.39 N T S10 0.93 N C
Example 3.16 Determine the forces in all the members of the truss as shown in
Fig. E3.16(a). All inclined members are at 45° with the horizontal.
Solution Consider equilibrium of joint H [Fig. E3.16(b)]:
Æ≈
S11 S12
ÂF x 0:
2
=-
2
(1)


S11 S12
≠ ÂF y 0:
2
=
2
+ 3500 (2)
Truss, Frames, and Cables 131

Fig. E3.16
Solving Eqs (1) and (2), we obtain,
S12 –2474.87 and S11 2474.87
Consider equilibrium of joint F [Fig. E3.16(c)]:

S7 S11
≠ ÂF y 0:
2
=-
2
Hence, S7 –S11 –2474.87
Æ≈
S7 S11
ÂF x 0: S5 +
2
=
2
So, S5 3500
Consider equilibrium of joint G [Fig. E3.16(d)]:
≈ S9 S12
≠ ÂF y
0:
2
=-
2
So, S9 2474.87
Æ≈

ÂF
S9 S12
x 0: S10 + =
2 2
Hence, S10 –3500
From inspection of joint K, we can write
S8 S7 –2474.87
S6 S9 2474.87
132  Engineering Mechanics

Consider equilibrium of joint D [Fig. E3.16(e)]:


Æ≈
S1 S2 S6
ÂF x = 0:
2
+
2
= S5 +
2

Therefore, S1 + S2 = 7424.62 (3)


≈ S1 S2 S6
≠ ÂF y = 0:
2
=
2
+
2
Therefore, S1 – S2 = 2474.87 (4)
Solving Eqs (3) and (4), we have,
S1 = 4949.75
S2 = 2474.87
Consider equilibrium of joint E [Fig. E3.16(f)]:
Æ≈
S3 S4 S8
ÂF x = 0:
2
+
2
= S10 +
2
So, S3 + S4 = –7424.62 (5)
≈ S3 S8 S4
≠ ÂF y = 0:
2
+
2
=
2
So, S3 – S4 = 2474.87 (6)
Solving Eqs (5) and (6), we obtain S3 = –2474.87 and S4 = – 4949.87.
Result: Force Magnitude Nature Force Magnitude Nature
S1 4949.75 N T S7 2474.87 N C
S2 2474.87 N T S8 2474.87 N C
S3 2474.87 N C S9 2474.87 N T
S4 4949.87 N C S10 3500 N C
S5 3500 N T S11 2474.87 N T
S6 2474.87 N T S12 2474.87 N C

Example 3.17  Find the forces in all the members of the loaded truss as shown in
Fig. E3.17(a).
A

B C

D
F
E
Truss, Frames, and Cables 133

Fig. E3.17
–1
Solution From geometry, q tan (5/4) 51.34°.
Consider equilibrium of the whole truss [Fig. E3.17(b)]:

ÂF
≠ y 0: RDV 35

ÂM at D 0: RA – 10 35 – 8

Therefore, RA 28
Æ≈

ÂF x 0: RDH RA 28
Consider equilibrium of joint A [Fig. E3.17(c)]:
Æ≈

ÂF x 0: S1 cos q –RA

So, S1 –44.82

≠ ÂF y 0: S2 –S1 sin q 35
From observation of joint D [Fig. E3.17(d)], we have, S5 –35 and S6 –28.
Consider equilibrium of joint F [Fig. E3.17(e)]:

≠ ÂF y 0: S9 sin q 35

Hence, S9 44.82
Æ≈

ÂF x 0: S8 –S9 cos q –28

Consider equilibrium of joint E [Fig. E3.17(f)]:


Æ≈

ÂF x 0: S4 cos q + S6 S8
So, S4 0

≠ ÂF y 0: S7 + S4 sin q 0
So, S7 0
134 Engineering Mechanics

Consider equilibrium of joint B [Fig. E3.17(g)]:


Æ≈

ÂF x 0: S3 –S1 cos q 28

Result: Force Magnitude Nature


S1 44.82 N C
S2 35 N T
S3 28 N T
S4 0
S5 35 N C
S6 28 N C
S7 0
S8 28 N C
S9 44.82 N T
Example 3.18 Compute all the forces of the members of the given truss [Fig.
E3.18(a)].
Solution Consider equilibrium of the whole truss [Fig. E3.18(b)]:

ÂM C 0: RD – 2.5 25 – 2.5 + 40 – 5 + 50 – 2.5


Therefore, RD 155

≠ ÂF y 0: RCV 30 + 25 – RD –100
Æ≈

ÂF x 0: RCH –50 – 40 –90

From observation of joint D, we see that S8 –RD –155. Again from one observa-
tion of joint A, we can say that
S1 –40 and S2 –30
Consider equilibrium of joint C [Fig. E3.18(c)]:
Æ≈

ÂF
S7
x 0: –RCH
2
So, S7 127.28

ÂF
S7
≠ y 0: S6 –RCV – 10
2
Consider equilibrium of joint B [Fig. E3.18(d)]:

ÂF
S3
≠ y 0: + S2 S6
2
Hence, S3 56.57
Æ≈
S3
ÂF x 0:
2
+ S5 –50

So, S5 –90
Consider equilibrium of joint F [Fig. E3.18(e)]:

ÂF
S3
≠ y 0: S4 –25 – –65
2
Truss, Frames, and Cables  135

F
A

E
B

C D

Fig. E3.18

Result: Force Magnitude Nature


S1 40 N C
S2 30 N C
S3 56.57 N T
S4 65 N C
S5 90 N C
S6 10 N T
S7 127.28 N T

Example 3.19  Determine the member forces of the overhang truss [Fig. E3.19(a)].
Ê ˆ Ê ˆ
Solution  Here, q = tan–1 Á 4 ˜ = 48.81°  and  a = tan–1 Á 2 ˜ = 29.74°
Ë 3.5 ¯ Ë 3.5 ¯
12 kN 15 kN
15 kN B C
12 kN  D
B
3.5 m C 1 3.5 m
D
4 E
8 3
5 2 RAH 
4m 7 F
E
9 6
A 3.5 m F
RAV RF
(a) (b) FBD of whole truss
136 Engineering Mechanics

Fig. E3.19
Consider equilibrium of whole truss [Fig. E3.19(b)]:

ÂM at A 0: RF – 3.5 12 – 7 + 15 – 10.5

Therefore, RF 69
Consider equilibrium of joint D [Fig. E3.19(c)]:

≠ ÂF y 0: S2 sin a –15

So, S2 –30.24
Æ≈

ÂF x 0: S1 –S2 cos a 26.26

From observation of joint C, we obtain S3 –12 and S4 S1 26.26.


Consider equilibrium of joint E [Fig. E3.19(d)]:
Æ≈

ÂF x 0: S5 cos a + S6 cos a S2 cos a

So, S5 + S6 –30.24 (1)



≠ ÂF y 0: S2 sin a + S5 sin a + S3 S6 sin a
Hence, S6 – S5 –54.43 (2)
Solving Eqs (1) and (2), we obtain S5 12.1 and S6 –42.34.
Consider equilibrium of joint F [Fig. E3.19(e)]:
Æ≈

ÂF x 0: S9 S6 cos a –36.76

≠ F y 0: S7 + S6 sin a + RF 0
So, S7 – 48
Consider equilibrium of joint B [Fig. E3.19(f)]:
Æ≈

ÂF x 0: S8 cos q S 4 + S5 cos D

Therefore, S8 55.83
Truss, Frames, and Cables  137

Result: Force Magnitude Nature Force Magnitude Nature


S1 26.26 N T S6 42.34 N C
S2 30.24 N C S7 48 N C
S3 12 N C S8 55.83 N T
S4 26.26 N T S9 36.76 N C
S5 12.1 N T
Example 3.20  Determine the axial forces induced in all the members of the loaded
truss [Fig. E3.20(a)].

E D

A
B C

Fig. E3.20

Solution  Here q = tan–1 Ê 5 ˆ = 26.57°


ÁË 10 ˜¯
Ê 5ˆ
and a = tan–1 ÁË ˜¯ = 45°
5
Consider FBD of the whole truss. Taking moment equilibrium at A [Fig. E3.20(b)],
we have
ÂM at A = 0: 10 ¥ 5 + 20 ¥ 10 + 25 ¥ 15 – 10 ¥ 5 = RC ¥ 10

Therefore, RC = 57.5
Æ≈

ÂF x = 0: RAH = 10
138 Engineering Mechanics


≠ ÂF y 0: RAV 10 + 20 + 25 – RC –2.5
Consider equilibrium of joint A [Fig. E3.20(c)]:

≠ ÂF y 0: S1 sin q –RAV

So, S1 5.59
Æ≈

ÂF x 0: S2 –S1 cos q – RAH –15


From observation of joint B, we obtain S3 0 and S4 S2 –15.
Consider equilibrium of joint F [Fig. E3.20(d)]:
Æ≈

ÂF x 0: (S5 + S6) cos q S1 cos q

Therefore, S5 + S6 5.59 (1)



≠ ÂF y 0: S6 sin q – 10 – S5 sin q – S1 sin q 0

S 6 – S5 27.95 (2)
Solving Eqs (1) and (2), we have S6 16.77 and S5 –11.18.
Consider equilibrium of joint E [Fig. E3.20(e)]:

≠ ÂF y 0: S6 sin q + S7 + 20 0

So, S7 –27.50
Æ≈

ÂF x 0: S8 S6 cos q 15
Consider equilibrium of joint D [Fig. E3.20(f)]:

ÂF
S9
≠ y 0: –25
2
Hence, S9 –35.56
Result: Force Magnitude Nature
S1 5.59 kN T
S2 15 kN C
S3 0
S4 15 kN C
S5 11.18 kN C
S6 16.77 kN T
S7 27.50 kN C
S8 15 kN T
S9 35.36 kN C
Example 3.21 Determine the forces in the members AC, DE, and GH of the truss
loaded and supported as shown in Fig. E3.21(a).
Solution Here, q 45°. From observation of joint D, we can say S10 –10.
Consider equilibrium of whole truss [Fig. E3.21(b)]:

Â
RH
M at A 0: 10 – 2 + 10 – 4 + 10 – 6 + 5 – 8 –8
2
Truss, Frames, and Cables  139

A C D G H

B E F

Fig. E3.21
Hence, RH = 28.28
Æ≈
RAH RAV RH
ÂF x = 0:
2
+
2
=
2
Therefore, RAH + RAV = 28.28 (1)
≈ RAH RAV RH
≠ ÂF y = 0:
2
-
2
+ 5 + 10 + 10 + 10 + 5 –
2
=0

RAH – RAV = –28.28 (2)


Solving Eqs (1) and (2), we obtain RAH = 0 and RAV = 28.28.
Consider equilibrium of joint H [Fig. E3.21(c)]:
≈ S8 RH
≠ ÂF y = 0: 5+
2
=
2
So, S8 = 21.21
Æ≈
S8 RH
ÂF x = 0: S4 +
2
+
2
=0

So, S4 = –35
Consider equilibrium of joint A [Fig. E3.21(d)]:
≈ S7 RAV
≠ ÂF y = 0: 5+
2
=
2
So, S7 = 21.21
140 Engineering Mechanics

Æ≈

ÂF
S7 RAV
x 0: S1 + + 0
2 2
So, S1 –35
Result: Force Magnitude Nature
S1 35 kN C
S10 10 kN C
S4 35 kN C
Example 3.22 A truss is supported and loaded as shown in Fig. E3.22(a). Determine
the axial forces in the members AC and CD.

Fig. E3.22
1.5 2 - 1.5
Solution Here, tan q and tan a
20 4
or q 36.87° and a 7.13°
Consider equilibrium of joint G [Fig. E3.22(b)]:

≠ ÂF y 0: SFG sin q – 4500

So, SFG –7500


Æ≈

ÂF x 0: SEG –SFG cos q 6000 N


From observation of joint E, we obtain SCE SEG 6000 N and SEF –3000 N.
Consider equilibrium of joint F [Fig. E3.22(c)]:
Æ≈

ÂF x 0: SFG cos q SFC cos q + SDF cos a

Therefore, 0.8SFC + 0.99SDF –6000 (1)



≠ ÂF y
0: SEF + SFG sin q + SFC sin q SDF sin a

0.6SFC – 0.124SDF 7500 (2)


Solving Eqs (1) and (2), we have SFC 9637.91 N and SDF –13848.82 N.
Truss, Frames, and Cables  141

Consider equilibrium of joint C [Fig. E3.22(d)]:



≠ ÂF y = 0: SCD = –SFC sin q = –5782.76 N
Æ≈
ÂF x = 0: SAC = SCE + SFC cos q = 13710.32 N

Example 3.23  Find the forces in all the members of the loaded truss [Fig.
E3.23(a)].

 E

 
F
B D

Fig. E3.23
4-2
Solution  Here, f = tan–1 2 = 26.57°  and  q = tan–1 = 14.04°
4 8
2 + 4 tan q
tan  a= = 0.75
4
Hence, a = 36.87°
Consider equilibrium of whole truss [Fig. E3.23(b)]:

ÂM at A = 0: RBH ¥ 4 = 17 ¥ 8

Therefore, RBH = 34
Æ≈
ÂF x = 0: RAH = –RBH = –34


≠ ÂF y = 0: RAV = 17
142  Engineering Mechanics

From observation of joint F, we can say S8 = 0 and S9 = 17.


Consider equilibrium of joint E [Fig. E3.23(c)]:
Æ≈
ÂF x = 0: S6 cos q + S7 cos f = 0

Hence, 0.97S6 + 0.89S7 = 0 (1)



≠ Fy = 0: Â S6 sin q = S7 sin f + S9
0.24 S6 – 0.447 S7 = 17 (2)
Solving Eqs (1) and (2), we obtain S6 = 23.38 and S7 = –25.48.
Consider equilibrium of joint D [Fig. E3.23(d)]:
Æ≈

 F = 0:
x S4 = S7 cos f = –22.79

≠ Â F = 0: y S5 = –S7 sin f = 11.40
Consider equilibrium of joint A [Fig. E3.23(e)]:
Æ≈
ÂF x = 0: S1 cos q = –RAH

So, S1 = 35.05

≠ ÂF y = 0: S3 + S1 sin q = RAV

So, S3 = 8.5
Consider equilibrium of joint B [Fig. E3.23(f)]:

≠ ÂF y = 0: S2 sin a = –S3
So, S2 = –14.17
Result: Force Magnitude Nature
S 1 35.05 kN T
S2 14.17 kN C
S 3 8.50 kN T
S 4 22.79 kN C
S 5 11.40 kN T
S6 23.38 kN T
S 7 25.48 kN C
S8 0
S9 17.00 kN T
Example 3.24  Determine the axial forces in all the members of the loaded truss
[Fig. E3.24(a)].
45 kN
C
25 kN 15 kN
C
B 6 8 D 1m
B D

7
1 5 9 12 E
3 11 3m
2 4 10 13 E A F G H
A
3m F 3m G 3m H 3m

(a)
Truss, Frames, and Cables 143

Fig. E3.24
Ê 1ˆ Ê 3ˆ
Solution Here, q tan–1
ÁË 3 ˜¯ 18.43° and a tan–1 ÁË 3 ˜¯ 45°
Consider FBD of the whole truss [Fig. E3.24(b)]:

ÂM at A
0: REV – 12 25 – 3 + 45 – 6 + 15 – 9

Therefore, REV 40
Consider FBD of joint E [Fig. E3.24(c)]:

≠ ÂF y 0: S12 sin a –REV

So, S12 –56.57


Æ≈

ÂF x 0: S13 –S12 cos a 40

Consider equilibrium of joint D [Fig. E3.24(d)]:


Æ≈

ÂF x 0: S8 cos q + S9 cos a S12 cos a

Hence, 0.95S8 + 0.707S9 –40 (1)



≠ ÂF y 0: S8 sin q (S9 + S12) sin a + 15

Hence, 0.32S8 – 0.707S9 –25 (2)


Solving Eqs (1) and (2), we obtain S8 –51.18 and S9 12.19. From observation of
joints F and H, S3 0, S11 0, and S10 S13 40.
Consider equilibrium of joint C [Fig. E3.24(e)]:
Æ≈

ÂF x 0: S6 cos q S8 cos q

So, S6 S8 –51.18

≠ ÂF y 0: S7 + 45 + (S6 + S8) sin q 0
144 Engineering Mechanics

So, S7 –12.64
Consider equilibrium of joint B [Fig. E3.24(f)]:
Æ≈

ÂF
S1 S5
x 0: = + S6 cos q
2 2
Hence, S1 – S5 –68.67 (3)

ÂF
S1 S5
≠ y 0: + + 25 S6 sin q
2 2
Hence, S1 + S5 –58.24 (4)
Solving Eqs (3) and (4), we obtain S1 –63.46 and S5 5.22.
Consider equilibrium of joint G [Fig. E3.24(g)]:
Æ≈

ÂF x 0:
S9
2
+ S10
S5
2
+ S4

Hence, S4 44.93
From observation of joint F, S2 S4 44.93.
Result: Force Magnitude Nature Force Magnitude Nature
S1 63.46 kN C S8 51.18 kN C
S2 44.93 kN T S9 12.19 kN T
S3 0 S10 40.00 kN T
S4 44.93 kN T S11 0
S5 5.22 kN T S12 56.57 kN C
S6 51.18 kN C S13 40.00 kN T
S7 12.64 kN C
Example 3.25 Solve the simple truss for the forces in the members EF and EG shown
in Fig. E3.25(a). All interior angles are 60° or 120°.

Fig. E3.25
Truss, Frames, and Cables  145

Solution  Consider equilibrium of joint A [Fig. E3.25(b)]:


Æ≈
ÂF x = 0: SAB cos 30° = SAF cos 30°

So, SAB = SAF



≠ ÂF y = 0: (SAF + SAB) sin 30° = 20

Therefore, SAB = SAF = 20


Consider equilibrium of joint F [Fig. E3.25(c)]:
Æ≈
ÂF x = 0: SGF cos 30° = –SAF cos 30°

So, SGF = –20



≠ ÂF y = 0: SEF + SGF sin 30° = SAF sin 30°
So, SEF = 20
Consider equilibrium of joint E [Fig. E3.25(d)]:
Æ≈

ÂF x = 0: SDE cos 30° = –SEG cos 30°

Hence, SDE = –SEG



≠ ÂF y = 0: SDE sin 30° = SEF + SEG sin 30°

SEG = –20

Example 3.26  Find the axial forces of the simply supported truss as shown in Fig.
E3.26(a).

B
A

C
E
D

Fig. E3.26
Solution  Here q = 45°.
146  Engineering Mechanics

Consider equilibrium of whole truss [Fig. E3.26(b)]:


ÂM at C
= 0: RE ¥ 6 = 5 cos 30° ¥ 3 + 5 sin 30° ¥ 3
Therefore, RE = 3.415

≠ ÂF y = 0: RCV = 5 cos 30° – RE = 0.915
Æ≈
ÂF x = 0: RCH = 5 sin 30° = 2.5
From observation of joint E, we can say
S1 = –RE = –3.415  and  S2 = 0
Consider equilibrium of joint C [Fig. E3.26(c)]:
≈ S6
≠ ÂF y = 0:
2
+ RCV = 0

So, S6 = –1.29
Æ≈

ÂF x = 0: RCH – S7 –
S6
2
=0

So, S7 = 3.41
Consider equilibrium of joint D [Fig. E3.26(d)]:
Æ≈
S3
ÂF x = 0:
2
= S7

So, S3 = 4.82

S
≠ ÂF y = 0: S5 + 3 = 0
2
So, S5 = –3.41
Consider equilibrium of joint A [Fig. E3.26(e)]:
Æ≈
S3
ÂF x = 0: S4 +
2
=0

So, S4 = –3.41
Result: Force Magnitude Nature
S1 3.415 kN C
S2 0
S3 4.82 kN T
S4 3.41 kN C
S5 3.41 kN C
S6 1.29 kN C
S7 3.41 kN T
Example 3.27  Determine the ground reactions of a Wichert Truss, loaded and
supported as in Fig. E3.27(a).

 

Truss, Frames, and Cables  147

Solution  Approach I: SAB


Here, tan a = 3 = 1 and tan b = 3 A
a
12 4 4
SAD
As the external force is vertical, no horizontal force com-
ponent at hinge support F will exist.
RA (c)
Let RF = P Newton
SFD
SFE
So, RA = RC = 500 - P N
2 b b
Consider force equilibrium of joint A. (Fig. E3.27a) F

≠ ÂF y = 0: SAB sin a = – RA
(d)
RF

P - 500 SBD
or SAB =
2sin a SAD b
Æ≈ D
ÂF x = 0: SAD = – SAB cos a
SFD
500 - P (e)
    = cot a Fig. E3.27
2
Consider force equilibrium of joint F. (Fig. E3.27b)
Æ≈

ÂF x = 0: SFE cos b = SFD cos b

or SFE = SFD

≠ ÂF y = 0: (SFD + SFE) sin b = –RF

or 2SFD sin b = – P

or SFD = - P
2 Sin b
Consider force equilibrium of joint D. (Fig. E3.27c)

≠ ÂF y = 0: SFD sin b = SBD sin b
or SFD = SBD
Æ≈
ÂF x = 0: SAD = 2SFD cos b

500 - P 500 - P
or cot a = - P cos b or  ¥4+P¥ 4 =0
2 sin b 2 3

or 6(500 – P) + 4P = 0  or  P = 1500 N


So, RF = 1500 N and RA = RC = –500 N (downwards)

SAB
a
  A SAD

RA
(a) (b)
148  Engineering Mechanics

Approach II:  Henneberg’s Method SBD


Let us insert a member BF within the truss. As the truss is b
in equilibrium, BF will be a redundant member and thus D b
SAD
the axial force in member BF must be zero. (c) SFD
System I [Fig. E3.27(A): (a) to (d)]:  First we will find the SFD
SBF SFE
axial force in member BF when the external load 500 N at
B is active only and no support at F. Hence b b
F
RA = RC = 500 = 250 N (upwards). (d)
2

  SAB
a
A
SAD

RA
(f)

Consider force equilibrium of joint A.


≈ SBD
≠ Â Fy = 0 : SAB sin a = - 250
b
= - 250
D b
or SAB SAD
sin a
(g) SFD
Æ≈

ÂF x = 0: SAD = - SAB cos a = 250 cot a SFD


SBF SFE
b b
Consider force equilibrium of joint D. F

ÂF = 0 :
≠ y
SFD = SBD RF
(h)
Æ≈

ÂF = 0 :
x 2 SFD cos b = SAD
Fig. E3.27 (A)

or SFD = 250 cot a = 125 cot a


2 cos b cos b
Consider force equilibrium of joint F.
Æ≈

ÂF x = 0: SFD = SFE


≠ ÂF y = 0: SBF = - 2SFD sin b
= - 2 ¥ 125 cot a sin b
cos b
= -250 cot a tan b = -250 ¥ 4 ¥ 3 = -750 (1)
4
System II [Fig. E3.27(A): (e) to (h)]:  Second, we will determine the axial force in
member BF, where no external load acts at B but support exists at F. Let us assume
RF = P. In the absence of all external forces in the truss and also due to symmetry,
we can infer as
RA = RC = P/2,
i.e., acting downwards.
Truss, Frames, and Cables 149

Consider force equilibrium of joint A.



≠ ÂF y = 0: SAB sin a = RA

or SAB = P
2 sin a
Æ≈

ÂF SAD = - SAB cos a = - P cos a = - P


x = 0: 2 sin a 2 tan a
Consider force equilibrium at joint D. Proper observation suggests that, SBD = SFD .
Æ≈

ÂF x = 0: 2 SFD cos b = SAD

SFD = - P ¥ 1 = - P
or .
2 tan a 2 cos b 4 tan a cos b
Consider force equilibrium at joint F . Proper observation suggests that SFD SFE.

≠ ÂF y = 0: SBF + RF + 2SFD sin b = 0

P sin b P tan b
or SBF = - P + = -P = P ¥ 3 ¥ 4 -P = P (2)
2 tan a cos b 2 tan a 2 4 1 2
Applying the method of superposition, considering Eqs (1) and (2), the summation
will be zero.
So, -750 + P = 0
2
or P 1500
So, RF 1500 N
Therefore, RA RC 1500 750 N (downwards)
2
Finally, RA RC 250 – 750 – 500 N (downwards)

Example 3.28 Compute the axial forces induced in all the members of the simply
supported truss as shown in Fig. E3.28(a).
150 Engineering Mechanics

Solution From observation of the symmetrical truss, it can be concluded that there
will be no horizontal component of reaction force at hinge A. Also
RA RB 22,500 N
On inspection of joints A and B, it can be said
S5 0 and S11 0, S1 –22500 and
S10 –22500
Consider equilibrium of joint E [Fig. E3.28(b)]:

S3
≠ ÂF y 0:
2
–S1

Therefore, S3 31819.81
Æ≈
S3
ÂF x 0: S2 -
2
–22500

Consider equilibrium of joint C [Fig. E3.28(c)]:


≈ S3 S4
≠ ÂF y 0:
2
=-
2
So, S4 –31819.81
Æ≈
S4 S3
ÂF x 0:
2
+ S6
2
So, S6 45000
Consider equilibrium of joint D [Fig. E3.28(d)]:

S7 S8
≠ ÂF y 0:
2
+
2
0

Hence, S7 + S8 0 (1) Fig. E3.28


Æ≈
S7 S8
ÂF x 0: S6 +
2
=
2
Hence, S 8 – S7 63639.61 (2)
Solving Eqs (1) and (2), we obtain, S8 31819.81 and S7 –31819.81.
Consider equilibrium of joint G [Fig. E3.28(e)]:
Æ≈
S8
ÂF x 0: S9 -
2
–22500

Result: Force Magnitude Nature


S1 22500 N C
S2 22500 N C
S3 31819.81 N T
S4 31819.81 N C
S5 0
S6 45000 N T
S7 31819.81 N C
S8 31819.81 N T
S9 22500 N C
S10 22500 N C
S11 0
Truss, Frames, and Cables  151

Example 3.29  Determine the induced forces in each member of the truss loaded and
supported as shown in Fig. E3.29(a).
Solution  Here,
Ê 6 ˆ
q = tan–1 Á = 53.13°  and  a = tan–1 ÊÁ 6 ˆ˜ = 33.69°
Ë 4.5 ˜¯ Ë 9¯
Consider FBD of whole truss [Fig. E3.29(b)]:

ÂM at B = 0: RA ¥ 18 = 6 ¥ 13.5 + 9 ¥ 4.5

Therefore, RA = 6.75
From symmetry and loading condition of the truss it can be concluded that there will
be no horizontal component of reaction force at hinge B.
C D

A B
E F

Fig. E3.29

Consider equilibrium of joint A [Fig. E3.29(c)]:



≠ ÂF y = 0: S1 sin q = –RA

So, S1 = –8.44
Æ≈
ÂF x = 0: S4 = –S1 cos q = 5.06
From observation of joint E, we can say that S7 = 6 and S5 = S4 = 5.06.
Consider equilibrium of joint C [Fig. E3.29(d)]:

≠ ÂF y = 0: S9 sin a + S7 + S1 sin q = 0

Hence, S9 = 1.36
Æ≈
ÂF x
= 0: S2 + S9 cos a = S1 cos q

Hence, S2 = –6.19
Consider equilibrium of joint D [Fig. E3.29(e)]:
Æ≈
ÂF x = 0: S3 cos q = S2
152 Engineering Mechanics

So, S3 –10.32

≠ ÂF y 0: S8 –S3 sin q 8.26

Consider equilibrium of joint F [Fig. E3.29(f )]:


Æ≈

ÂF x 0: S6 S5 + S9 cos a 6.19

Result: Force Magnitude Nature


S1 8.44 kN C
S2 6.19 kN C
S3 10.32 kN C
S4 5.06 kN T
S5 5.06 kN T
S6 6.19 kN T
S7 6.00 kN T
S8 8.26 kN T
S9 1.36 kN T

Example 3.30 Determine the forces in each member of the loaded truss as shown
in Fig. E3.30(a).

Fig. E3.30

Solution Consider equilibrium of joint A [Fig. E3.30(b)]:


Æ≈
S2 S3
ÂF x 0:
2
+
2
0

S2 + S3 0 (1)


S2 S3
≠ ÂF y 0:
2
=
2
+5

S2 – S3 7.07 (2)
Truss, Frames, and Cables 153

Solving Eqs (1) and (2), we obtain S2 3.53 and S3 –3.53.


Consider equilibrium of joint E [Fig. E3.30(c)]:
≈ S2 S7
≠ ÂF y
0:
2
+
2
0

So, S7 –3.53
Æ≈
S2 S7
ÂF x 0:
2
= S1 +
2

So, S1 5.00
Consider equilibrium of joint B [Fig. E3.30(d)]:

S3 S6
≠ ÂF y 0:
2
+
2
0

So, S6 3.53
Æ≈
S3 S6
ÂF x 0:
2
S4 +
2
So, S4 –5.00
From observation of joint F, we obtain, S8 S6 3.53 and S5 S7 –3.53.
Result: Force Magnitude Nature
S1 5.00 kN T
S2 3.53 kN T
S3 3.53 kN C
S4 5.00 kN C
S5 3.53 kN C
S6 3.53 kN T
S7 3.53 kN C
S8 3.53 kN T
Example 3.31 Neglecting any horizontal component of force in the supports,
determine the axial forces in the members of the Pratt roof truss.
Solution Here q tan–1 ÊÁ 3 ˆ˜ 26.57° and a tan–1 ÊÁ 3 ˆ˜ 45°
Ë 6¯ Ë 3¯
Considering FBD of the whole truss [Fig. E3.31(b)], we can say that
RA RB 5 – 1.5/2 3.75 kN
and there will be no horizontal component of force at A.
From observation of joint F, S13 0.
154  Engineering Mechanics

C D

A B
F

Fig. E3.31
Consider equilibrium of joint A [Fig. E3.31(c)]:

≠ ÂF y = 0: S1 sin q + RA = 1.5

Hence, S1 = –5.03
Æ≈
ÂF x
= 0: S8 + S1 cos q = 0

Hence, S8 = 4.5
Consider equilibrium of joint C [Fig. E3.31(d)]:
Æ≈
ÂF x = 0: S2 cos q = S1 cos q

So, S2 = –5.03

≠ ÂF y = 0: S2 sin q = 1.5 + S9 + S1 sin q

So, S9 = –1.5
Consider equilibrium of joint G [Fig. E3.31(e)]:

≠ ÂF y = 0: S11 sin a + S9 = 0

So, S11 = 2.12


Æ≈
ÂF x = 0: S11 cos a + S7 = S8

So, S7 = 3.00
From symmetry of the truss, we can conclude that
S4 = S1 = –5.03, S3 = S2 = –5.03, S5 = S8 = 4.50, S6 = S7 = 3.00
S10 = S9 = –1.50 and S12 = S11 = 2.12
Result: Force Magnitude Nature
S1, S4 5.03 kN C
S2, S3 5.03 kN C
Next Page

Truss, Frames, and Cables 155

S5, S8 4.50 kN T
S6, S7 3.00 kN T
S9, S10 1.50 kN C
S11, S12 2.12 kN T
S13 0

Example 3.32  Neglecting the horizontal component of force in support A, determine


the axial forces induced in all the members of the given truss [Fig. E3.32(a)].
Ê ˆ
Solution  Here, q = tan–1 4 = 53.13°. From DAFC, we see that
Ë 3¯
–CFD = –BCF + –BAF
or q = (90° – q) + a

Fig. E3.32

Therefore, a = 16.26°

Again AC = 32 + 42 = 5 m
CD = AC sin a = 1.4 m
DE = AE cos q = 3.6 m
Consider equilibrium of whole truss and consider moment at A [Fig. E3.32(b)]:

ÂM at A  = 0: – 4.9 ¥ 1.4 + 9.1 ¥ 3.6 – RE ¥ 6 = 0

Therefore, RE = 4.32

≠ ÂFy = 0: RA + RE = (4.9 + 14.0 + 9.1) cos q

Therefore, RA = 12.48

You might also like